OB CBA 2

Réussis tes devoirs et examens dès maintenant avec Quizwiz!

After reviewing the reports of a pregnant patient, the nurse infers that there might be a high risk for intrauterine growth restriction (IUGR). What could be the reason for this? The amniotic fluid index (AFI) is: 1 Less than 5 cm. 2 Equal to or more than 10 cm. 3 Between 5 and 10 cm. 4 More than 25 cm

1 An AFI less than 5 cm indicates oligohydramnios. Oligohydramnios is associated with intrauterine growth restriction and congenital anomalies. An AFI of 10 cm or greater indicates that the fetus is normal. AFI values between 5 and 10 cm are considered low normal, indicating a comparatively low risk for congenital anomalies. An AFI greater than 25 cm indicates polyhydramnios. This is associated with neural tube defects and obstruction of the fetal gastrointestinal tract.

Which physiologic parameters does the nurse check in the ultrasound report to assess fetal well-being? Select all that apply. 1 Amniotic fluid volume (AFV) 2 Fetal breathing movements (FBMs) 3 Fetal limb and head movements 4 Daily count of fetal movements 5 Fluid volume in the nape of the fetal neck

1, 2, 3 Assessment of physiologic parameters such as AFV, FBMs, and limb and head movements of the fetus by ultrasonography gives a reliable picture of fetal well-being. Abnormalities in the amniotic fluid volume are frequently associated with fetal disorders. Fetal breathing and limb and head movements reflect the status of the central nervous system. Daily fetal movement count is the most common method used to assess fetal activity. Ultrasound is not used to assess the daily fetal movement count. The fluid volume in the nape of the fetal neck is measured to assess structural abnormalities in the fetus.

A new mother asks the nurse when the "soft spot" on her son's head will go away. The nurse's answer is based on the knowledge that the anterior fontanel closes after birth by _____ months.

18

After reviewing the Doppler umbilical flow reports of a pregnant patient, the nurse advises the patient to quit smoking immediately. Which finding in the report could be the reason for this instruction? 1 High amniotic fluid volume (AFV) 2 High systolic-to-diastolic (S/D) ratio 3 Low amniotic fluid volume (AFV) 4 Low systolic-to-diastolic (S/D) ratio

2 Exposure to nicotine from maternal smoking has been reported to increase the fetal S/D ratio. An elevated S/D ratio indicates a poorly perfused placenta. To improve the blood supply to the placenta, the patient should quit smoking as soon as possible. The AFV cannot be assessed through Doppler umbilical blood flow study. Moreover, smoking does not affect amniotic fluid volume. Smoking increases the S/D ratio; it does not decrease it.

Arrange the steps the nurse takes while performing transvaginal ultrasonography for a pregnant patient, in the correct order. 1. Cover the transducer probe with a probe cover. 2. Position the pregnant patient in the lithotomy position. 3. Position the probe for proper view of pelvic structures. 4. Lubricate the transducer probe with water-soluble gel. 5. Insert the transducer probe into the patient's vagina.

2, 1, 4, 5, 3 While performing transvaginal ultrasonography, the nurse should first position the patient in the lithotomy position to ensure the optimal view of pelvic structures. The transducer probe is then covered with a suitable probe cover. Then the probe is lubricated with a water-soluble gel to increase penetration of ultrasonic waves. The probe is then inserted into the patient's vagina. Finally, the position of the probe is adjusted for a better view of the inner pelvic structures.

A nonstress test (NST) is ordered on a pregnant woman at 37 weeks of gestation. What are the most appropriate teaching points to include when explaining the procedure to the woman? Select all that apply. 1 After 20 minutes, a nonreactive reading indicates the test is complete. 2 Vibroacoustic stimulation may be used during the test. 3 Drinking orange juice before the test is appropriate. 4 A needle biopsy may be needed to stimulate contractions. 5 Two sensors are placed on the abdomen to measure contractions and fetal heart tones.

2, 3, 5 Vibroacoustic stimulation is often used to stimulate fetal activity if the initial NST result is nonreactive and thus hopefully shortens the time required to complete the test (Greenberg, Druzin, and Gabbe, 2012). A nonreactive test requires further evaluation. The testing period is often extended, usually for an additional 20 minutes, with the expectation that the fetal sleep state will change and the test will become reactive. Care providers sometimes suggest that the woman drink orange juice or be given glucose to increase her blood sugar level and thereby stimulate fetal movements. Although this practice is common, there is no evidence that it increases fetal activity (Greenberg, Druzin, and Gabbe, 2012). A needle biopsy is not part of a NST. The FHR is recorded with a Doppler transducer, and a tocodynamometer is applied to detect uterine contractions or fetal movements. The tracing is observed for signs of fetal activity and a concurrent acceleration of FHR.

While reviewing the ultrasound reports of a patient, the nurse notices a floating fetus in the scanned image. What potential fetal risks should the nurse interpret from this finding? Select all that apply: 1 Renal agenesis 2 Growth restriction 3 Neural tube defects 4 Gastrointestinal obstruction 5 Cardiac disease.

3, 4 A floating fetus is seen in cases of elevated amniotic fluid volume, or polyhydramnios. Polyhydramnios is associated with neural tube defects and gastrointestinal obstruction. Renal agenesis and severe intrauterine growth restriction are associated with oligohydramnios, or low amniotic fluid volume. A low amount of fluid may not result in a floating fetus in the scanned image. The amniotic fluid level is unrelated to cardiac disease in the fetus.

During a prenatal checkup, the patient who is 7 months pregnant reports that she is able to feel about two kicks in an hour. The nurse refers the patient for an ultrasound. What is the primary reason for this referral? To check: 1 For fetal anomalies 2 Gestational age 3 Fetal position 4 For fetal well-being

4 Fetal kick count is a simple method to determine the presence of complications related to fetal oxygenation and activity level. The fetal kick count during the third trimester of pregnancy is approximately 30 kicks an hour; a count lower than that is an indication of poor health of the fetus. Fetal anomalies may not affect the oxygenation levels of the fetus. The nurse already knows the gestational age of the fetus; therefore the nurse need not refer the woman for ultrasonography to find the gestational age. Fetal position does not affect the activity level of the fetus.

A pregnant patient is about to undergo magnetic resonance imaging (MRI). What information does the nurse give the patient before the procedure? The patient will: 1 Be positioned in a lithotomy position. 2 Need to take fluids to have a full bladder. 3 Be able to move freely during the procedure. 4 Not have pain during the process

4 MRI is a noninvasive technique that causes little pain. Therefore the patient undergoing MRI should be advised to not worry about pain. The patient undergoing MRI should be positioned in a supine position. The lithotomy position is not suitable for this procedure. The patient need not take fluids before the scan to ensure a full bladder because this procedure enables a full pelvic view without a full bladder. The patient should be instructed not to move during the scanning process because it may blur the images obtained.

Which occurrence is associated with cervical dilation and effacement? a. Bloody show b. Lightening c. False labor d. Bladder distention

A (As the cervix begins to soften, dilate, and efface, expulsion of the mucous plug that sealed the cervix during pregnancy occurs. This causes rupture of small cervical capillaries. Cervical dilation and effacement do not occur with false labor. Lightening is the descent of the fetus toward the pelvic inlet before labor. Bladder distention occurs when the bladder is not emptied frequently. It may slow down the descent of the fetus during labor.)

Within the first hour after birth, the nurse would expect to find the woman's fundus: a) At the level of the umbilicus b) One fingerbreadth below the umbilicus c) Between the umbilicus and symphysis pubis d) 2 cm above the umbilicus

A (At the level of the umbilicus Rationale: After birth, the fundus is located midline between the umbilicus and symphysis pubis but then slowly rises to the level of the umbilicus during the first hour after birth. Then the uterus contracts, approximately 1 cm (or fingerbreadth) each day after birth.)

A patient with gestational hypertension is prescribed labetalol hydrochloride (Normodyne) therapy, which is continued after giving birth. What does the nurse instruct the patient about breastfeeding? A. "You may breastfeed the infant if you desire." B. "Breastfeeding may cause convulsions in the infant." C. "Breastfeed only once a day and use infant formulas." D. "There may be high levels of the drug in the breast milk."

A (Labetalol hydrochloride (Normodyne) has a low concentration in breast milk, so the patient can breastfeed the infant. Breastfeeding is safe and will not cause convulsions or any side effects in the infant. Infant formulas are used only if the mother is unable to breastfeed the infant or if the mother does not desire to breastfeed.)

Which clinical reports does the nurse evaluate to identify ectopic pregnancy in a patient? Select all that apply. A. Quantitative human chorionic gonadotropin (β-hCG) levels B. Transvaginal ultrasound C. Progesterone level D. Thyroid test reports E. Kleihauer-Betke (KB) test

A, B, C (An ectopic pregnancy is indicated when β-hCG levels are >1500 milli-international units/mL but no intrauterine pregnancy is seen on the transvaginal ultrasound. A transvaginal ultrasound is repeated to verify if the pregnancy is inside the uterus. A progesterone level <5 ng/mL indicates ectopic pregnancy. Thyroid test reports need to be evaluated in case the patient has hyperemesis gravidarum, as hyperthyroidism is associated with this disorder. The KB test is used to determine transplacental hemorrhage.)

39. Most women with uncomplicated pregnancies can use the nurse as their primary source for nutritional information. The nurse or midwife should refer a client to a registered dietitian for in-depth nutritional counseling in the following situations (Select all that apply). a. Preexisting or gestational illness such as diabetes b. Ethnic or cultural food patterns c. Obesity d. Vegetarian diet e. Allergy to tree nuts

A, B, C, D

Which conditions during pregnancy can result in preeclampsia in the patient? Select all that apply. A. Genetic abnormalities B. Dietary deficiencies C. Abnormal trophoblast invasion D. Cardiovascular changes E. Maternal hypotension

A, B, C, D (Current theories consider that genetic abnormalities and dietary deficiencies can result in preeclampsia. Abnormal trophoblast invasion causes fetal hypoxia and results in maternal hypertension. Cardiovascular changes stimulate the inflammatory system and result in preeclampsia in the pregnant patient. Maternal hypertension, and not hypotension, after 20 weeks' gestation is known as preeclampsia.)

Which hypertensive disorders can occur during pregnancy? Select all that apply. A. Chronic hypertension B. Preeclampsia-eclampsia C. Hyperemesis gravidarum D. Gestational hypertension E. Gestational trophoblastic disease

A, B, D (Chronic hypertension refers to hypertension that developed in the pregnant patient before 20 weeks' gestation. Preeclampsia refers to hypertension and proteinuria that develops after 20 weeks' gestation. Eclampsia is the onset of seizure activity in a pregnant patient with preeclampsia. Gestational hypertension is the onset of hypertension after 20 weeks' gestation. Gestational trophoblastic disease and hyperemesis gravidarum are not hypertensive disorders. Gestational trophoblastic disease refers to a disorder without a viable fetus that is caused by abnormal fertilization. Hyperemesis gravidarum is excessive vomiting during pregnancy that may result in weight loss and electrolyte imbalance.)

A nurse is caring for a client in her fourth stage of labor. Which of the following assessments would indicate normal physiologic changes occurring during the fourth stage of labor? Select all that apply. a) Well-contracted uterus in the midline b) Mild uterine cramping and shivering c) Increase in the blood pressure d) Decreased intra-abdominal pressure e) Decrease in the pulse rate

A, B, D (Rationale: The normal physiologic changes for which a nurse should assess during the fourth stage of labor are a well-contracted uterus in the midline of the abdomen, mild cramping pain and generalized shivering, and decreased intra-abdominal pressure. Hemodynamic changes are due to normal blood loss during delivery, causing moderate tachycardia and a slight fall in the blood pressure during the fourth stage of labor. A fall in the pulse rate and increased blood pressure are not normal findings occurring during the fourth stage of labor.)

Which factors influence cervical dilation (Select all that apply)? a. Strong uterine contractions b. The force of the presenting fetal part against the cervix c. The size of the female pelvis d. The pressure applied by the amniotic sac e. Scarring of the cervix

A, B, D, E (Dilation of the cervix occurs by the drawing upward of the musculofibrous components of the cervix, which is caused by strong uterine contractions. Pressure exerted by the amniotic fluid while the membranes are intact or by the force applied by the presenting part also can promote cervical dilation. Scarring of the cervix as a result of a previous infection or surgery may slow cervical dilation. Pelvic size does not affect cervical dilation.)

What does the nurse assess to detect the presence of a hypertensive disorder in a pregnant patient? Select all that apply. A. Proteinuria B. Epigastric pain C. Placenta previa D. Presence of edema E. Blood pressure (BP)

A, B, D, E (Proteinuria indicates hypertension in a pregnant patient. Proteinuria is concentration ≥300 mg/24 hours in a 24-hour urine collection. The nurse needs to assess the patient for epigastric pain because it indicates severe preeclampsia. Hypertension is likely to cause edema or swollen ankles as a result of greater hydrostatic pressure in the lower parts of the body. Therefore the nurse needs to assess the patient for the presence of edema. Accurate measurement of BP will help detect the presence of any hypertensive disorder. A systolic BP greater than 140 mm Hg or a diastolic BP greater than 90 mm Hg will indicate hypertension. Placenta previa is a condition wherein the placenta is implanted in the lower uterine segment covering the cervix, which causes bleeding when the cervix dilates.)

The nurse is caring for a woman who is at 24 weeks of gestation with suspected severe preeclampsia. Which signs and symptoms would the nurse expect to observe? (Select all that apply.) A. Decreased urinary output and irritability B. Transient headache and +1 proteinuria C. Ankle clonus and epigastric pain D. Platelet count of less than 100,000/mm3 and visual problems E. Seizure activity and hypotension

A, C, D (Decreased urinary output and irritability are signs of severe eclampsia. Ankle clonus and epigastric pain are signs of severe eclampsia. Platelet count of less than 100,000/mm3 and visual problems are signs of severe preeclampsia. A transient headache and +1 proteinuria are signs of preeclampsia and should be monitored. Seizure activity and hyperreflexia are signs of eclampsia.)

The nurse is caring for a woman who is at 24 weeks of gestation with suspected severe preeclampsia. Which signs and symptoms should the nurse expect to observe? Select all that apply. A. Decreased urinary output and irritability B. Transient headache and +1 proteinuria C. Ankle clonus and epigastric pain D. Platelet count of less than 100,000/mm3 and visual problems E. Seizure activity and hypotension

A, C, D (Decreased urinary output and irritability are signs of severe eclampsia. Ankle clonus and epigastric pain are signs of severe eclampsia. Platelet count of less than 100,000/mm3 and visual problems are signs of severe preeclampsia. A transient headache and +1 proteinuria are signs of preeclampsia and should be monitored.Seizure activity and hyperreflexia are signs of eclampsia. Test-Taking Tip: Do not worry if you select the same numbered answer repeatedly, because there usually is no pattern to the answers.)

Which opiate causes euphoria, relaxation, drowsiness, and detachment from reality and has possible effects on the pregnancy, including preeclampsia, intrauterine growth restriction, and premature rupture of membranes? A. Heroin B. Alcohol C. Phencyclidine palmitate (PCP) D. Cocaine

A. Heroin The opiates include opium, heroin, meperidine, morphine, codeine, and methadone. The signs and symptoms of heroin use are euphoria, relaxation, relief from pain, detachment from reality, impaired judgment, drowsiness, constricted pupils, nausea, constipation, slurred speech, and respiratory depression. Possible effects on pregnancy include preeclampsia, intrauterine growth restriction, miscarriage, premature rupture of membranes, infections, breech presentation, and preterm labor. Alcohol is not an opiate. PCP is not an opiate. Cocaine is not an opiate.

Thalassemia is a relatively common anemia in which: A. an insufficient amount of hemoglobin is produced to fill the red blood cells (RBCs). B. RBCs have a normal life span but are sickled in shape. C. folate deficiency occurs. D. there are inadequate levels of vitamin B12.

A. an insufficient amount of hemoglobin is produced to fill the red blood cells (RBCs). Thalassemia is a hereditary disorder that involves the abnormal synthesis of the á or â chains of hemoglobin. An insufficient amount of hemoglobin is produced to fill the RBCs. This is the underlying description for sickle cell anemia. Folate deficiency is the most common cause of megaloblastic anemias during pregnancy. B12 deficiency must also be considered if the pregnant woman presents with anemia.

Hypothyroidism occurs in 2 to 3 pregnancies per 1000. Pregnant women with untreated hypothyroidism are at risk for: (Select all that apply.) A. miscarriage. B. macrosomia. C. gestational hypertension. D. placental abruption. E. stillbirth.

A. miscarriage. C. gestational hypertension. D. placental abruption. E. stillbirth. Hypothyroidism is often associated with both infertility and an increased risk of miscarriage. Infants born to mothers with hypothyroidism are more likely to be of low birth weight or preterm. These outcomes can be improved with early diagnosis and treatment. Pregnant women with hypothyroidism are more likely to experience both preeclampsia and gestational hypertension. Placental abruption and stillbirth are risks associated with hypothyroidism. Placental abruption and stillbirth are risks associated with hypothyroidism.

18. As relates to the father's acceptance of the pregnancy and preparation for childbirth, the maternity nurse should know that: A. the father goes through three phases of acceptance of his own. B. the father's attachment to the fetus cannot be as strong as that of the mother because it does not start until after birth. C. in the last 2 months of pregnancy, most expectant fathers suddenly get very protective of their established lifestyle and resist making changes to the home. D. typically men remain ambivalent about fatherhood right up to the birth of their child.

ANS: A A father typically goes through three phases of development to reach acceptance of fatherhood: the announcement phase, the moratorium phase, and the focusing phase. The father-child attachment can be as strong as the mother-child relationship and can also begin during pregnancy. In the last 2 months of pregnancy, many expectant fathers work hard to improve the environment of the home for the child. Typically, the expectant father's ambivalence ends by the first trimester, and he progresses to adjusting to the reality of the situation and then to focusing on his role.

14. A woman who has completed one pregnancy with a fetus (or fetuses) reaching the stage of fetal viability is called a: a. primipara. b. primigravida. c. multipara. d. nulligravida.

ANS: A A primipara is a woman who has completed one pregnancy with a viable fetus. To remember terms, keep in mind: gravida is a pregnant woman; para comes from parity, meaning a viable fetus; primi means first; multi means many; and null means none. A primigravida is a woman pregnant for the first time. A multipara is a woman who has completed two or more pregnancies with a viable fetus. A nulligravida is a woman who has never been pregnant.

24. Before the physician performs an external version, the nurse should expect an order for a: a. tocolytic drug. b. contraction stress test (CST). c. local anesthetic. d. Foley catheter.

ANS: A A tocolytic drug will relax the uterus before and during version, thus making manipulation easier. CST is used to determine the fetal response to stress. A local anesthetic is not used with external version. The bladder should be emptied; however, catheterization is not necessary.

7. A woman is undergoing a nipple-stimulated contraction stress test (CST). She is having contractions that occur every 3 minutes. The fetal heart rate (FHR) has a baseline of approximately 120 beats/min without any decelerations. The interpretation of this test is said to be: a. negative. b. positive. c. satisfactory. d. unsatisfactory.

ANS: A Adequate uterine activity necessary for a CST consists of the presence of three contractions in a 10-minute time frame. If no decelerations are observed in the FHR pattern with the contractions, the findings are considered to be negative. A positive CST indicates the presence of repetitive later FHR decelerations. Satisfactory and unsatisfactory are not applicable terms.

7. Individual irregularities in the ovarian (menstrual) cycle are most often caused by: A. variations in the follicular (preovulatory) phase. B. an intact hypothalamic-pituitary feedback mechanism. C. a functioning corpus luteum. D. a prolonged ischemic phase.

ANS: A Almost all variations in the length of the ovarian cycle are the result of variations in the length of the follicular phase. An intact hypothalamic-pituitary feedback mechanism is regular, not irregular. The luteal phase begins after ovulation. The corpus luteum depends on the ovulatory phase and fertilization. During the ischemic phase, the blood supply to the functional endometrium is blocked and necrosis develops. The functional layer separates from the basal layer, and menstrual bleeding begins.

31. A pregnant patient would like to know a good food source of calcium other than dairy products. Your best answer is: A. legumes. B. yellow vegetables. C. lean meat. D. whole grains.

ANS: A Although dairy products contain the greatest amount of calcium, it is also found in legumes, nuts, dried fruits, and some dark green leafy vegetables. Yellow vegetables are rich in vitamin A. Lean meats are rich in protein and phosphorus. Whole grains are rich in zinc and magnesium.

2. A woman at 10 weeks of gestation who is seen in the prenatal clinic with presumptive signs and symptoms of pregnancy likely will have: A. amenorrhea. B. positive pregnancy test. C. Chadwick's sign. D. Hegar's sign.

ANS: A Amenorrhea is a presumptive sign of pregnancy. Presumptive signs of pregnancy are felt by the woman. A positive pregnancy test, the presence of Chadwick's sign, and the presence of Hegar's sign are all probable signs of pregnancy.

25. The nurse providing newborn stabilization must be aware that the primary side effect of maternal narcotic analgesia in the newborn is: a. respiratory depression. b. bradycardia. c. acrocyanosis. d. tachypnea.

ANS: A An infant delivered within 1 to 4 hours of maternal analgesic administration is at risk for respiratory depression from the sedative effects of the narcotic. Bradycardia is not the anticipated side effect of maternal analgesics. Acrocyanosis is an expected finding in a newborn and is not related to maternal analgesics. The infant who is having a side effect to maternal analgesics normally would have a decrease in respirations, not an increase.

2. A 39-year-old primigravida thinks that she is about 8 weeks pregnant, although she has had irregular menstrual periods all her life. She has a history of smoking approximately one pack of cigarettes a day, but she tells you that she is trying to cut down. Her laboratory data are within normal limits. What diagnostic technique could be used with this pregnant woman at this time? A. Ultrasound examination B. Maternal serum alpha-fetoprotein (MSAFP) screening C. Amniocentesis D. Nonstress test (NST)

ANS: A An ultrasound examination could be done to confirm the pregnancy and determine the gestational age of the fetus. It is too early in the pregnancy to perform MSAFP screening, amniocentesis, or NST. MSAFP screening is performed at 16 to 18 weeks of gestation, followed by amniocentesis if MSAFP levels are abnormal or if fetal/maternal anomalies are detected. NST is performed to assess fetal well-being in the third trimester.

4. A woman in preterm labor at 30 weeks of gestation receives two 12-mg doses of betamethasone intramuscularly. The purpose of this pharmacologic treatment is to: a. stimulate fetal surfactant production. b. reduce maternal and fetal tachycardia associated with ritodrine administration. c. suppress uterine contractions. d. maintain adequate maternal respiratory effort and ventilation during magnesium sulfate therapy.

ANS: A Antenatal glucocorticoids given as intramuscular injections to the mother accelerate fetal lung maturity. Inderal would be given to reduce the effects of ritodrine administration. Betamethasone has no effect on uterine contractions. Calcium gluconate would be given to reverse the respiratory depressive effects of magnesium sulfate therapy.

6. A key finding from the Human Genome Project is: A. approximately 20,500 genes make up the genome. B. all human beings are 80.99% identical at the DNA level. C. human genes produce only one protein per gene; other mammals produce three proteins per gene. D. single gene testing will become a standardized test for all pregnant patients in the future.

ANS: A Approximately 20,500 genes make up the human genome; this is only twice as many as make up the genomes of roundworms and flies. Human beings are 99.9% identical at the DNA level. Most human genes produce at least three proteins. Single gene testing (e.g., alpha-fetoprotein) is already standardized for prenatal care.

15. The nurse caring for the laboring woman should know that meconium is produced by: A. fetal intestines. B. fetal kidneys. C. amniotic fluid. D. the placenta.

ANS: A As the fetus nears term, fetal waste products accumulate in the intestines as dark green-to-black, tarry meconium.

5. During a patient's physical examination the nurse notes that the lower uterine segment is soft on palpation. The nurse would document this finding as: A. Hegar's sign. B. McDonald's sign. C. Chadwick's sign. D. Goodell's sign.

ANS: A At approximately 6 weeks of gestation, softening and compressibility of the lower uterine segment occurs; this is called Hegar's sign. McDonald's sign indicates a fast food restaurant. Chadwick's sign is the blue-violet coloring of the cervix caused by increased vascularity; this occurs around the fourth week of gestation. Softening of the cervical tip is called Goodell's sign, which may be observed around the sixth week of pregnancy.

20. With regard to the process of augmentation of labor, the nurse should be aware that it: a. is part of the active management of labor that is instituted when the labor process is unsatisfactory. b. relies on more invasive methods when oxytocin and amniotomy have failed. c. is a modern management term to cover up the negative connotations of forceps-assisted birth. d. uses vacuum cups.

ANS: A Augmentation is part of the active management of labor that stimulates uterine contractions after labor has started but is not progressing satisfactorily. Augmentation uses amniotomy and oxytocin infusion, as well as some gentler, noninvasive methods. Forceps-assisted births and vacuum-assisted births are appropriately used at the end of labor and are not part of augmentation.

24. With regard to medications, herbs, shots, and other substances normally encountered by pregnant women, the maternity nurse should be aware that: A. both prescription and over-the-counter (OTC) drugs that otherwise are harmless can be made hazardous by metabolic deficiencies of the fetus. B. the greatest danger of drug-caused developmental deficits in the fetus is seen in the final trimester. C. killed-virus vaccines (e.g., tetanus) should not be given during pregnancy, but live-virus vaccines (e.g., measles) are permissible. D. no convincing evidence exists that secondhand smoke is potentially dangerous to the fetus.

ANS: A Both prescription and OTC drugs that otherwise are harmless can be made hazardous by metabolic deficiencies of the fetus. This is especially true for new medications and combinations of drugs. The greatest danger of drug-caused developmental defects exists in the interval from fertilization through the first trimester, when a woman may not realize that she is pregnant. Live-virus vaccines should be part of after birth care; killed-virus vaccines may be administered during pregnancy. Secondhand smoke is associated with fetal growth restriction and increases in infant mortality.

12. Compared with contraction stress test (CST), nonstress test (NST) for antepartum fetal assessment: A. has no known contraindications. B. has fewer false-positive results. C. is more sensitive in detecting fetal compromise. D. is slightly more expensive.

ANS: A CST has several contraindications. NST has a high rate of false-positive results, is less sensitive than the CST, and is relatively inexpensive.

21. As related to the care of the patient with anemia, the nurse should be aware that: A. it is the most common medical disorder of pregnancy. B. it can trigger reflex brachycardia. C. the most common form of anemia is caused by folate deficiency. D. thalassemia is a European version of sickle cell anemia.

ANS: A Combined with any other complication, anemia can result in congestive heart failure. Reflex bradycardia is a slowing of the heart in response to the blood flow increases immediately after birth. The most common form of anemia is iron deficiency anemia. Both thalassemia and sickle cell hemoglobinopathy are hereditary but not directly related or confined to geographic areas.

18. A normal uterine activity pattern in labor is characterized by: a. contractions every 2 to 5 minutes. b. contractions lasting about 2 minutes. c. contractions about 1 minute apart. d. a contraction intensity of about 1000 mm Hg with relaxation at 50 mm Hg.

ANS: A Contractions normally occur every 2 to 5 minutes and last less than 90 seconds (intensity 800 mm Hg) with about 30 seconds in between (20 mm Hg or less).

28. When the pregnant diabetic woman experiences hypoglycemia while hospitalized, the nurse should intervene by having the patient: A. eat six saltine crackers. B. drink 8 ounces of orange juice with 2 tsp of sugar added. C. drink 4 ounces of orange juice followed by 8 ounces of milk. D. eat hard candy or commercial glucose wafers.

ANS: A Crackers provide carbohydrates in the form of polysaccharides. Orange juice and sugar will increase the blood sugar but not provide a slow-burning carbohydrate to sustain the blood sugar. Milk is a disaccharide and orange juice is a monosaccharide. They will provide an increase in blood sugar but will not sustain the level. Hard candy or commercial glucose wafers provide only monosaccharides.

15. Which statement regarding acronyms in nutrition is accurate? A. Dietary reference intakes (DRIs) consist of recommended dietary allowances (RDAs), adequate intakes (AIs), and upper limits (ULs). B. RDAs are the same as ULs except with better data. C. AIs offer guidelines for avoiding excessive amounts of nutrients. D. They all refer to green leafy vegetables, whole grains, and fruit.

ANS: A DRIs consist of RDAs, AIs, and ULs. AIs are similar to RDAs except that they deal with nutrients about which data are insufficient for certainty (RDA status). ULs are guidelines for avoiding excesses of nutrients for which excess is toxic. Green leafy vegetables, whole grains, and fruit are important, but they are not the whole nutritional story.

16. Which statement would indicate that the patient requires additional instruction about breast self-examination? A. "Yellow discharge from my nipple is normal if I am having my period." B. "I should check my breasts at the same time each month, like after my period." C. "I should also feel in my armpit area while performing my breast examination." D. "I should check each breast in a set way, such as in a circular motion."

ANS: A Discharge from the nipples requires further examination from a health care provider. "I should check my breasts at the same time each month, like after my period," "I should also feel in my armpit area while performing my breast examination," and "I should check each breast in a set way, such as in a circular motion" all indicate successful learning.

36. Fibrocystic changes in the breast most often appear in women in their 20s and 30s. The etiology is unknown but it may be an imbalance of estrogen and progesterone. The nurse who cares for this patient should be aware that treatment modalities are conservative. One proven modality that may provide relief is: A. diuretic administration. B. including caffeine daily in the diet. C. increased vitamin C supplementation. D. application of cold packs to the breast as necessary.

ANS: A Diuretic administration plus a decrease in sodium and fluid intake are recommended. Although not supported by research, some advocate eliminating dimethylxanthines (caffeine) from the diet. Smoking should also be avoided, and alcohol consumption should be reduced. Vitamin E supplements are recommended; however, the patient should avoid megadoses because this is a fat-soluble vitamin. Pain-relief measures include applying heat to the breast, wearing a supportive bra, and taking nonsteroidal anti-inflammatory drugs.

27. Postcoital contraception with Ella: A. requires that the first dose be taken within 72 hours of unprotected intercourse. B. requires that the woman take second and third doses at 24 and 36 hours after the first dose. C. must be taken in conjunction with an IUD insertion. D. is commonly associated with the side effect of menorrhagia.

ANS: A Emergency contraception is most effective when used within 72 hours of intercourse; however, it may be used with lessened effectiveness 120 hours later. Insertion of the copper IUD within 5 days of intercourse may also be used and is up to 99% effective. The most common side effect of postcoital contraception is nausea.

3. The nurse providing care for the laboring woman comprehends that accelerations with fetal movement: a. are reassuring. b. are caused by umbilical cord compression. c. warrant close observation. d. are caused by uteroplacental insufficiency.

ANS: A Episodic accelerations in the fetal heart rate (FHR) occur during fetal movement and are indications of fetal well-being. Umbilical cord compression results in variable decelerations in the FHR. Accelerations in the FHR are an indication of fetal well-being and do not warrant close observation. Uteroplacental insufficiency would result in late decelerations in the FHR.

29. The placenta allows exchange of oxygen, nutrients, and waste products between the mother and fetus by: A. contact between maternal blood and fetal capillaries within the chorionic villi. B. interaction of maternal and fetal pH levels within the endometrial vessels. C. a mixture of maternal and fetal blood within the intervillous spaces. D. passive diffusion of maternal carbon dioxide and oxygen into the fetal capillaries.

ANS: A Fetal capillaries within the chorionic villi are bathed with oxygen-rich and nutrient-rich maternal blood within the intervillous spaces. The endometrial vessels are part of the uterus. There is no interaction with the fetal blood at this point. Maternal and fetal bloods do not normally mix. Maternal carbon dioxide does not enter into the fetal circulation.

14. Which description of the four stages of labor is correct for both the definition and the duration? a. First stage: onset of regular uterine contractions to full dilation; less than 1 hour to 20 hours b. Second stage: full effacement to 4 to 5 cm; visible presenting part; 1 to 2 hours c. Third stage: active pushing to birth; 20 minutes (multiparous woman), 50 minutes (nulliparous woman) d. Fourth stage: delivery of the placenta to recovery; 30 minutes to 1 hour

ANS: A Full dilation may occur in less than 1 hour, but in first-time pregnancies full dilation can take up to 20 hours. The second stage of labor extends from full dilation to birth and takes an average of 20 to 50 minutes, although 2 hours is still considered normal. The third stage of labor extends from birth to the expulsion of the placenta and usually takes a few minutes. The fourth stage begins after the expulsion of the placenta and lasts until homeostasis is reestablished (approximately 2 hours).

16. With regard to protein in the diet of pregnant women, nurses should be aware that: A. many protein-rich foods are also good sources of calcium, iron, and B vitamins. B. many women need to increase their protein intake during pregnancy. C. as with carbohydrates and fat, no specific recommendations exist for the amount of protein in the diet. D. high-protein supplements can be used without risk by women on macrobiotic diets.

ANS: A Good protein sources such as meat, milk, eggs, and cheese have a lot of calcium and iron. Most women already eat a high-protein diet and do not need to increase their intake. Protein is sufficiently important that specific servings of meat and dairy are recommended. High-protein supplements are not recommended because they have been associated with an increased incidence of preterm births.

22. Leopold maneuvers would be an inappropriate method of assessment to determine: a. gender of the fetus. b. number of fetuses. c. fetal lie and attitude. d. degree of the presenting part's descent into the pelvis.

ANS: A Leopold maneuvers help identify the number of fetuses, the fetal lie and attitude, and the degree of descent of the presenting part into the pelvis. The gender of the fetus is not a goal of the examination at this time.

25. Which viral sexually transmitted infection is characterized by a primary infection followed by recurrent episodes? A. Herpes simplex virus (HSV)-2 B. Human papillomavirus (HPV) C. Human immunodeficiency virus (HIV) D. Cytomegalovirus (CMV)

ANS: A The initial HSV genital infection is characterized by multiple painful lesions, fever, chills, malaise, and severe dysuria; it may last 2 to 3 weeks. Recurrent episodes of HSV infection commonly have only local symptoms that usually are less severe than the symptoms of the initial infection. With HPV infection, lesions are a chronic problem. HIV is a retrovirus. Seroconversion to HIV positivity usually occurs within 6 to 12 weeks after the virus has entered the body. Severe depression of the cellular immune system associated with HIV infection characterizes acquired immunodeficiency syndrome (AIDS). AIDS has no cure. In most adults, the onset of CMV infection is uncertain and asymptomatic. However, the disease may become a chronic, persistent infection.

9. With regard to the association of maternal diabetes and other risk situations affecting mother and fetus, nurses should be aware that: A. Diabetic ketoacidosis (DKA) can lead to fetal death at any time during pregnancy. B. Hydramnios occurs approximately twice as often in diabetic pregnancies. C. Infections occur about as often and are considered about as serious in diabetic and nondiabetic pregnancies. D. Even mild to moderate hypoglycemic episodes can have significant effects on fetal well-being.

ANS: A Prompt treatment of DKA is necessary to save the fetus and the mother. Hydramnios occurs 10 times more often in diabetic pregnancies. Infections are more common and more serious in pregnant women with diabetes. Mild-to-moderate hypoglycemic episodes do not appear to have significant effects on fetal well-being.

1. Intrauterine growth restriction (IUGR) is associated with numerous pregnancy-related risk factors. (Select all that apply.) A. Poor material weight gain B. Chronic maternal infections C. Gestational hypertension D. Premature rupture of membranes E. Smoking

ANS: A, B, C, E Poor material weight gain, chronic infections disease, gestational hypertension, and smoking are all risk factors associated with IUGR. Premature rupture of membranes is associated with preterm labor, not IUGR.

2. Which congenital malformations result from multifactorial inheritance? (Select all that apply.) A. Cleft lip B. Congenital heart disease C. Cri du chat syndrome D. Anencephaly E. Pyloric stenosis

ANS: A, B, D, E All these congenital malformations are associated with multifactorial inheritance. Cri du chat syndrome is related to a chromosome deletion.

2. A woman has just moved to the United States from Mexico. She is 3 months pregnant and has arrived for her first prenatal visit. During her assessment interview, you discover that she has not had any immunizations. Which immunizations should she receive at this point in her pregnancy? (Select all that apply.) A. Tetanus B. Diphtheria C. Chickenpox D. Rubella E. Hepatitis B

ANS: A, B, E Immunization with live or attenuated live viruses is contraindicated during pregnancy because of potential teratogenicity. Vaccines consisting of killed viruses may be used. Immunizations that may be administered during pregnancy include tetanus, diphtheria, recombinant hepatitis B, and rabies vaccines. Live-virus vaccines include those for measles (rubeola and rubella), chickenpox, and mumps.

1. The diagnosis of pregnancy is based on which positive signs of pregnancy? (Select all that apply.) A. Identification of fetal heartbeat B. Palpation of fetal outline C. Visualization of the fetus D. Verification of fetal movement E. Positive hCG test

ANS: A, C, D Identification of fetal heartbeat, visualization of the fetus, and verification of fetal movement are all positive, objective signs of pregnancy. Palpation of fetal outline and a positive hCG test are probable signs of pregnancy. A tumor also can be palpated. Medication and tumors may lead to false-positive results on pregnancy tests.

1. Which changes take place in the woman's reproductive system, days or even weeks before the commencement of labor? (Select all that apply.) a. Lightening b. Exhaustion c. Bloody show d. Rupture of membranes e. Decreased fetal movement

ANS: A, C, D Signs that precede labor may include lightening, urinary frequency, backache, weight loss, surge of energy, bloody show, and rupture of membranes. Many women experience a burst of energy before labor. A decrease in fetal movement is an ominous sign that does not always correlate with labor.

3. In planning for the care of a 30-year-old woman with pregestational diabetes, the nurse recognizes that the most important factor affecting pregnancy outcome is the: A. mother's age. B. number of years since diabetes was diagnosed. C. amount of insulin required prenatally. D. degree of glycemic control during pregnancy.

ANS: D Women with excellent glucose control and no blood vessel disease should have good pregnancy outcomes.

27. What correctly matches the type of deceleration with its likely cause? a. Early deceleration—umbilical cord compression b. Late deceleration—uteroplacental inefficiency c. Variable deceleration—head compression d. Prolonged deceleration—cause unknown

ANS: B Late deceleration is caused by uteroplacental inefficiency. Early deceleration is caused by head compression. Variable deceleration is caused by umbilical cord compression. Prolonged deceleration has a variety of either benign or critical causes.

Assessment reveals that a woman's cervix is approximately 1 cm in length. The nurse would document this as: a) 100% effaced b) 50% effaced c) 0% effaced d) 75% effaced

B (50% effaced Rationale: A cervix 1 cm in length is described as 50% effaced. A cervix that measures approximately 2 cm in length is described as 0% effaced. A cervix 1/2 cm in length would be described as 75% effaced. A cervix 0 cm in length would be described as 100% effaced.)

2. The nurse providing care for a woman with preterm labor who is receiving terbutaline would include which intervention to identify side effects of the drug? a. Assessing deep tendon reflexes (DTRs) b. Assessing for chest discomfort and palpitations c. Assessing for bradycardia d. Assessing for hypoglycemia

ANS: B Terbutaline is a β2-adrenergic agonist that affects the cardiopulmonary and metabolic systems of the mother. Signs of cardiopulmonary decompensation would include chest pain and palpitations. Assessing DTRs would not address these concerns. β2-Adrenergic agonist drugs cause tachycardia, not bradycardia. The metabolic effect leads to hyperglycemia, not hypoglycemia.

5. The nurse should be aware that an effective plan to achieve adequate pain relief without maternal risk is most effective if: a. the mother gives birth without any analgesic or anesthetic. b. the mother and family's priorities and preferences are incorporated into the plan. c. the primary health care provider decides the best pain relief for the mother and family. d. the nurse informs the family of all alternative methods of pain relief available in the hospital setting.

ANS: B The assessment of the woman, her fetus, and her labor is a joint effort of the nurse and the primary health care providers, who consult with the woman about their findings and recommendations. The needs of each woman are different and many factors must be considered before a decision is made whether pharmacologic methods, nonpharmacologic methods, or a combination of the two will be used to manage labor pain.

34. A patient whose cervix is dilated to 5 cm is considered to be in which phase of labor? a. Latent phase b. Active phase c. Second stage d. Third stage

ANS: B The latent phase is from the beginning of true labor until 3 cm of cervical dilation. The active phase of labor is characterized by cervical dilation of 4 to 7 cm. The second stage of labor begins when the cervix is completely dilated until the birth of the baby. The third stage of labor is from the birth of the baby until the expulsion of the placenta. This patient is in the active phase of labor.

28. Nurses alert to signs of the onset of the second stage of labor can be certain that this stage has begun when: a. the woman has a sudden episode of vomiting. b. the nurse is unable to feel the cervix during a vaginal examination. c. bloody show increases. d. the woman involuntarily bears down.

ANS: B The only certain objective sign that the second stage has begun is the inability to feel the cervix because it is fully dilated and effaced. Vomiting, an increase in bloody show, and involuntary bearing down are only suggestions of second-stage labor.

30. To assist the woman after delivery of the infant, the nurse knows that the blood patch is used after spinal anesthesia to relieve: a. hypotension. b. headache. c. neonatal respiratory depression. d. loss of movement.

ANS: B The subarachnoid block may cause a postspinal headache resulting from loss of cerebrospinal fluid from the puncture in the dura. When blood is injected into the epidural space in the area of the dural puncture, it forms a seal over the hole to stop leaking of cerebrospinal fluid. Hypotension is prevented by increasing fluid volume before the procedure. Neonatal respiratory depression is not an expected outcome with spinal anesthesia. Loss of movement is an expected outcome of spinal anesthesia.

1. Maternal hypotension is a potential side effect of regional anesthesia and analgesia. What nursing interventions could you use to raise the patient's blood pressure? (Select all that apply.) a. Place the woman in a supine position. b. Place the woman in a lateral position. c. Increase intravenous (IV) fluids. d. Administer oxygen. e. Perform a vaginal examination.

ANS: B, C, D Nursing interventions for maternal hypotension arising from analgesia or anesthesia include turning the woman to a lateral position, increasing IV fluids, administering oxygen via face mask, elevating the woman's legs, notifying the physician, administering an IV vasopressor, and monitoring the maternal and fetal status at least every 5 minutes until these are stable. Placing the patient in a supine position would cause venous compression, thereby limiting blood flow to and oxygenation of the placenta and fetus. A sterile vaginal examination has no bearing on maternal blood pressure.

3. Autoimmune disorders often occur during pregnancy because a large percentage of women with an autoimmune disorder are of childbearing age. Identify all disorders that fall into the category of collagen vascular disease. A. Multiple sclerosis B. Systemic lupus erythematosus C. Antiphospholipid syndrome D. Rheumatoid arthritis E. Myasthenia gravis

ANS: B, C, D, E Multiple sclerosis is not an autoimmune disorder. This patchy demyelinization of the spinal cord may be a viral disorder. Autoimmune disorders (collagen vascular disease) make up a large group of conditions that disrupt the function of the immune system of the body. They include those listed, as well as systemic sclerosis.

1. Complications and risks associated with cesarean births include: (Select all that apply.) a. placental abruption. b. wound dehiscence. c. hemorrhage. d. urinary tract infections. e. fetal injuries.

ANS: B, C, D, E Placental abruption and placenta previa are both indications for cesarean birth and are not complications thereof. Wound dehiscence, hemorrhage, urinary tract infection, and fetal injuries are all possible complications and risks associated with delivery by cesarean section.

11. To prevent gastrointestinal upset, patients should be instructed to take iron supplements: A. on a full stomach. B. at bedtime. C. after eating a meal. D. with milk.

ANS: BPatients should be instructed to take iron supplements at bedtime. Iron supplements are best absorbed if they are taken when the stomach is empty. Bran, tea, coffee, milk, and eggs may reduce absorption. Iron can be taken at bedtime if abdominal discomfort occurs when it is taken between meals.

38. Which patient is most at risk for fibroadenoma of the breast? A. A 38-year-old woman B. A 50-year-old woman C. A 16-year-old girl D. A 27-year-old woman

ANS: C Although it may occur at any age, fibroadenoma is most common in the teenage years. Ductal ectasia and intraductal papilloma become more common as a woman approaches menopause. Fibrocystic breast changes are more common during the reproductive years.

3. A laboring woman received an opioid agonist (meperidine) intravenously 90 minutes before she gave birth. Which medication should be available to reduce the postnatal effects of Demerol on the neonate? a. Fentanyl (Sublimaze) b. Promethazine (Phenergan) c. Naloxone (Narcan) d. Nalbuphine (Nubain)

ANS: C An opioid antagonist can be given to the newborn as one part of the treatment for neonatal narcosis, which is a state of central nervous system (CNS) depression in the newborn produced by an opioid. Opioid antagonists such as naloxone (Narcan) can promptly reverse the CNS depressant effects, especially respiratory depression. Fentanyl, promethazine, and nalbuphine do not act as opioid antagonists to reduce the postnatal effects of Demerol on the neonate. Although meperidine (Demerol) is a low-cost medication and readily available, the use of Demerol in labor has been controversial because of its effects on the neonate.

15. As relates to the use of tocolytic therapy to suppress uterine activity, nurses should be aware that: a. the drugs can be given efficaciously up to the designated beginning of term at 37 weeks. b. there are no important maternal (as opposed to fetal) contraindications. c. its most important function is to afford the opportunity to administer antenatal glucocorticoids. d. if the patient develops pulmonary edema while receiving tocolytics, intravenous (IV) fluids should be given.

ANS: C Buying time for antenatal glucocorticoids to accelerate fetal lung development may be the best reason to use tocolytics. Once the pregnancy has reached 34 weeks, the risks of tocolytic therapy outweigh the benefits. There are important maternal contraindications to tocolytic therapy. Tocolytic-induced edema can be caused by IV fluids.

17. While working with the pregnant woman in her first trimester, the nurse is aware that chorionic villus sampling (CVS) can be performed during pregnancy at: A. 4 weeks. B. 8 weeks. C. 10 weeks. D. 14 weeks.

ANS: C CVS can be performed in the first or second trimester, ideally between 10 and 13 weeks of gestation. During this procedure, a small piece of tissue is removed from the fetal portion of the placenta. If performed after 9 completed weeks of gestation, the risk of limb reduction is no greater than in the general population.

39. The drug of choice for treatment of gonorrhea is: A. penicillin G. B. tetracycline. C. ceftriaxone. D. acyclovir.

ANS: C Ceftriaxone is effective for treatment of all gonococcal infections. Penicillin is used to treat syphilis. Tetracycline is prescribed for chlamydial infections. Acyclovir is used to treat herpes genitalis.

23. Which nursing assessment indicates that a woman who is in second-stage labor is almost ready to give birth? a. Fetal head is felt at 0 station during the vaginal examination. b. Bloody mucous discharge increases. c. Vulva bulges and encircles the fetal head. d. Membranes rupture during a contraction.

ANS: C During the active pushing (descent) phase, the woman has strong urges to bear down as the presenting part of the fetus descends and presses on the stretch receptors of the pelvic floor. The vulva stretches and begins to bulge, encircling the fetal head. Birth of the head occurs when the station is +4. A 0 station indicates engagement. Bloody show occurs throughout the labor process and is not an indication of an imminent birth. Rupture of membranes can occur at any time during the labor process and does not indicate an imminent birth.

27. To manage her diabetes appropriately and ensure a good fetal outcome, the pregnant woman with diabetes will need to alter her diet by: A. eating six small equal meals per day. B. reducing carbohydrates in her diet. C. eating her meals and snacks on a fixed schedule. D. increasing her consumption of protein.

ANS: C Having a fixed meal schedule will provide the woman and the fetus with a steadier blood sugar level, provide better balance with insulin administration, and help prevent complications. It is more important to have a fixed meal schedule than equal division of food intake. Approximately 45% of the food eaten should be in the form of carbohydrates.

28. The laboring woman who imagines her body opening to let the baby out is using a mental technique called: a. dissociation. b. effleurage. c. imagery. d. distraction.

ANS: C Imagery is a technique of visualizing images that will assist the woman in coping with labor. Dissociation helps the woman learn to relax all muscles except those that are working. Effleurage is self-massage. Distraction can be used in the early latent phase by having the woman engage in another activity.

25. The nurse caring for the woman in labor should understand that increased variability of the fetal heart rate may be caused by: a. narcotics. b. barbiturates. c. methamphetamines. d. tranquilizers.

ANS: C Narcotics, barbiturates, and tranquilizers may be causes of decreased variability; methamphetamines may cause increased variability.

24. Because the risk for childbirth complications may be revealed, nurses should know that the point of maximal intensity (PMI) of the fetal heart tone (FHT) is: a. usually directly over the fetal abdomen. b. in a vertex position heard above the mother's umbilicus. c. heard lower and closer to the midline of the mother's abdomen as the fetus descends and rotates internally. d. in a breech position heard below the mother's umbilicus.

ANS: C Nurses should be prepared for the shift. The PMI of the FHT usually is directly over the fetal back. In a vertex position it is heard below the mother's umbilicus. In a breech position it is heard above the mother's umbilicus.

7. In practical terms regarding genetic health care, nurses should be aware that: A. genetic disorders affect people of all socioeconomic backgrounds, races, and ethnic groups equally. B. genetic health care is more concerned with populations than individuals. C. the most important of all nursing functions is providing emotional support to the family during counseling. D. taking genetic histories is the province of large universities and medical centers.

ANS: C Nurses should be prepared to help with various stress reactions from a couple facing the possibility of a genetic disorder. Although anyone may have a genetic disorder, certain disorders appear more often in certain ethnic and racial groups. Genetic health care is highly individualized because treatments are based on the phenotypic responses of the individual. Individual nurses at any facility can take a genetic history, although larger facilities may have better support services.

31. A woman who is older than 35 years may have difficulty achieving pregnancy primarily because: A. personal risk behaviors influence fertility. B. she has used contraceptives for an extended time. C. her ovaries may be affected by the aging process. D. prepregnancy medical attention is lacking.

ANS: C Once the mature woman decides to conceive, a delay in becoming pregnant may occur because of the normal aging of the ovaries. Older adults participate in fewer risk behaviors than younger adults. The past use of contraceptives is not the problem. Prepregnancy medical care is both available and encouraged.

26. The nurse should know that once human immunodeficiency virus (HIV) enters the body, seroconversion to HIV positivity usually occurs within: A. 6 to 10 days. B. 2 to 4 weeks. C. 6 to 12 weeks. D. 6 months.

ANS: C Seroconversion to HIV positivity usually occurs within 6 to 12 weeks after the virus has entered the body.

12. With regard to endometriosis, nurses should be aware that: A. it is characterized by the presence and growth of endometrial tissue inside the uterus. B. it is found more often in African-American women than in white or Asian women. C. it may worsen with repeated cycles or remain asymptomatic and disappear after menopause. D. it is unlikely to affect sexual intercourse or fertility.

ANS: C Symptoms vary among women, ranging from nonexistent to incapacitating. With endometriosis, the endometrial tissue is outside the uterus. Symptoms vary among women, ranging from nonexistent to incapacitating. Endometriosis is found equally in white and African-American women and is slightly more prevalent in Asian women. Women can experience painful intercourse and impaired fertility.

12. A woman's cousin gave birth to an infant with a congenital heart anomaly. The woman asks the nurse when such anomalies occur during development. Which response by the nurse is most accurate? A. "We don't really know when such defects occur." B. "It depends on what caused the defect." C. "They occur between the third and fifth weeks of development." D. "They usually occur in the first 2 weeks of development."

ANS: C The cardiovascular system is the first organ system to function in the developing human. Blood vessel and blood formation begins in the third week, and the heart is developmentally complete in the fifth week. "We don't really know when such defects occur" is an inaccurate statement. Regardless of the cause, the heart is vulnerable during its period of development, the third to fifth weeks. "They usually occur in the first 2 weeks of development" is an inaccurate statement.

21. While taking a diet history, the nurse might be told that the expectant mother has cravings for ice chips, cornstarch, and baking soda. This represents a nutritional problem known as: a. preeclampsia. b. pyrosis. c. pica. d. purging.

ANS: C The consumption of foods low in nutritional value or of nonfood substances (e.g., dirt, laundry starch) is called pica.

10. The nurse should be aware that the partner's main role in pregnancy is to: A. provide financial support. B. protect the pregnant woman from "old wives' tales." C. support and nurture the pregnant woman. D. make sure the pregnant woman keeps prenatal appointments.

ANS: C The partner's main role in pregnancy is to nurture the pregnant woman and to respond her feelings of vulnerability. In older societies, the man enacted the ritual couvade. Changing cultural and professional attitudes have encouraged fathers' participation in the birth experience over the past 30 years.

22. A woman in labor is breathing into a mouthpiece just before the start of her regular contractions. As she inhales, a valve opens, and gas is released. She continues to inhale the gas slowly and deeply until the contraction starts to subside. When the inhalation stops, the valve closes. This procedure is: a. not used much anymore. b. likely to be used in the second stage of labor but not in the first stage. c. an application of nitrous oxide. d. a prelude to cesarean birth.

ANS: C This is an application of nitrous oxide, which could be used in either the first or second stage of labor (or both) as part of the preparation for a vaginal birth. Nitrous oxide is self-administered and found to be very helpful.

20. A woman had unprotected intercourse 36 hours ago and is concerned that she may become pregnant because it is her "fertile" time. She asks the nurse about emergency contraception. The nurse tells her that: A. it is too late; she needed to begin treatment within 24 hours after intercourse. B. Preven, an emergency contraceptive method, is 98% effective at preventing pregnancy. C. an over-the-counter antiemetic can be taken 1 hour before each contraceptive dose to prevent nausea and vomiting. D. the most effective approach is to use a progestin-only preparation.

ANS: C To minimize the side effect of nausea that occurs with high doses of estrogen and progestin, the woman can take an over-the-counter antiemetic 1 hour before each dose. Emergency contraception is used within 72 hours of unprotected intercourse to prevent pregnancy. Postcoital contraceptive use is 74% to 90% effective at preventing pregnancy. Oral emergency contraceptive regimens may include progestin-only and estrogen-progestin pills. Women with contraindications to estrogen use should use progestin-only pills.

26. The phenomenon of someone other than the mother-to-be experiencing pregnancy-like symptoms such as nausea and weight gain applies to the: A. mother of the pregnant woman. B. couple's teenage daughter. C. sister of the pregnant woman. D. expectant father.

ANS: D An expectant father's experiencing pregnancy-like symptoms is called the couvade syndrome.

22. The standard of care for obstetrics dictates that an internal version may be used to manipulate the: a. fetus from a breech to a cephalic presentation before labor begins. b. fetus from a transverse lie to a longitudinal lie before cesarean birth. c. second twin from an oblique lie to a transverse lie before labor begins. d. second twin from a transverse lie to a breech presentation during vaginal birth.

ANS: D Internal version is used only during vaginal birth to manipulate the second twin into a presentation that allows it to be born vaginally. For internal version to occur, the cervix needs to be completely dilated.

19. For the labor nurse, care of the expectant mother begins with any or all of these situations, with the exception of: a. the onset of progressive, regular contractions. b. the bloody, or pink, show. c. the spontaneous rupture of membranes. d. formulation of the woman's plan of care for labor.

ANS: D Labor care begins when progressive, regular contractions begin; the blood-tinged mucoid vaginal discharge appears; or fluid is discharged from the vagina. The woman and nurse can formulate their plan of care before labor or during treatment.

10. Which fetal heart rate (FHR) finding would concern the nurse during labor? a. Accelerations with fetal movement b. Early decelerations c. An average FHR of 126 beats/min d. Late decelerations

ANS: D Late decelerations are caused by uteroplacental insufficiency and are associated with fetal hypoxemia. They are considered ominous if persistent and uncorrected. Accelerations in the FHR are an indication of fetal well-being. Early decelerations in the FHR are associated with head compression as the fetus descends into the maternal pelvic outlet; they generally are not a concern during normal labor.

26. Which of the following statements is the most complete and accurate description of medical abortions? A. They are performed only for maternal health. B. They can be achieved through surgical procedures or with drugs. C. They are mostly performed in the second trimester. D. They can be either elective or therapeutic.

ANS: D Medical abortions are performed through the use of medications (rather than surgical procedures). They are mostly done in the first trimester, and they can be either elective (the woman's choice) or therapeutic (for reasons of maternal or fetal health).

30. Which collection of risk factors most likely would result in damaging lacerations (including episiotomies)? a. A dark-skinned woman who has had more than one pregnancy, who is going through prolonged second-stage labor, and who is attended by a midwife. b. A reddish-haired mother of two who is going through a breech birth. c. A dark-skinned, first-time mother who is going through a long labor. d. A first-time mother with reddish hair whose rapid labor was overseen by an obstetrician.

ANS: D Reddish-haired women have tissue that is less distensible than that of darker-skinned women and therefore may have less efficient healing. First time mothers are also more at risk, especially with breech births, long second-stage labors, or rapid labors in which there is insufficient time for the perineum to stretch. The rate of episiotomies is higher when obstetricians rather than midwives attend births.

5. A pregnant woman experiencing nausea and vomiting should: a. Drink a glass of water with a fat-free carbohydrate before getting out of bed in the morning. b. Eat small, frequent meals (every 2 to 3 hours). c. Increase her intake of high-fat foods to keep the stomach full and coated. d. Limit fluid intake throughout the day.

b. Eat small, frequent meals (every 2 to 3 hours).

The first stage of labor is often a time of introspection. In light of this, which of the following would guide your planning of nursing care? a) A woman should be left entirely alone during this period. b) A woman may spend time thinking about what is happening to her. c) No nursing care is needed to be done during this time. d) A woman will rarely speak or laugh during this period.

B (A woman may spend time thinking about what is happening to her. Rationale: Women need a support person with them during all stages of labor.)

Nurses can help their clients by keeping them informed about the distinctive stages of labor. What description of the phases of the first stage of labor is accurate? a) Latent: milk, regular contractions; no dilation; blood show; duration of 2 to 4 hours b) Active: moderate, regular contractions; 4 to 7cm dilation; duration of 3 to 6 hours c) Lull: no contractions; dilation stable; duration of 20 to 60 minutes d) Transition: very strong but irregular contractions; 8 to 10cm dilation; duration of 1 to 2 hours

B (Active: moderate, regular contractions; 4 to 7cm dilation; duration of 3 to 6 hours.)

There are four essential components of labor. The first is the passageway. It is composed of the bony pelvis and soft tissues. What is one component of the passageway? a) Perineum b) Cervix c) False pelvis d) Uterus

B (Cervix Rationale: The cervix and vagina are soft tissues that form the part of the passageway known as the birth canal.)

Which statement is the best rationale for assessing maternal vital signs between contractions? a. During a contraction, assessing fetal heart rates is the priority. b. Maternal circulating blood volume increases temporarily during contractions. c. Maternal blood flow to the heart is reduced during contractions. d. Vital signs taken during contractions are not accurate.

B (During uterine contractions, blood flow to the placenta temporarily stops, causing a relative increase in the mother's blood volume, which in turn temporarily increases blood pressure and slows pulse. It is important to monitor fetal response to contractions; however, this question is concerned with the maternal vital signs. Maternal blood flow is increased during a contraction. Vital signs are altered by contractions but are considered accurate for that period of time.)

The slight overlapping of cranial bones or shaping of the fetal head during labor is called: a. Lightening. b. Molding. c. Ferguson reflex. d. Valsalva maneuver.

B (Fetal head formation is called molding. Molding also permits adaptation to various diameters of the maternal pelvis. Lightening is the mother's sensation of decreased abdominal distention, which usually occurs the week before labor. The Ferguson reflex is the contraction urge of the uterus after stimulation of the cervix. The Valsalva maneuver describes conscious pushing during the second stage of labor.)

The primary difference between the labor of a nullipara and that of a multipara is the: a. Amount of cervical dilation. b. Total duration of labor. c. Level of pain experienced. d. Sequence of labor mechanisms.

B (In a first-time pregnancy, descent is usually slow but steady; in subsequent pregnancies, descent is more rapid, resulting in a shorter duration of labor. Cervical dilation is the same for all labors. Level of pain is individual to the woman, not to the number of labors she has experienced. The sequence of labor mechanisms is the same with all labors.)

All statements about normal labor are true except: a) A single fetus presents by vertex b) It is completed within 8 hours c) A regular progression of contractions, effacement, dilation, and descent occurs d) No complications are involved

B (It is completed within 8 hours)

One of the theories about the onset of labor is the prostaglandin theory. While not being conclusively proven that the action of prostaglandins initiate labor, it is known that prostaglandins do play a role in labor. What is an action of prostaglandins? a) Stimulates uterine muscle to relax b) Softens cervix c) Initiates relaxation of perineum d) Initiates cervical dilation

B (Softens cervix Rationale: The prostaglandin theory is another theory of labor initiation. Prostaglandins influence labor in several ways, which include softening the cervix and stimulating the uterus to contract. However, evidence supporting the theory that prostaglandins are the agents that trigger labor to begin is inconclusive.)

The nurse providing care for the laboring woman realizes that variable fetal heart rate (FHR) decelerations are caused by: a. Altered fetal cerebral blood flow. c. Uteroplacental insufficiency. b. Umbilical cord compression. d. Fetal hypoxemia.

B Variable decelerations can occur any time during the uterine contracting phase and are caused by compression of the umbilical cord. Altered fetal cerebral blood flow would result in early decelerations in the FHR. Uteroplacental insufficiency would result in late decelerations in the FHR. Fetal hypoxemia would result in tachycardia initially and then bradycardia if hypoxia continues.

2. Which meal would provide the most absorbable iron? a. Toasted cheese sandwich, celery sticks, tomato slices, and a grape drink b. Oatmeal, whole wheat toast, jelly, and low-fat milk c. Black bean soup, wheat crackers, orange sections, and prunes d. Red beans and rice, cornbread, mixed greens, and decaffeinated tea

c. Black bean soup, wheat crackers, orange sections, and prunes

A tiered system of categorizing FHR has been recommended by regulatory agencies. Nurses, midwives, and physicians who care for women in labor must have a working knowledge of fetal monitoring standards and understand the significance of each category. These categories include (Select all that apply): a. Reassuring. b. Category I. c. Category II. d. Nonreassuring. e. Category III.

B, C, E The three tiered system of FHR tracings include Category I, II, and III. Category I is a normal tracing requiring no action. Category II FHR tracings are indeterminate. This category includes tracings that do not meet Category I or III criteria. Category III tracings are abnormal and require immediate intervention.

A pregnant patient with severe preeclampsia who is being transported to a tertiary care center needs to be administered magnesium sulfate injection for seizure activity. What actions does the nurse take when administering the drug? Select all that apply. A. A 10-g dose is administered in the buttock. B. A local anesthetic is added to the solution. C. The Z-track technique is used to inject the drug. D. The injection site is massaged after the injection. E. The subcutaneous route is used to inject the drug.

B, C, D (The nurse adds a local anesthetic to the solution to reduce pain that is caused by the injection. The Z-track technique is used to inject the drug so that the drug is injected in the intramuscular (IM) tissue safely. The nurse gently massages the site after administering the injection to reduce pain. The nurse administers two separate injections of 5 g in each buttock. Magnesium sulfate injections are administered in the IM layer and not the subcutaneous layer.)

What are the possible causes of miscarriage during early pregnancy? Select all that apply. A. Premature dilation of cervix B. Chromosomal abnormalities C. Endocrine imbalance D. Hypothyroidism E. Antiphospholipid antibodies

B, C, D, E (Chromosomal abnormalities account for 50% of all early pregnancy losses. Endocrine imbalance is caused by luteal phase defects, hypothyroidism, and diabetes mellitus in pregnant patients and results in miscarriage. Antiphospholipid antibodies also increase the chances of miscarriage in pregnant patients. Premature dilation of the cervix may cause a second-trimester loss and is usually seen in patients between 12 and 20 weeks' gestation.)

A pregnant patient with chronic hypertension is at risk for placental abruption. Which symptoms of abruption does the nurse instruct the patient to be alert for? Select all that apply. A. Weight loss B. Abdominal pain C. Vaginal bleeding D. Shortness of breath E. Uterine tenderness

B, C, E (The nurse instructs the pregnant patient to be alert for abdominal pain, vaginal bleeding, and uterine tenderness as these indicates placental abruption. Weight loss indicates fluid and electrolyte loss and not placental abruption. Shortness of breath indicates inadequate oxygen, which is usually seen in a patient who is having cardiac arrest.)

A pregnant woman in her first trimester with a history of epilepsy is transported to the hospital via ambulance after suffering a seizure in a restaurant. The nurse expects which health care provider orders to be included in the plan of care? (Select all that apply.) A. valproate (Depakote). B. Serum lab levels of medications. C. Abdominal ultrasounds. D. Prenatal vitamins with vitamin D. E. carbamazepine (Tegretol).

B. Serum lab levels of medications. C. Abdominal ultrasounds. D. Prenatal vitamins with vitamin D. Carbamazepine (Tegretol) and valproate (Depakote) should be avoided if possible during pregnancy, especially during the first trimester, because their use is associated with NTDs in the fetus. Checking lab levels of medications, performing abdominal ultrasounds to assess fetal growth, and taking prenatal vitamins with vitamin D are all expected interventions for a pregnant woman diagnosed with epilepsy.

From 4% to 8% of pregnant women have asthma, making it one of the most common preexisting conditions of pregnancy. Severity of symptoms usually peaks: A. in the first trimester. B. between 24 to 36 weeks of gestation. C. during the last 4 weeks of pregnancy. D. immediately postpartum

B. between 24 to 36 weeks of gestation. Women often have few symptoms of asthma during the first trimester. The severity of symptoms peaks between 24 and 36 weeks of gestation. Asthma appears to be associated with intrauterine growth restriction and preterm birth. During the last 4 weeks of pregnancy symptoms often subside. The period between 24 and 36 weeks of pregnancy is associated with the greatest severity of symptoms. Issues have often resolved by the time the woman delivers.

A pregnant woman with cardiac disease is informed about signs of cardiac decompensation. She should be told that the earliest sign of decompensation is most often: A. orthopnea. B. decreasing energy levels. C. moist frequent cough and frothy sputum. D. crackles (rales) at the bases of the lungs on auscultation.

B. decreasing energy levels. Orthopnea is a finding that appears later when a failing heart reduces renal perfusion and fluid accumulates in the pulmonary interstitial space, leading to pulmonary edema. Decreasing energy level (fatigue) is an early finding of heart failure. Care must be taken to recognize it as a warning rather than a typical change of the third trimester. Cardiac decompensation is most likely to occur early in the third trimester, during childbirth, and during the first 48 hours following birth. A moist, frequent cough appears later when a failing heart reduces renal perfusion and fluid accumulates in the pulmonary interstitial space, leading to pulmonary edema. Crackles and rales appear later when a failing heart reduces renal perfusion and fluid accumulates in the pulmonary interstitial space, leading to pulmonary edema.

The nurse is preparing to discharge a 30-year-old woman who has experienced a miscarriage at 10 weeks of gestation. Which statement by the woman would indicate a correct understanding of the discharge instructions? A. "I will not experience mood swings since I was only at 10 weeks of gestation." B. "I will avoid sexual intercourse for 6 weeks and pregnancy for 6 months." C. "I should eat foods that are high in iron and protein to help my body heal." D. "I should expect the bleeding to be heavy and bright red for at least 1 week."

C ("I should eat foods that are high in iron and protein to help my body heal." After a miscarriage a woman may experience mood swings and depression from the reduction of hormones and the grieving process. Sexual intercourse should be avoided for 2 weeks or until the bleeding has stopped and should avoid pregnancy for 2 months. A woman who has experienced a miscarriage should be advised to eat foods that are high in iron and protein to help replenish her body after the loss. The woman should not experience bright red, heavy, profuse bleeding; this should be reported to the health care provider.)

A woman is documented on the labor and delivery board to be 7cm dilated. Her family wants to know how long she will be in labor. The nurse should provide which information to the family? a) "She is doing well, in the second stage and it could be anytime now." b) "She is in the transition phase of labor and it will be with in 2 to 3 hours, might be sooner." c) "She is in active labor, she is progressing at this point and we will keep you posted." d) "She is still in early latent labor and has much too long to go to tell when she will deliver."

C ("She is in active labor, she is progressing at this point and we will keep you posted." Rationale: At 7cm dilated, she is considered in Active Phase of labor. There is no science that can predict the length of labor. She is progressing in labor and it is not best to give the family a specific time frame.)

A fetus is assessed at 2 cm above the ischial spines. The nurse would document fetal station as: a) +2 b) 0 c) -2 d) +4

C (-2 Rationale: When the presenting part is above the ischial spines, it is noted as a negative station. Since the measurement is 2 cm, the station would be -2. A 0 station indicates that the fetal presenting part is at the level of the ischial spines. Positive stations indicate that the presenting part is below the level of the ischial spines.)

The nurse is preparing to discharge a 30-year-old woman who has experienced a miscarriage at 10 weeks of gestation. Which statement by the woman indicates a correct understanding of the discharge instructions? A. "I will not experience mood swings since I was only at 10 weeks of gestation." B. "I will avoid sexual intercourse for 6 weeks and pregnancy for 6 months." C. "I should eat foods that are high in iron and protein to help my body heal." D. "I should expect the bleeding to be heavy and bright red for at least 1 week."

C (A woman who has experienced a miscarriage should be advised to eat foods that are high in iron and protein to help replenish her body after the loss. After a miscarriage, a woman may experience mood swings and depression from the reduction of hormones and the grieving process. Sexual intercourse should be avoided for 2 weeks or until the bleeding has stopped and should avoid pregnancy for 2 months. The woman should not experience bright red, heavy, profuse bleeding; this should be reported to the health care provider.)

Which of the following would alert the nurse that the client is in the transition phase of labor? a) Reduction of rectal pressure b) Decrease in the bloody show c) Beginning urge to bear down d) Enthusiasm in the client

C (Beginning urge to bear down Rationale: Starting of the urge to bear down is a feature associated with the transition phase of labor. The transition phase is the last phase of the first stage of labor. In this phase the process of cervical dilatation is completed. During this phase the client experiences an increase in rectal pressure, an increase in the bloody show and an urge to bear down. The contractions are stronger and hence the client feels irritable, restless and nauseous. The client feels enthusiastic during the latent phase and not the transition phase.)

Braxton Hicks contractions are termed "practice contractions" and occur throughout pregnancy. When the woman's body is getting ready to go into labor, it begins to show anticipatory signs of impending labor. Among these signs are Braxton Hicks contractions that are more frequent and stronger in intensity. What differentiates Braxton Hicks contractions from true labor? a) Braxton Hicks contractions get closer together with activity b) Braxton Hicks contractions cause "ripening" of the cervix. c) Braxton Hicks contractions usually decrease in intensity with walking d) Braxton Hicks contractions do not last long enough to be true labor

C (Braxton Hicks contractions usually decrease in intensity with walking Rationale: Braxton Hicks contractions occur more frequently and are more noticeable as pregnancy approaches term. These irregular, practice contractions usually decrease in intensity with walking and position changes.)

There are four essential components of labor. The first is the passageway. It is composed of the bony pelvis and soft tissues. What is one component of the passageway? a) Uterus b) False pelvis c) Cervix d) Perineum

C (Cervix Rationale: The cervix and vagina are soft tissues that form the part of the passageway known as the birth canal.)

Which nursing assessment indicates that a woman who is in second-stage labor is almost ready to give birth? a. The fetal head is felt at 0 station during vaginal examination. b. Bloody mucus discharge increases. c. The vulva bulges and encircles the fetal head. d. The membranes rupture during a contraction.

C (During the active pushing (descent) phase, the woman has strong urges to bear down as the presenting part of the fetus descends and presses on the stretch receptors of the pelvic floor. The vulva stretches and begins to bulge encircling the fetal head. Birth of the head occurs when the station is +4. A 0 station indicates engagement. Bloody show occurs throughout the labor process and is not an indication of an imminent birth. Rupture of membranes can occur at any time during the labor process and does not indicate an imminent birth.)

While providing care to a patient in active labor, the nurse should instruct the woman that: a. The supine position commonly used in the United States increases blood flow. b. The "all fours" position, on her hands and knees, is hard on her back. c. Frequent changes in position will help relieve her fatigue and increase her comfort. d. In a sitting or squatting position, her abdominal muscles will have to work harder.

C (Frequent position changes relieve fatigue, increase comfort, and improve circulation. Blood flow can be compromised in the supine position; any upright position benefits cardiac output. The "all fours" position is used to relieve backache in certain situations. In a sitting or squatting position, the abdominal muscles work in greater harmony with uterine contractions.)

At 37 weeks of gestation, the patient is in a severe automobile crash where her abdomen was hit by the steering wheel and her seat belt. What actions would the emergency room nurse expect to perform upon the patient's arrival at the hospital? A. Stay with the patient, assure a patent airway is present, and keep the patient as calm as possible. B. Move the patient's skirt to determine if any vaginal bleeding is present, find out who to call, and monitor the level of consciousness. C. Assess the patient's vital signs, determine location and severity of pain, and establish continual fetal heart rate monitoring. D. Obtain arterial blood gases, obtain a hemoglobin and hematocrit, and oxygen saturation rate.

C (Full assessment of the patient and her fetus are essential and include vital signs, continual fetal heart rate monitoring, determining the location and severity of pain, whether any vaginal bleeding is dark red or bright red, and the status of the abdomen, which would be expected to be rigid or "board like." Staying with the patient, assuring a patent airway is present, and keeping the patient as calm as possible would be appropriate at the crash site before the arrival of emergency medical services (EMS). The current status of the patient and fetus are thepriority. The health care provider would prescribe the arterial blood gases and other laboratory work after the patient is assessed and stabilized.)

If a fetus is in an ROA position during labor, you would interpret this to mean the fetus is a) Presenting with the face as the presenting part b) In a common breech delivery position c) In a longitudinal lie facing the left posterior d) Facing the right anterior abdominal quadrant

C (In a longitudinal lie facing the left posterior Rationale: ROA (right occiput anterior) means the occiput of the fetal head points toward the mother's right anterior pelvis; the head is the presenting part.)

What does the nurse advise a pregnant patient who is prescribed phenazopyridine (Pyridium) for cystitis? A. "Avoid sweet foods in diet." B. "Limit exposure to sunlight." C. "Do not wear contact lenses." D. "Restrict oral fluids to 125 mL per hour."

C (Phenazopyridine (Pyridium) colors the tears orange. Therefore the nurse instructs the patient to avoid wearing contact lenses. Sweet foods are avoided in patients with diabetes mellitus, because they can cause fluctuating glucose levels, which may harm the fetus. Exposure to sunlight is avoided when the patient is receiving methotrexate therapy, because it causes photosensitivity. Oral fluids are restricted in patients who are at risk for pulmonary edema.)

What instruction does the nurse provide to a pregnant patient with mild preeclampsia? A. "You need to be hospitalized for fetal evaluation." B. "Nonstress testing can be done once every month." C. "Fetal movement counts need to be evaluated daily." D. "Take complete bed rest during the entire pregnancy."

C (Preeclampsia can affect the fetus and may cause fetal growth restrictions, decreased amniotic fluid volume, abnormal fetal oxygenation, low birth weight, and preterm birth. Therefore the fetal movements need to be evaluated daily. Patients with mild preeclampsia can be managed at home effectively and need not be hospitalized. Nonstress testing is performed once or twice per week to determine fetal well-being. Patients need to restrict activity, but complete bed rest is not advised because it may cause cardiovascular deconditioning, muscle atrophy, and psychological stress.)

Signs of a threatened abortion (miscarriage) are noted in a woman at 8 weeks of gestation. What is an appropriate management approach for this type of abortion? A. Prepare the woman for a dilation and curettage (D&C). B. Place the woman on bed rest for at least 1 week and reevaluate. C. Prepare the woman for an ultrasound and blood work. D. Comfort the woman by telling her that if she loses this baby, she may attempt to get pregnant again in 1 month.

C (Prepare the woman for an ultrasound and blood work. D&C is not considered until signs of the progress to an inevitable abortion are noted or the contents are expelled and incomplete. Bed rest is recommended for 48 hours initially. Repetitive transvaginal ultrasounds and measurement of human chorionic gonadotropin (hCG) and progesterone levels may be performed to determine if the fetus is alive and within the uterus. If the pregnancy is lost, the woman should be guided through the grieving process. Telling the client that she can get pregnant again soon is not a therapeutic response because it discounts the importance of this pregnancy.)

The nurse caring for the woman in labor should understand that increased variability of the fetal heart rate may be caused by: a. Narcotics. c. Methamphetamines. b. Barbiturates. d. Tranquilizers.

C Narcotics, barbiturates, and tranquilizers may be causes of decreased variability; methamphetamines may cause increased variability.

Signs of a threatened abortion (miscarriage) are noted in a woman at 8 weeks of gestation. What is an appropriate management approach for this type of abortion? A. Prepare the woman for a dilation and curettage (D&C). B. Place the woman on bed rest for at least 1 week and reevaluate. C. Prepare the woman for an ultrasound and bloodwork. D. Comfort the woman by telling her that if she loses this baby, she may attempt to get pregnant again in 1 month.

C (Repetitive transvaginal ultrasounds and measurement of human chorionic gonadotropin (hCG) and progesterone levels may be performed to determine if the fetus is alive and within the uterus. If the pregnancy is lost , the woman should be guided through the grieving process. D&C is not considered until signs of the progress to an inevitable abortion are noted or the contents are expelled and incomplete. Bed rest is recommended for 48 hours initially. Telling the woman that she can get pregnant again soon is not a therapeutic response because it discounts the importance of this pregnancy.)

Which position would the nurse suggest for second-stage labor if the pelvic outlet needs to be increased? a) Semirecumbent b) Sitting c) Squatting d) Side-lying

C (Squatting)

With regard to the turns and other adjustments of the fetus during the birth process, known as the mechanism of labor, nurses should be aware that: a. The seven critical movements must progress in a more or less orderly sequence. b. Asynclitism sometimes is achieved by means of the Leopold maneuver. c. The effects of the forces determining descent are modified by the shape of the woman's pelvis and the size of the fetal head. d. At birth the baby is said to achieve "restitution" (i.e., a return to the C-shape of the womb).

C (The size of the maternal pelvis and the ability of the fetal head to mold also affect the process. The seven identifiable movements of the mechanism of labor occur in combinations simultaneously, not in precise sequences. Asynclitism is the deflection of the baby's head; the Leopold maneuver is a means of judging descent by palpating the mother's abdomen. Restitution is the rotation of the baby's head after the infant is born.)

The nurse caring for a woman in labor understands that prolonged decelerations: a. Are a continuing pattern of benign decelerations that do not require intervention. b. Constitute a baseline change when they last longer than 5 minutes. c. Usually are isolated events that end spontaneously. d. Require the usual fetal monitoring by the nurse.

C Prolonged decelerations usually are isolated events that end spontaneously. However, in certain combinations with late and/or variable decelerations, they are a danger sign that requires the nurse to notify the physician or midwife immediately. A deceleration that lasts longer than 10 minutes constitutes a baseline change.

When educating a group of nursing students about the theories of onset of labor, the nurse identifies which of the following factors as the possible causes for onset of labor? Select all that apply. a) Increase in the production of progesterone b) Fall in the estrogen at 34-35 weeks of pregnancy c) Prostaglandin production in the myometrium d) Increase in the fetal cortisol levels e) Release of oxytocin by the pituitary

C, D, E (Rationale: The possible causes for the onset of labor include increase in the fetal cortisol levels, release of oxytocin by the posterior pituitary and the production of prostaglandins. Progesterone withdrawal, and not an increase, initiates labor. There is a rise in the estrogen levels at 34-35 weeks of pregnancy. Estrogen stimulates prostaglandin production and also promotes the release of oxytocin.)

The baseline fetal heart rate (FHR) is the average rate during a 10-minute segment. Changes in FHR are categorized as periodic or episodic. These patterns include both accelerations and decelerations. The labor nurse is evaluating the patient's most recent 10-minute segment on the monitor strip and notes a late deceleration. This is likely to be caused by which physiologic alteration (Select all that apply)? a. Spontaneous fetal movement b. Compression of the fetal head c. Placental abruption d. Cord around the baby's neck e. Maternal supine hypotension

C, E Late decelerations are almost always caused by uteroplacental insufficiency. Insufficiency is caused by uterine tachysystole, maternal hypotension, epidural or spinal anesthesia, IUGR, intraamniotic infection, or placental abruption. Spontaneous fetal movement, vaginal examination, fetal scalp stimulation, fetal reaction to external sounds, uterine contractions, fundal pressure and abdominal palpation are all likely to cause accelerations of the FHR. Early decelerations are most often the result of fetal head compression and may be caused by uterine contractions, fundal pressure, vaginal examination, and placement of an internal electrode. A variable deceleration is likely caused by umbilical cord compression. This may happen when the umbilical cord is around the baby's neck, arm, leg, or other body part or when there is a short cord, a knot in the cord, or a prolapsed cord.

Maternal and neonatal risks associated with gestational diabetes mellitus are: A. maternal premature rupture of membranes and neonatal sepsis. B. maternal hyperemesis and neonatal low birth weight. C. Maternal preeclampsia and fetal macrosomia. D. maternal placenta previa and fetal prematurity.

C. Maternal preeclampsia and fetal macrosomnia Premature rupture of membranes and neonatal sepsis are not risks associated with gestational diabetes. Hyperemesis is not seen with gestational diabetes, nor is there an association with low birth weight of the infant. Women with gestational diabetes have twice the risk of developing hypertensive disorders such as preeclampsia, and the baby usually has macrosomia. Placental previa and subsequent prematurity of the neonate are not risks associated with gestational diabetes.

During pregnancy, alcohol withdrawal may be treated using: A. disulfiram (Antabuse). B. corticosteroids. C. benzodiazepines. E. aminophylline.

C. benzodiazepines. Disulfiram is contraindicated in pregnancy because it is teratogenic. Corticosteroids are not used to treat alcohol withdrawal. Symptoms that occur during alcohol withdrawal can be managed with short-acting barbiturates or benzodiazepines. Aminophylline is not used to treat alcohol withdrawal.

The factors that affect the process of labor and birth, known commonly as the five Ps, include all except: a. Passenger. b. Powers. c. Passageway. d. Pressure.

D (The five Ps are passenger (fetus and placenta), passageway (birth canal), powers (contractions), position of the mother, and psychologic response.)

The patient is having a routine prenatal visit and asks the nurse what the childbirth education teacher meant when she used the term zero station. What is the best response by the nurse? a) "This is just a way of determining your progress in labor." b) This indicates that you start labor within the next 24 hours." c) "This means +1 and the baby is entering the true pelvis." d) "The presenting part is at the true pelvis and is engaged."

D ("The presenting part is at the true pelvis and is engaged." Rationale: Zero station is when the fetus is engaged in the pelvis, or has dropped. This is an encouraging sign for the patient. This sign is indicative that labor may be beginning, but there is no set time frame regarding when it will start. Labor has not started yet, and the fetus has not begun to move out of the uterus.)

A pregnant woman, multipara, has been in labor for several hours. She cries out that her contractions are getting harder and that she can't do this. The patient is really irritable, nauseated, annoyed, and fearful of being left alone. Considering the client's behavior, the nurse would expect the cervix to be dilated how many centimeters? a) 5-6 b) 0-2 c) 3-5 d) 8-10

D (8-10 Rationale: The reaction of the patient is indicative of entering or being in the transition phase of labor, stage 1. The dilation would be 8-10 cm. Before that, when dilation is 0-7 cm, the patient has an easier time using positive coping skills.)

Nurses can advise their clients that all are signs that precede labor except: a) A return of urinary frequency as a result of increased bladder pressure b) Persistent low backache from relaxed pelvic joints c) Stronger and more frequent uterine (Braxton Hicks) contractions d) A decline in energy, as the body stores up for labor

D (A decline in energy, as the body stores up for labor)

Which basic type of pelvis includes the correct description and percentage of occurrence in women? a. Gynecoid: classic female; heart shaped; 75% b. Android: resembling the male; wider oval; 15% c. Anthropoid: resembling the ape; narrower; 10% d. Platypelloid: flattened, wide, shallow; 3%

D (A platypelloid pelvis is flattened, wide, and shallow; about 3% of women have this shape. The gynecoid shape is the classical female shape, slightly ovoid and rounded; about 50% of women have this shape. An android, or malelike, pelvis is heart shaped; about 23% of women have this shape. An anthropoid, or apelike, pelvis is oval and wider; about 24% of women have this shape.)

Assessment reveals that the fetus of a client in labor is in the vertex presentation. The nurse determines that the presenting part is which of the following? a) Brow b) Buttocks c) Shoulders d) Occiput

D (Occiput Rationale: With a vertex presentation, a type of cephalic presentation, the fetal presenting part is the occiput. The shoulders are the presenting part when the fetus is in a shoulder presentation. The brow or sinciput is the presenting part when a fetus is in a brow presentation. The buttocks are the presenting part when a fetus is in a breech presentation.)

A client in labor has been admitted to the labor and birth suite. The nurse assessing her notes that the fetus is in a cephalic presentation. Which of the following should the nurse identify by the term presentation? a) Relation of the different fetal body parts to one another b) Relationship of the presenting part to the maternal pelvis c) Relation of the fetal presenting part to the maternal ischial spine d) Part of the fetal body entering the maternal pelvis first

D (Part of the fetal body entering the maternal pelvis first Rationale: The term presentation is the part of the fetal body that is entering the maternal pelvis first. Relationship of the presenting part to the sides of the maternal pelvis is called the position. Attitude is the term that describes the relation of the different fetal body parts to one another. Relation of the fetal presenting part to maternal ischial spine is termed the station.)

Which is a priority nursing action when a pregnant patient with severe gestational hypertension is admitted to the health care facility? A. Prepare the patient for cesarean delivery. B. Administer intravenous (I.V.) and oral fluids. C. Provide diversionary activities during bed rest. D. Administer the prescribed magnesium sulfate.

D (The nurse administers the prescribed magnesium sulfate to the patient to prevent eclamptic seizures. I.V. oral fluids are indicated when there is severe dehydration in the patient. It is important to provide diversionary activities during bed rest, but it is secondary in this case. A patient who has experienced a multisystem trauma is prepared for cesarean delivery if there is no evidence of a maternal pulse, which increases the chance of maternal survival.)

During the second stage of labor, a woman is generally a) very aware of activities immediately around her. b) anxious to have people around her. c) no longer in need of a support person. d) turning inward to concentrate on body sensations.

D (Turning inward to concentrate on body sensations. Rationale: Second-stage contractions are so unusual that most women are unable to think of things other than what is happening inside their body.)

When assessing the relative advantages and disadvantages of internal and external electronic fetal monitoring, nurses comprehend that both: a. Can be used when membranes are intact. b. Measure the frequency, duration, and intensity of uterine contractions. c. May need to rely on the woman to indicate when uterine activity (UA) is occurring. d. Can be used during the antepartum and intrapartum periods.

D External monitoring can be used in both periods; internal monitoring can be used only in the intrapartum period. For internal monitoring the membranes must have ruptured, and the cervix must be sufficiently dilated. Internal monitoring measures the intensity of contractions; external monitoring cannot do this. With external monitoring, the woman may need to alert the nurse that UA is occurring; internal monitoring does not require this.

While evaluating an external monitor tracing of a woman in active labor whose labor is being induced, the nurse notes that the fetal heart rate (FHR) begins to decelerate at the onset of several contractions and returns to baseline before each contraction ends. The nurse should: a. Change the woman's position. b. Discontinue the oxytocin infusion. c. Insert an internal monitor. d. Document the finding in the client's record.

D The FHR indicates early decelerations, which are not an ominous sign and do not require any intervention. The nurse should simply document these findings.

A nurse may be called on to stimulate the fetal scalp: a. As part of fetal scalp blood sampling. b. In response to tocolysis. c. In preparation for fetal oxygen saturation monitoring. d. To elicit an acceleration in the fetal heart rate (FHR).

D The scalp can be stimulated using digital pressure during a vaginal examination. Fetal scalp blood sampling involves swabbing the scalp with disinfectant before a sample is collected. The nurse would stimulate the fetal scalp to elicit an acceleration of the FHR. Tocolysis is relaxation of the uterus. Fetal oxygen saturation monitoring involves the insertion of a sensor.

During a physical assessment of an at-risk client, the nurse notes generalized edema, crackles at the base of the lungs, and some pulse irregularity. These are most likely signs of: A. euglycemia. B. rheumatic fever. C. pneumonia. D. cardiac decompensation.

D. cardiac decompensation. Euglycemia is a condition of normal glucose levels. These symptoms indicate cardiac decompensation. Rheumatic fever can cause heart problems, but it does not present with these symptoms, which indicate cardiac decompensation. Pneumonia is an inflammation of the lungs and would not likely generate these symptoms, which indicate cardiac decompensation. Symptoms of cardiac decompensation may appear abruptly or gradually.

In planning for the care of a 30-year-old woman with pregestational diabetes, the nurse recognizes that the most important factor affecting pregnancy outcome is the: A. mother's age. B. number of years since diabetes was diagnosed. C. amount of insulin required prenatally. D. degree of glycemic control during pregnancy.

D. degree of glycemic control during pregnancy. Although advanced maternal age may pose some health risks, for the woman with pregestational diabetes the most important factor remains the degree of glycemic control during pregnancy. The number of years since diagnosis is not as relevant to outcomes as the degree of glycemic control. The key to reducing risk in the pregestational diabetic woman is not the amount of insulin required but rather the level of glycemic control. Women with excellent glucose control and no blood vessel disease should have good pregnancy outcomes.

3. Which nutrient's recommended dietary allowance (RDA) is higher during lactation than during pregnancy? a. Energy (kcal) b. Iron c. Vitamin A d. Folic acid

a. Energy (kcal)

22. With regard to nutritional needs during lactation, a maternity nurse should be aware that: a. The mother's intake of vitamin C, zinc, and protein now can be lower than during pregnancy. b. Caffeine consumed by the mother accumulates in the infant, who may be unusually active and wakeful. c. Critical iron and folic acid levels must be maintained. d. Lactating women can go back to their prepregnant calorie intake.

b. Caffeine consumed by the mother accumulates in the infant, who may be unusually active and wakeful.

38. In teaching the pregnant adolescent about nutrition, the nurse should: a. Emphasize the need to eliminate common teen snack foods because they are too high in fat and sodium. b. Determine the weight gain needed to meet adolescent growth and add 35 lb. c. Suggest that she not eat at fast-food restaurants to avoid foods of poor nutritional value. d. Realize that most adolescents are unwilling to make dietary changes during pregnancy.

b. Determine the weight gain needed to meet adolescent growth and add 35 lb.

14. After you complete your nutritional counseling for a pregnant woman, you ask her to repeat your instructions so you can assess her understanding of the instructions given. Which statement indicates that she understands the role of protein in her pregnancy? a. "Protein will help my baby grow." b. "Eating protein will prevent me from becoming anemic." c. "Eating protein will make my baby have strong teeth after he is born." d. "Eating protein will prevent me from being diabetic."

a. "Protein will help my baby grow."

35. A pregnant patient would like to know a good food source of calcium other than dairy products. Your best answer is: a. Legumes b. Yellow vegetables c. Lean meat d. Whole grains

a. Legumes

27. Three servings of milk, yogurt, or cheese plus two servings of meat, poultry, or fish adequately supply the recommended amount of protein for a pregnant woman. Many patients are concerned about the increased levels of mercury in fish and may be afraid to include this source of nutrients in their diet. Sound advice by the nurse to assist the client in determining which fish is safe to consume would include: a. Canned white tuna is a preferred choice. b. Avoid shark, swordfish, and mackerel. c. Fish caught in local waterways are the safest. d. Salmon and shrimp contain high levels of mercury.

b. Avoid shark, swordfish, and mackerel.

7. Which statement made by a lactating woman would lead the nurse to believe that the woman might have lactose intolerance? a. "I always have heartburn after I drink milk." b. "If I drink more than a cup of milk, I usually have abdominal cramps and bloating." c. "Drinking milk usually makes me break out in hives." d. "Sometimes I notice that I have bad breath after I drink a cup of milk."

b. "If I drink more than a cup of milk, I usually have abdominal cramps and bloating."

13. Women with an inadequate weight gain during pregnancy are at higher risk of giving birth to an infant with: a. Spina bifida. b. Intrauterine growth restriction. c. Diabetes mellitus. d. Down syndrome.

b. Intrauterine growth restriction

9. A 27-year-old pregnant woman had a preconceptual body mass index (BMI) of 18.0. The nurse knows that this woman's total recommended weight gain during pregnancy should be at least: a. 20 kg (44 lb). b. 16 kg (35 lb). c. 12.5 kg (27.5 lb). d. 10 kg (22 lb).

c. 12.5 kg (27.5 lb).

8. A pregnant woman's diet history indicates that she likes the following list of foods. The nurse would encourage this woman to consume more of which food to increase her calcium intake? a. Fresh apricots b. Canned clams c. Spaghetti with meat sauce d. Canned sardines

d. Canned sardines

24. When counseling a client about getting enough iron in her diet, the maternity nurse should tell her that: a. Milk, coffee, and tea aid iron absorption if consumed at the same time as iron. b. Iron absorption is inhibited by a diet rich in vitamin C. c. Iron supplements are permissible for children in small doses. d. Constipation is common with iron supplements.

d. Constipation is common with iron supplements.

10. A woman in week 34 of pregnancy reports that she is very uncomfortable because of heartburn. The nurse would suggest that the woman: a. Substitute other calcium sources for milk in her diet. b. Lie down after each meal. c. Reduce the amount of fiber she consumes. d. Eat five small meals daily.

d. Eat five small meals daily.

11. A woman has come to the clinic for preconception counseling because she wants to start trying to get pregnant in 3 months. She can expect the following advice: a. "Discontinue all contraception now." b. "Lose weight so that you can gain more during pregnancy." c. "You may take any medications you have been taking regularly." d. "Make sure that you include adequate folic acid in your diet."

d. "Make sure that you include adequate folic acid in your diet."

28. Nutrition is one of the most significant factors influencing the outcome of a pregnancy. It is an alterable and important preventive measure for various potential problems, such as low birth weight and prematurity. While completing the physical assessment of the pregnant client, the nurse can evaluate the client's nutritional status by observing a number of physical signs. Which sign would indicate that the client has unmet nutritional needs? a. Normal heart rate, rhythm, and blood pressure b. Bright, clear, shiny eyes c. Alert, responsive, and good endurance d. Edema, tender calves, and tingling

d. Edema, tender calves, and tingling

Which test does the nurse recommend for the patient to help assess fetal genetic abnormalities? 1 Amniotic fluid volume (AFV) 2 Fetal body movements 3 Nuchal translucency (NT) 4 Fetal heart activity

3 The NT ultrasound screening technique is used to measure fluid in the nape of the fetal neck between 10 and 14 weeks' gestation. Fluid volume greater than 3 mm is considered abnormal. NT is used mostly to identify possible fetal genetic abnormalities. AFV, fetal body movements, and fetal heart activity are measured to assess fetal well-being.

The initial descent of the fetus into the pelvis to zero station is which one of the cardinal movements of labor? a) Engagement b) Felxion c) Extension d) Expulsion

A (Engagement Rationale: The movement of the fetus into the pelvis from the upper uterus is engagement. This is the first cardinal movement of the fetus in preparation for the spontaneous vaginal delivery. Flexion occurs as the fetus encounters resistance from the soft tissues and muscles of the pelvic floor. Extension is the state in which the fetal head is well flexed with the chin on the chest as the fetus travels through the birth canal. Expulsion occurs after delivery of the anterior and posterior shoulders.)

Which presentation is described accurately in terms of both presenting part and frequency of occurrence? a. Cephalic: occiput; at least 95% b. Shoulder: scapula; 10% to 15% c. Breech: sacrum; 10% to 15% d. Cephalic: cranial; 80% to 85%

A (In cephalic presentations (head first), the presenting part is the occiput; this occurs in 96% of births. In a breech birth, the sacrum emerges first; this occurs in about 3% of births. In shoulder presentations, the scapula emerges first; this occurs in only 1% of births.)

Which instructions does the nurse give to a patient who is prescribed methotrexate therapy for dissolving the tubal pregnancy? A. "Discontinue folic acid supplements." B. "Get adequate exposure to sunlight." C. "Take stronger analgesics for severe pain." D. "Vaginal intercourse is safe during the therapy."

A (The nurse advises the patient to discontinue folic acid supplements as they interact with methotrexate and may exacerbate ectopic rupture in the patient. Exposure to sunlight is avoided as the therapy makes the patient photosensitive. Analgesics stronger than acetaminophen are avoided, because they may mask symptoms of tubal rupture. Vaginal intercourse is avoided until the pregnancy is dissolved completely.)

A normal uterine activity pattern in labor is characterized by: a. Contractions every 2 to 5 minutes. b. Contractions lasting about 2 minutes. c. Contractions about 1 minute apart. d. A contraction intensity of about 1000 mm Hg with relaxation at 50 mm Hg.

A Contractions normally occur every 2 to 5 minutes and last less than 90 seconds (intensity 800 mm Hg) with about 30 seconds in between (20 mm Hg or less).

The nurse providing care for the laboring woman comprehends that accelerations with fetal movement: a. Are reassuring. b. Are caused by umbilical cord compression. c. Warrant close observation. d. Are caused by uteroplacental insufficiency.

A Episodic accelerations in the fetal heart rate (FHR) occur during fetal movement and are indications of fetal well-being. Umbilical cord compression results in variable decelerations in the FHR. Accelerations in the FHR are an indication of fetal well-being and do not warrant close observation. Uteroplacental insufficiency would result in late decelerations in the FHR.

The nurse knows that proper placement of the tocotransducer for electronic fetal monitoring is located: a. Over the uterine fundus. c. Inside the uterus. b. On the fetal scalp. d. Over the mother's lower abdomen.

A The tocotransducer monitors uterine activity and should be placed over the fundus, where the most intensive uterine contractions occur. The tocotransducer is for external use.

13. Appendicitis may be difficult to diagnose in pregnancy because the appendix is: A. displaced upward and laterally, high and to the right. B. displaced upward and laterally, high and to the left. C. deep at McBurney point. D. displaced downward and laterally, low and to the right.

ANS: A The appendix is displaced high and to the right, beyond McBurney point.

12. Which behavior indicates that a woman is "seeking safe passage" for herself and her infant? A. She keeps all prenatal appointments. B. She "eats for two." C. She drives her car slowly. D. She wears only low-heeled shoes.

ANS: A The goal of prenatal care is to foster a safe birth for the infant and mother. Although eating properly, driving carefully, and using proper body mechanics all are healthy measures that a mother can take, obtaining prenatal care is the optimal method for providing safety for both herself and her baby.

1. Congenital disorders refer to conditions that are present at birth. These disorders may be inherited and caused by environmental factors or maternal malnutrition. Toxic exposures have the greatest effect on development between 15 and 60 days of gestation. For the nurse to be able to conduct a complete assessment of the newly pregnant patient, she should understand the significance of exposure to known human teratogens. These include: (Select all that apply.) A. infections. B. radiation. C. maternal conditions. D. drugs. E. chemicals.

ANS: A, B, C, D, E Exposure to radiation and numerous infections may result in profound congenital deformities. These include but are not limited to varicella, rubella, syphilis, parvovirus, cytomegalovirus, and toxoplasmosis. Certain maternal conditions such as diabetes and phenylketonuria may also affect organs and other parts of the embryo during this developmental period. Drugs such as antiseizure medication and some antibiotics as well as chemicals, including lead, mercury, tobacco, and alcohol, also may result in structural and functional abnormalities.

4. Because of its size and rigidity, the fetal head has a major effect on the birth process. Which bones comprise the structure of the fetal skull? (Select all that apply.) a. Parietal b. Temporal c. Fontanel d. Occipital e. Femoral

ANS: A, B, D The fetal skull has two parietal bones, two temporal bones, an occipital bone, and a frontal bone. The fontanels are membrane-filled spaces.

8. The musculoskeletal system adapts to the changes that occur during pregnancy. A woman can expect to experience what change? A. Her center of gravity will shift backward. B. She will have increased lordosis. C. She will have increased abdominal muscle tone. D. She will notice decreased mobility of her pelvic joints.

ANS: B An increase in the normal lumbosacral curve (lordosis) develops, and a compensatory curvature in the cervicodorsal region develops to help the woman maintain her balance. The center of gravity shifts forward. She will have decreased muscle tone. She will notice increased mobility of her pelvic joints.

19. The U.S. Centers for Disease Control and Prevention (CDC) recommends that HPV be treated with patient-applied: A. miconazole ointment. B. topical podofilox 0.5% solution or gel. C. penicillin given intramuscularly for two doses. D. metronidazole by mouth.

ANS: B Available treatments are imiquimod, podophyllin, and podofilox. Miconazole ointment is used to treat athlete's foot. Intramuscular penicillin is used to treat syphilis. Metronidazole is used to treat bacterial vaginosis.

14. Maternal nutritional status is an especially significant factor of the many factors that influence the outcome of pregnancy because: A. it is very difficult to adjust because of people's ingrained eating habits. B. it is an important preventive measure for a variety of problems. C. women love obsessing about their weight and diets. D. a woman's preconception weight becomes irrelevant.

ANS: B Nutritional status draws so much attention not only for its effect on a healthy pregnancy and birth but also because significant changes are within relatively easy reach.

3. The nurse teaches a pregnant woman about the presumptive, probable, and positive signs of pregnancy. The woman demonstrates understanding of the nurse's instructions if she states that a positive sign of pregnancy is: A. a positive pregnancy test. B. fetal movement palpated by the nurse-midwife. C. Braxton Hicks contractions. D. quickening.

ANS: B Positive signs of pregnancy are attributed to the presence of a fetus, such as hearing the fetal heartbeat or palpating fetal movement. A positive pregnancy test and Braxton Hicks contractions are probable signs of pregnancy. Quickening is a presumptive sign of pregnancy.

19. The measurement of lecithin in relation to sphingomyelin (L/S ratio) is used to determine fetal lung maturity. Which ratio reflects maturity of the lungs? a. 1.4:1 b. 1.8:1 c. 2:1 d. 1:1

ANS: C A ratio of 2:1 indicates a two-to-one ratio of L/S, an indicator of lung maturity. Ratios of 1.4:1, 1.8:1, and 1:1 indicate immaturity of the fetal lungs.

10. The nurse providing care for a woman with gestational diabetes understands that a laboratory test for glycosylated hemoglobin Alc: A. is now done for all pregnant women, not just those with or likely to have diabetes. B. is a snapshot of glucose control at the moment. C. would be considered evidence of good diabetes control with a result of 5% to 6%. D. is done on the patient's urine, not her blood.

ANS: C A score of 5% to 6% indicates good control. This is an extra test for diabetic women, not one done for all pregnant women. This test defines glycemic control over the previous 4 to 6 weeks. Glycosylated hemoglobin level tests are done on the blood.

13. Nurses with an understanding of cultural differences regarding likely reactions to pain may be better able to help patients. Nurses should know that _____ women may be stoic until late in labor, when they may become vocal and request pain relief. a. Chinese b. Arab or Middle Eastern c. Hispanic d. African-American

ANS: C Hispanic women may be stoic early and more vocal and ready for medications later. Chinese women may not show reactions to pain. Medical interventions must be offered more than once. Arab or Middle Eastern women may be vocal in response to labor pain from the start. They may prefer pain medications. African-American women may express pain openly; use of medications for pain is more likely to vary with the individual.

1. When assessing a patient for amenorrhea, the nurse should be aware that this is unlikely to be caused by: A. anatomic abnormalities. B. type 1 diabetes mellitus. C. lack of exercise. D. hysterectomy.

ANS: C Lack of exercise is not a cause of amenorrhea. Strenuous exercise may cause amenorrhea. Anatomic abnormalities, type 1 diabetes mellitus, and hysterectomy all are possible causes of amenorrhea.

25. Which factor is known to increase the risk of gestational diabetes mellitus? A. Underweight before pregnancy B. Maternal age younger than 25 years C. Previous birth of large infant D. Previous diagnosis of type 2 diabetes mellitus

ANS: C Previous birth of a large infant suggests gestational diabetes mellitus. Obesity (BMI of 30 or greater) creates a higher risk for gestational diabetes. A woman younger than 25 years generally is not at risk for gestational diabetes mellitus. The person with type 2 diabetes mellitus already has diabetes and will continue to have it after pregnancy. Insulin may be required during pregnancy because oral hypoglycemia drugs are contraindicated during pregnancy.

15. A woman with asthma is experiencing a after birth hemorrhage. Which drug would not be used to treat her bleeding because it may exacerbate her asthma? A. Pitocin B. Nonsteroidal anti-inflammatory drugs (NSAIDs) C. Hemabate D. Fentanyl

ANS: C Prostaglandin derivatives should not be used to treat women with asthma because they may exacerbate symptoms. Pitocin would be the drug of choice to treat this woman's bleeding because it would not exacerbate her asthma. NSAIDs are not used to treat bleeding. Fentanyl is used to treat pain, not bleeding.

22. When counseling a patient about getting enough iron in her diet, the maternity nurse should tell her that: A. milk, coffee, and tea aid iron absorption if consumed at the same time as iron. B. iron absorption is inhibited by a diet rich in vitamin C. C. iron supplements are permissible for children in small doses. D. constipation is common with iron supplements.

ANS: D Constipation can be a problem. Milk, coffee, and tea inhibit iron absorption when consumed at the same time as iron. Vitamin C promotes iron absorption. Children who ingest iron can get very sick and even die.

16. In the current practice of childbirth preparation, emphasis is placed on: a. the Dick-Read (natural) childbirth method. b. the Lamaze (psychoprophylactic) method. c. the Bradley (husband-coached) method. d. having expectant parents attend childbirth preparation in any or no specific method.

ANS: D Encouraging expectant parents to attend childbirth preparation class is most important because preparation increases a woman's confidence and thus her ability to cope with labor and birth. Although still popular, the "method" format of classes is being replaced with other offerings such as Hypnobirthing and Birthing from Within.

13. One of the alterations in cyclic bleeding that occurs between periods is called: a. oligomenorrhea. b. menorrhagia. c. leiomyoma. d. metrorrhagia.

ANS: D Metrorrhagia is bleeding between periods. It can be caused by progestin injections and implants. Oligomenorrhea is infrequent or scanty menstruation. Menorrhagia is excessive menstruation. Leiomyoma is a common cause of excessive bleeding.

3. Which symptom is considered a warning sign and should be reported immediately by the pregnant woman to her health care provider? A. Nausea with occasional vomiting B. Fatigue C. Urinary frequency D. Vaginal bleeding

ANS: D Signs and symptoms that must be reported include severe vomiting, fever and chills, burning on urination, diarrhea, abdominal cramping, and vaginal bleeding. These symptoms may be signs of potential complications of the pregnancy. Nausea with occasional vomiting, fatigue, and urinary frequency are normal first-trimester complaints. Although they may be worrisome or annoying to the mother, they usually are not indications of pregnancy problems.

19. The nurse recognizes that a nonstress test (NST) in which two or more fetal heart rate (FHR) accelerations of 15 beats/min or more occur with fetal movement in a 20-minute period is: a. nonreactive. b. positive. c. negative. d. reactive.

ANS: D The NST is reactive (normal) when two or more FHR accelerations of at least 15 beats/min (each with a duration of at least 15 seconds) occur in a 20-minute period. A nonreactive result means that the heart rate did not accelerate during fetal movement. A positive result is not used with NST. Contraction stress test (CST) uses positive as a result term. A negative result is not used with NST. CST uses negative as a result term.

30. The nurse who is teaching a group of women about breast cancer would tell the women that: A. risk factors identify more than 50% of women who will develop breast cancer. B. nearly 90% of lumps found by women are malignant. C. 1 in 10 women in the United States will develop breast cancer in her lifetime. D. the exact cause of breast cancer is unknown.

ANS: D The exact cause of breast cancer is unknown. Risk factors help to identify less than 30% of women in whom breast cancer eventually will develop. Women detect about 90% of all breast lumps. Of this 90%, only 20% to 25% are malignant. One in eight women in the United States will develop breast cancer in her lifetime.

14. In her work with pregnant women of various cultures, a nurse practitioner has observed various practices that seemed strange or unusual. She has learned that cultural rituals and practices during pregnancy seem to have one purpose in common. Which statement best describes that purpose? A. To promote family unity B. To ward off the "evil eye" C. To appease the gods of fertility D. To protect the mother and fetus during pregnancy

ANS: D The purpose of all cultural practices is to protect the mother and fetus during pregnancy. Although many cultures consider pregnancy normal, certain practices are expected of women of all cultures to ensure a good outcome. Cultural prescriptions tell women what to do, and cultural proscriptions establish taboos. The purposes of these practices are to prevent maternal illness resulting from a pregnancy-induced imbalanced state and to protect the vulnerable fetus.

3. In evaluating the effectiveness of magnesium sulfate for the treatment of preterm labor, what finding would alert the nurse to possible side effects? a. Urine output of 160 mL in 4 hours b. Deep tendon reflexes 2+ and no clonus c. Respiratory rate of 16 breaths/min d. Serum magnesium level of 10 mg/dL

ANS: D The therapeutic range for magnesium sulfate management is 5 to 8 mg/dL. A serum magnesium level of 10 mg/dL could lead to signs and symptoms of magnesium toxicity, including oliguria and respiratory distress. Urine output of 160 mL in 4 hours, deep tendon reflexes 2+ with no clonus, and respiratory rate of 16 breaths/min are normal findings.

Concerning the third stage of labor, nurses should be aware that: a) The placenta eventually detaches itself from a flaccid uterus b) The duration of the third stage may be as short as 3 to 5 minutes c) It is important that the dark, roughened maternal surface of the placenta appear before the shiny fetal surface d) The major risk for women during the third stage is a rapid heart rate

B (The duration of the third stage may be as short as 3 to 5 minutes)

A woman with severe preeclampsia is receiving a magnesium sulfate infusion. The nurse becomes concerned after assessment when the woman exhibits: A. a sleepy, sedated affect. B. a respiratory rate of 10 breaths/min. C. deep tendon reflexes of 2+. D. absent ankle clonus.

B (a respiratory rate of 10 breaths/min. Because magnesium sulfate is a central nervous system (CNS) depressant, the client will most likely become sedated when the infusion is initiated. A respiratory rate of 10 breaths/min indicates that the client is experiencing respiratory depression (bradypnea) from magnesium toxicity. Deep tendon reflexes of 2+ are a normal finding. Absent ankle clonus is a normal finding.)

According to standard professional thinking, nurses should auscultate the fetal heart rate (FHR): a. Every 15 minutes in the active phase of the first stage of labor in the absence of risk factors. b. Every 20 minutes in the second stage, regardless of whether risk factors are present. c. Before and after ambulation and rupture of membranes. d. More often in a woman's first pregnancy.

C The FHR should be auscultated before and after administration of medications and induction of anesthesia. In the active phase of the first stage of labor, the FHR should be auscultated every 30 minutes if no risk factors are involved; with risk factors it should be auscultated every 15 minutes. In the second stage of labor the FHR should be auscultated every 15 minutes if no risk factors are involved; with risk factors it should be auscultated every 5 minutes. The fetus of a first-time mother is automatically at greater risk.

33. The most important reason for evaluating the pattern of weight gain in pregnancy is to: a. Prevent excessive adipose tissue deposits b. Identify potential nutritional problems or complications of pregnancy c. Assess the need to limit caloric intake in obese women d. Determine cultural influences on the woman's diet

b. Identify potential nutritional problems or complications of pregnancy

A pregnant patient with a normal fetal kick count has come for a regular nonstress testing session. The nurse notices that there are no heart accelerations after 40 minutes of testing. What diagnostic testing will the nurse include in the plan of care? 1 Contraction stress test 2 Biophysical profile test 3 Maternal serum analysis 4 Doppler blood flow test

1 A normal fetal kick count is an indication of fetal activity. The patient has undergone nonstress testing, which may have high false-positive rates. Therefore the patient may be scheduled for a contraction stress test. Biophysical profile testing allows detailed assessment of the physical and physiologic characteristics of the fetus. Because the kick count is normal, biophysical profile testing is not needed. Maternal serum analysis is done to determine fetal abnormalities. It is not advised in this case, because the fetal kick count indicates adequate fetal activity. Doppler blood flow analysis is a noninvasive test for analyzing fetal circulation. It cannot be used to assess the fetal heart rate.

Biochemical examination of the amniotic fluid of a pregnant patient yields the following results: lecithin-to-sphingomyelin (L/S) ratio, 2:1; surfactant-to-albumin (S/A) ratio, 60 mg/g; and phosphatidylglycerol (PG) present. What conclusions will the nurse draw from this report? 1 The fetal lungs are well developed. 2 The gestational age is 36 weeks. 3 The fetus has a neural tube defect. 4 The fetus has an open neural tube defect.

1 Biochemical findings such as an L/S ratio of 2:1, an S/A ratio of 60 mg/g, and the presence of PG in amniotic fluid indicate that the fetal lungs are well developed. The gestational age can be predicted only with the help of creatinine and lipid levels in the amniotic fluid. Creatinine levels greater than 2 mg/dL in amniotic fluid indicate that the gestational age is more than 36 weeks. The presence of alpha-fetoprotein (AFP) in the amniotic fluid indicates a neural tube defect in the fetus. The nurse needs to assess AFP levels in the amniotic fluid to determine whether the fetus has an open neural tube defect. A high AFP level in amniotic fluid after 15 weeks' gestation indicates that the fetus has an open neural tube defect.

The nurse sees a woman for the first time when she is 30 weeks pregnant. The woman has smoked throughout the pregnancy, and fundal height measurements now are suggestive of growth restriction in the fetus. In addition to ultrasound to measure fetal size, what tool is useful in confirming the diagnosis? 1 Doppler blood flow analysis 2 Contraction stress test (CST) 3 Amniocentesis 4 Daily fetal movement counts

1 Doppler blood flow analysis allows the examiner to study the blood flow noninvasively in the fetus and placenta. It is a helpful tool in the management of high risk pregnancies because of intrauterine growth restriction (IUGR), diabetes mellitus, multiple fetuses, or preterm labor. Because of the potential risk of inducing labor and causing fetal distress, a CST is not performed on a woman whose fetus is preterm. Indications for an amniocentesis include diagnosis of genetic disorders or congenital anomalies, assessment of pulmonary maturity, and the diagnosis of fetal hemolytic disease, not IUGR. Fetal kick count monitoring is performed to monitor the fetus in pregnancies complicated by conditions that may affect fetal oxygenation. Although this may be a useful tool at some point later in this woman's pregnancy, it is not used to diagnose IUGR.

A 4-week pregnant patient is undergoing an ultrasound. The report shows an absence of fetal heart activity. What does the nurse infer about the fetus from the report? 1 Normal finding 2 Congenital abnormality 3 impaired growth 4 Cardiac disorder

1 Fetal heart activity begins around 6 weeks, so 4 weeks is too early to detect fetal heart activity, and this is a normal finding. Absence of fetal heart activity at an advanced gestational age may indicate congenital anomalies, impaired growth, or cardiac disorders.

The nurse is reviewing lab values to determine Rh incompatibility between mother and fetus. Which specific lab result should the nurse assess? 1 Indirect Coombs test 2 Hemoglobin level 3 hCG level 4 Maternal serum alpha-fetoprotein (MSAFP

1 The indirect Coombs test is a screening tool for Rh incompatibility. If the maternal titer for Rh antibodies is greater than 1:8, amniocentesis for determination of bilirubin in amniotic fluid is indicated to establish the severity of fetal hemolytic anemia. Hemoglobin reveals the oxygen carrying capacity of the blood. hCG is the hormone of pregnancy. Maternal serum alpha-fetoprotein (MSAFP) levels are used as a screening tool for NTDs in pregnancy.

A patient in the sixth month of pregnancy expresses her wish to see the fetus. What investigation does the nurse suggest for the patient to help her see the fetus? 1 Ultrasonography 2 Magnetic resonance imaging (MRI) 3 Computed tomography (CT) 4 Nuchal translucency (NT)

1 Three-dimensional (3D) or four-dimensional (4D) ultrasonography is advisable for women who want to see the fetus. MRI cannot be used in this case because it requires the fetus to be still for a long period of time for a clear image. CT uses ionizing radiation for imaging, which can be harmful to the fetus. Therefore CT is contraindicated for fetal imaging. NT is a specific ultrasonography screening procedure used to test for genetic abnormalities in the fetus.

A patient in the first trimester of pregnancy undergoes a triple marker screening test. On reviewing the report, the nurse infers that the fetus may have Down syndrome. What clinical findings are noted by the nurse in the test reports? Select all that apply. 1 High levels of beta-human chorionic gonadotropin (β-hCG) 2 Low levels of pregnancy-associated placental protein (PAPP-A) 3 Low levels of inhibin-A in the fetal blood 4 Low levels of maternal serum alpha-fetoprotein (MSAFP) 5 Low levels of unconjugated estriol in the fetal blood

1, 2 Triple marker screening, which is performed in the first trimester of pregnancy, includes the measurement of two maternal biomarkers: PAPP-A and free β-hCG. High levels of free β-hCG and low levels of PAPP-A in the first trimester indicate that the fetus has Down syndrome, or trisomy 21.Inhibin-A is a placental hormone. Low levels of inhibin-A also indicate the possibility of Down syndrome, but inhibin-A levels are not measured in the triple marker screen; these levels are measured in quad screening. A low level of MSAFP and unconjugated estriol also indicate Down syndrome, but these can be measured only in the second and third trimesters.

While performing the fetal acoustic stimulation test (FAST) in a patient, the nurse observes that there is no fetal response even after 3 minutes of testing. Which test does the nurse suggest? 1 Amniocentesis 2 Biophysical profile (BPP) 3 Cordocentesis 4 Coombs' test

2 Lack of response after 3 minutes of FAST indicates that the fetus has low activity levels. In this situation, to accurately assess fetal activity, the nurse should recommend a BPP of the fetus. Amniocentesis helps detect genetic abnormalities in the fetus. Fetal activity cannot be determined using this technique. In cordocentesis, the umbilical blood is tested for Rh incompatibility and hemolytic anemia in the fetus. Coombs' test is used to determine the presence of antibody incompatibilities in the fetus and the mother.

The nurse is reviewing the contraction stress test (CST) reports of a pregnant patient. The nurse expects the fetus to have meconium-stained amniotic fluid. What would be the reason for that conclusion? 1 Negative CST results 2 Positive CST results 3 Suspicious CST results 4 Unsatisfactory CST results

2 Meconium is normally stored in the infant's intestines until after birth, but sometimes (in cases of fetal distress and hypoxia) it is expelled into the amniotic fluid before birth. The amniotic fluid is then said to be meconium stained. Fewer than three contractions in 10 minutes or late decelerations occurring with 50% or more of contractions constitute positive CST results. Positive CST results are associated with meconium-stained amniotic fluid. Negative CST results indicate that the fetus is normal. Suspicious or unsatisfactory CST results are not associated with any other fetal conditions.

What parameter does the nurse check in the amniocentesis report of a pregnant patient to assess fetal lung growth? 1 Alfa-fetoprotein (AFP) levels 2 Lecithin-to-sphingomyelin (L/S) ratio 3 Creatinine levels in the blood 4 Antibody titer in the blood

2 The L/S ratio indicates fetal lung maturity. AFP is assessed to check for the presence of neural defects. Presence of creatinine in the amniotic fluid indicates that the patient's gestational age is more than 36 weeks. The antibody titer is used to determine Rh incompatibility in the fetus.

The primary health care provider advises a pregnant woman to undergo a Doppler blood flow analysis after reviewing the amniocentesis reports. What clinical condition in the fetus could be the reason for this referral? 1 Down syndrome 2 Hemolytic anemia 3 Potter syndrome 4 Fetal hydrops

2 The presence of bilirubin in the amniotic fluid indicates the possibility of hemolytic anemia in the fetus. The degree of hemolytic anemia can be determined by using Doppler blood flow analysis. The presence of the placental hormone inhibin-A in the quad screen indicates Down syndrome. The amniotic fluid index values are used to detect Potter syndrome. Fetal hydrops is caused by polyhydramnios, which can be assessed by ultrasound scanning.

After reviewing the triple marker screen reports of a patient who is in the second trimester of pregnancy, the nurse concludes that the fetus has trisomy 18. What factors in the report led to the nurse's conclusion? Select all that apply. 1 Low level of inhibin-A in the maternal serum 2 Low level of unconjugated estriol in serum 3 Elevated nuchal translucency (NT) in the fetus 4 Low level of maternal human chorionic gonadotrophin (hCG) 5 Low level of maternal serum alpha-fetoprotein (MSAFP)

2, 4, 5 The triple marker screen measures the levels of three maternal serum markers: unconjugated estriol, hCG, and MSAFP. Low values of unconjugated estriol, hCG, and MSAFP indicate that the fetus has trisomy 18. The quad screen has an additional serum marker: inhibin-A. A low inhibin-A level indicates the possibility of Down syndrome. NT is not a serum marker protein. Moreover, elevated NT indicates that the fetus has a chromosomal abnormality but does not specifically indicate that the fetus has trisomy 18.

The nurse is assessing a pregnant patient and finds that her blood pressure is 150/90 mm Hg. What procedure does the nurse recommend for this patient? 1 Nuchal translucency (NT) test 2 Chorionic villus sampling (CVS) 3 Doppler blood flow analysis 4 Percutaneous umbilical blood sampling (PUBS)

3 Maternal hypertension can cause serious adverse effects on the fetus. A blood pressure reading of 150/90 mm Hg indicates that the mother is hypertensive. To assess the effect of maternal hypertension on the fetus, the nurse should refer the patient for a Doppler blood flow analysis. It is a noninvasive ultrasonic technique used to study fetal blood flow. NT is a technique used to assess genetic abnormalities in the fetus. CVS is a prenatal test used to diagnose structural defects in the fetus. PUBS is used to assess the fetal circulation.

After reviewing the standard ultrasound scan reports of a pregnant patient, the nurse advises the patient to undergo a specialized ultrasound scan. What is the nurse's rationale for this suggestion? 1 To estimate the amniotic fluid volume 2 To identify the detailed fetal anatomy 3 To assess for physiologic abnormalities 4 To assess for fetal genetic abnormalities

3 Specialized or targeted ultrasound scans are performed only if a patient is suspected of carrying an anatomically or physiologically abnormal fetus. Limited ultrasound examination is used to estimate the amniotic fluid volume. Standard ultrasound scan is used to see the detailed anatomy of the fetus. Ultrasound scan is not used to find genetic abnormalities in the fetus.

The nurse finds that the nonstress test of a pregnant patient is nonreactive. Which factor in the report might have led the nurse to this finding? 1 No qualifying accelerations in a 20-minute period 2 Two qualifying accelerations in a 20-minute period 3 Less than two qualifying accelerations in a 20-minute period 4 More than two qualifying accelerations in a 20-minute period

3 The nonstress test is the most widely used technique for prenatal evaluation of the fetus. The results are either nonreactive or reactive. In a nonreactive test, there are less than two qualifying accelerations of the fetal heart rate in a 20-minute period. Absence of fetal heart rate accelerations during the nonstress test indicates that the fetus is sleeping. In a reactive test, there are at least two qualifying accelerations in a 20-minute time period. More than two fetal heart rate accelerations within a 20-minute time period also would be considered a reactive test.

The amniotic fluid index (AFI) of a pregnant patient is 3 cm. What clinical information related to the fetus does the nurse infer from this? 1 Neural tube defect 2 Fetal hydrops 3 Renal defects 4 Low activity level

3 The normal value of AFI is 10 cm or greater, with the upper limit of normal around 25 cm. An AFI less than 5 cm indicates oligohydramnios. This condition is associated with renal agenesis in the fetus. A high AFI indicates neural tube defects and fetal hydrops. The AFI is not directly related to fetal movement. Fetal activity can be assessed using ultrasonography.

After reviewing the biophysical profile (BPP) reports of a pregnant patient close to term, the nurse advises the patient to repeat the test on a weekly basis. What BPP score did the nurse find in the report? 1. 1 2. 4 3. 6 4. 9

4 If the BPP score is 8 to 10, then the test should be repeated weekly or twice weekly. If the BPP score is 0 to 2, then chronic asphyxia may be suspected. In this case the testing time should be extended to 120 minutes. If the BPP score is 4 after 36 weeks' gestation, then clinical conditions exist that may lead to an eminent delivery. If the BPP score is 4 before 32 weeks' gestation, the test should be repeated. If the BPP score is 6 at 36 to 37 weeks' gestation with positive fetal pulmonary testing, then delivery can be performed. If the BPP score is 6 before 36 weeks' gestation with negative pulmonary testing, then BPP can be repeated in 4 to 6 hours, and if oligohydramnios is present, then delivery can be done. The BPP provides an insight into fetal maturity and well-being and as such should be used as a diagnostic tool to plan and evaluate management of care. Findings are related to several factors involving both maternal and fetal characteristics.

The biophysical profile (BPP) testing report of a pregnant patient gives the following information: one episode of fetal breathing movement lasting for 30 seconds in a 30-minute observation; three limb movements of the fetus in 30 minutes; an amniotic fluid index greater than 5; a reactive nonstress test; and a BPP score of 1. The test is performed for 120 minutes. What does the nurse expect the primary health care provider to do? 1 Extend the test time to 120 minutes. 2 Repeat the test twice a week. 3 Repeat the test in 4 to 6 hours. 4 Consider delivery of the fetus.

4 If the BPP score is less than 2, regardless of gestational age, delivery can be performed. If the BPP score is 0 to 2 and chronic asphyxia is suspected, then testing time should be extended to 120 minutes. If the BPP score is 8 to 10 and a low risk for chronic asphyxia is suspected, then the test should be repeated at twice-weekly intervals. If the fetal pulmonary test result is negative and the BPP score is 6, then the BPP profile should be repeated in 4 to 6 hours.

Nurses should be aware of the strengths and limitations of various biochemical assessments during pregnancy, including that: 1 chorionic villus sampling (CVS) is becoming more popular because it provides early diagnosis. 2 screening for maternal serum alpha-fetoprotein (MSAFP) levels is recommended only for women at risk for neural tube defects. 3 percutaneous umbilical blood sampling (PUBS) is one of the quad-screen tests for Down syndrome. 4 MSAFP is a screening tool only; it identifies candidates for more definitive procedures.

4 MSAFP is a screening tool, not a diagnostic tool. Further diagnostic testing is indicated after an abnormal MSAFP. CVS does provide a rapid result, but it is declining in popularity because of advances in noninvasive screening techniques. MSAFP screening is recommended for all pregnant women. MSAFP, not PUBS, is part of the quad-screen tests for Down syndrome.

A woman who is at 36 weeks of gestation is having a nonstress test. Which statement by the woman indicates a correct understanding of the test? 1 "I will need to have a full bladder for the test to be done accurately." 2 "I should have my husband drive me home after the test because I may be nauseous." 3 "This test will help to determine if the baby has Down syndrome or a neural tube defect." 4 "This test will observe for fetal activity and an acceleration of the fetal heart rate to determine the well-being of the baby."

4 The nonstress test is one of the most widely used techniques to determine fetal well-being and is accomplished by monitoring fetal heart rate in conjunction with fetal activity and movements. An ultrasound is the test that requires a full bladder. An amniocentesis is the test that a pregnant woman should be driven home afterward. A maternal alpha-fetoprotein test is used in conjunction with unconjugated estriol levels, and human chorionic gonadotropin helps to determine Down syndrome.

A 40-year-old woman with a high body mass index is 10 weeks pregnant. Which diagnostic tool is appropriate to suggest to her at this time? 1 Biophysical profile 2 Amniocentesis 3 Maternal serum alpha-fetoprotein (MSAFP) 4 Transvaginal ultrasound

4 Transvaginal ultrasound is useful for obese women whose thick abdominal layers cannot be penetrated adequately with the abdominal approach. A biophysical profile is a method of biophysical assessment of fetal well-being in the third trimester. An amniocentesis is performed after the fourteenth week of pregnancy. A MSAFP test is performed from week 15 to week 22 of the gestation (weeks 16 to 18 are ideal). An ultrasound is the method of biophysical assessment of the infant that is performed at this gestational age.

A primary health care provider is performing a transabdominal amniocentesis procedure in a pregnant patient. Arrange the steps of the amniocentesis procedure in the correct order. 1. Separate the supernatant fluid and cellular components. 2. Collect the cellular components for chromosomal studies. 3. Centrifuge the collected amniotic fluid. 4. Collect the amniotic fluid under ultrasonographic visualization. 5. Collect the supernatant for chemical analysis.

4, 3, 1, 5, 2 Amniocentesis is performed to obtain the amniotic fluid, which contains the fetal cells. Amniotic fluid can be collected transabdominally under ultrasonographic visualization. After the amniotic fluid is collected, the fluid should be centrifuged. Centrifugation is done to separate the supernatant fluid and cellular components. Because the supernatant fluid has a lighter weight, it collects in the upper part of the centrifuge tube. This supernatant fluid is collected first and is sent for chemical analysis. Cellular components, which lie in the bottom of the centrifuge tube, are then collected to perform chromosomal analysis.

A pregnant patient reports abdominal pain in the right lower quadrant, along with nausea and vomiting. The patient's urinalysis report shows an absence of any urinary tract infection in the patient. A chest x-ray also rules out lower-lobe pneumonia. Which condition does the nurse suspect in the patient? A. Appendicitis B. Cholelithiasis C. Placenta previa D. Uterine rupture

A (Abdominal pain in the right lower quadrant, accompanied by nausea and vomiting, indicates appendicitis in a pregnant patient. Cholelithiasis is characterized by right upper quadrant pain. Placenta previa is a condition wherein the placenta is implanted in the lower uterine segment covering the cervix, which causes bleeding when the cervix dilates. Uterine rupture is seen in a pregnant patient as a result of trauma, which may cause fetal death.)

In caring for the woman with disseminated intravascular coagulation (DIC), what order should the nurse anticipate? A. Administration of blood B. Preparation of the woman for invasive hemodynamic monitoring C. Restriction of intravascular fluids D. Administration of steroids

A (Administration of blood Primary medical management in all cases of DIC involves correction of the underlying cause, volume replacement, blood component therapy, optimization of oxygenation and perfusion status, and continued reassessment of laboratory parameters. Central monitoring would not be ordered initially in a woman with DIC because this can contribute to more areas of bleeding. Management of DIC includes volume replacement, not volume restriction. Steroids are not indicated for the management of DIC.)

A woman at 37 weeks of gestation is admitted with a placental abruption after a motor vehicle accident. Which assessment data are most indicative of her condition worsening? A. Pulse (P) 112, respiration (R) 32, blood pressure (BP) 108/60; fetal heart rate (FHR) 166--178 B. P 98, R 22, BP 110/74; FHR 150--162 C. P 88, R 20, BP 114/70; FHR 140--158 D. P 80, R 18, BP 120/78; FHR 138--150

A (Bleeding is the most dangerous problem, which impacts the mother's well-being as well as that of her fetus. The decreasing blood volume would cause increases in pulse and respirations and a decrease in blood pressure. The fetus often responds to decreased oxygenation as a result of bleeding, causing a decrease in perfusion. This causes the fetus' heart rate to increase above the normal range of 120--160 beats per minute. The other options have measurements that are in the "normal" range and would not reflect a deterioration of the patient's physical status. Test-Taking Tip: Do not worry if you select the same numbered answer repeatedly, because there usually is no pattern to the answers.)

A fetus is in the LST position. The nurse interprets this as indicating which of the following as the presenting part? a) Buttocks b) Fetal head c) Shoulder d) Chin

A (Buttocks Rationale: The letter "S" indicates the sacrum or buttocks as the presenting part. The fetal head would be noted by the letter "O," indicating occiput. The fetal chin would be noted by the letter "M," indicating mentum. The fetal shoulder would be noted by the letter "A," indicating the acromion process.)

A petite, 5-foot tall, 95-pound woman who is 28 years old is about to deliver her first child and would like to have a vaginal delivery. She has two sisters, both of whom have given birth vaginally. She has gained 25 pounds during a normal, uneventful pregnancy. What type of pelvis would a nurse expect this woman to have upon assessment of the patient? a) Cannot be determined b) Android c) Platypelloid d) Gynecoid

A (Cannot be determined Rationale: Pelvis shape cannot be determined by the information included in the statement. Early in the pregnancy, particularly if a woman has never delivered a baby vaginally, the practitioner may take pelvic measurements to estimate the size of the true pelvis. This helps to determine if the size is adequate for vaginal delivery. However, these measurements do not consistently predict which women will have difficulty delivering vaginally, so most practitioners allow the woman to labor and attempt a vaginal birth.)

A client in labor is agitated and nervous about the birth of her child. The nurse explains to the client that fear and anxiety cause the release of certain compounds which can prolong labor. Which of the following is the nurse referring to in the explanation? a) Catecholamines b) Relaxin c) Prostaglandins d) Oxytocin

A (Catecholamines Rationale: Fear and anxiety cause the release of catecholamines, such as norepinephrine and epinephrine which stimulate the adrenergic receptors of the myometrium. This in turn interferes with effective uterine contractions and results in prolonged labor. Estrogen promotes the release of prostaglandins and oxytocin. Relaxin is a hormone that is involved in producing backache by acting on the pelvic joints. Prostaglandins, oxytocin and relaxin are not produced due to fear or anxiety in clients during labor.)

The nurse notes that the fetal head is at the vaginal opening and does not regress between contractions. The nurse interprets this finding as which of the following? a) Crowning b) Engagement c) Descent d) Restitution

A (Crowning Rationale: Crowning occurs when the top of the fetal head appears at the vaginal orifice and no longer regresses between contractions. Engagement occurs when the greatest transverse diameter of the head passes through the pelvic inlet. Descent is the downward movement of the fetal head until it is within the pelvic inlet. Restitution or external rotation occurs after the head is born and free of resistance. It untwists, causing the occiput to move about 45 degrees back to its original left or right position.)

The nurse would expect which maternal cardiovascular finding during labor? a. Increased cardiac output b. Decreased pulse rate c. Decreased white blood cell (WBC) count d. Decreased blood pressure

A (During each contraction, 400 mL of blood is emptied from the uterus into the maternal vascular system. This increases cardiac output by about 51% above baseline pregnancy values at term. The heart rate increases slightly during labor. The WBC count can increase during labor. During the first stage of labor, uterine contractions cause systolic readings to increase by about 10 mm Hg. During the second stage, contractions may cause systolic pressures to increase by 30 mm Hg and diastolic readings to increase by 25 mm Hg.)

Fetal circulation can be affected by many factors during labor. Accurate assessment of the laboring woman and fetus requires knowledge of these expected adaptations. Which factor will not affect fetal circulation during labor? a) Fetal position b) Uterine contractions c) Blood pressure d) Umbilical cord blood flow

A (Fetal position)

Which description of the four stages of labor is correct for both definition and duration? a. First stage: onset of regular uterine contractions to full dilation; less than 1 hour to 20 hours b. Second stage: full effacement to 4 to 5 cm; visible presenting part; 1 to 2 hours c. Third state: active pushing to birth; 20 minutes (multiparous women), 50 minutes (first-timer) d. Fourth stage: delivery of the placenta to recovery; 30 minutes to 1 hour

A (Full dilation may occur in less than 1 hour, but in first-time pregnancies it can take up to 20 hours. The second stage extends from full dilation to birth and takes an average of 20 to 50 minutes, although 2 hours is still considered normal. The third stage extends from birth to expulsion of the placenta and usually takes a few minutes. The fourth stage begins after expulsion of the placenta and lasts until homeostasis is reestablished (about 2 hours).)

A woman diagnosed with marginal placenta previa gave birth vaginally 15 minutes ago. At the present time, she is at the greatest risk for: A. hemorrhage. B. infection. C. urinary retention. D. thrombophlebitis.

A (Hemorrhage is the most immediate risk because the lower uterine segment has limited ability to contract to reduce blood loss. Infection is a risk because of the location of the placental attachment site; however, it is not a priority concern at this time. Placenta previa poses no greater risk for urinary retention than with a normally implanted placenta. There is no greater risk for thrombophlebitis than with a normally implanted placenta.)

The quantitative human chorionic gonadotropin (β-hCG) levels are high in a patient who is on methotrexate therapy for dissolving abdominal pregnancy. Which instruction does the nurse give to this patient? A. "Avoid sexual activity." B. "Avoid next pregnancy." C. "Avoid feeling sad and low." D. "Take folic acid without fail."

A (High β-hCG levels indicate that the abdominal pregnancy is not yet dissolved. Therefore the nurse advises the patient to avoid sexual activity until the β-hCG levels drop and the pregnancy is dissolved completely. If the patient engages in vaginal intercourse, the pelvic pressure may rupture the mass and cause pain. Abdominal pregnancy increases the chances of infertility or recurrent ectopic pregnancy in patients. However, the nurse need not instruct the patient to avoid further pregnancy, because it may increase the feelings of sadness and guilt in the patient. The nurse encourages the patient to share feelings of guilt or sadness related to pregnancy loss. Folic acid is contraindicated with methotrexate therapy, because it may exacerbate ectopic rupture.)

A woman with severe preeclampsia has been receiving magnesium sulfate by intravenous infusion for 8 hours. The nurse assesses the woman and documents the following findings: temperature 37.1° C, pulse rate 96 beats/min, respiratory rate 24 breaths/min, blood pressure 155/112 mm Hg, 3+ deep tendon reflexes, and no ankle clonus. The nurse calls the physician, anticipating an order for: A. hydralazine. B. magnesium sulfate bolus. C. diazepam. D. morphine

A (Hydralazine is an antihypertensive commonly used to treat hypertension in severe preeclampsia. An additional bolus of magnesium sulfate may be ordered for increasing signs of central nervous system irritability related to severe preeclampsia (e.g., clonus) or if eclampsia develops. Diazepam sometimes is used to stop or shorten eclamptic seizures. Calcium gluconate is used as the antidote for magnesium sulfate toxicity. The patient is not currently displaying any signs or symptoms of magnesium toxicity.)

After being rehydrated in the emergency department, a 24 year-old primipara in her 18th week of pregnancy is at home and is to rest at home for the next two days and take in small but frequent fluids and food as possible. Discharge teaching at the hospital by the nurse has been effective if the patient makes which statement? "A. I'm going to eat five to six small servings per day, which contain such foods and fluids as tea, crackers, or a few bites of baked potato." B. "A strip of bacon and a fried egg will really taste good as long as I eat them slowly." C. "As long as I eat small amounts and allow enough time for digestion, I can eat almost anything, like barbequed chicken or spaghetti." D. "I'm going to stay only on clear fluids for the next 24 hours and then add dairy products like eggs and milk."

A (Once the vomiting has stopped, feedings are started in small amounts at frequent intervals. In the beginning, limited amounts of oral fluids and bland foods such as crackers, toast, or baked chicken are offered. Clear fluids alone do not contain enough calories and contain no protein. Most women are able to take nourishment by mouth after several days of treatment. They should be encouraged to eat small, frequent meals and foods that sound appealing (e.g., nongreasy, dry, sweet, and salty foods). Test-Taking Tip: Many times the correct answer is the longest alternative given, but do not count on it. NCLEX item writers (those who write the questions) are also aware of this and attempt to avoid offering you such "helpful hints.")

Which condition is seen in a pregnant patient if uterine artery Doppler measurements in the second trimester of pregnancy are abnormal? A. Preeclampsia B. HELLP syndrome C. Molar pregnancy D. Gestational hypertension

A (Preeclampsia is a condition in which patients develop hypertension and proteinuria after 20 weeks' gestation. It can be diagnosed if uterine artery Doppler measurements in the second trimester of pregnancy are abnormal. HELLP syndrome is characterized by hemolysis (H), elevated liver enzymes (EL), and low platelet count (LP) in a patient with preeclampsia. Molar pregnancy refers to the growth of the placental trophoblast due to abnormal fertilization. Gestational hypertension is a condition in which hypertension develops in a patient after 20 weeks' gestation.)

With regard to primary and secondary powers, the maternity nurse should understand that: a) Primary powers are responsible for effacement and dilation of the cervix b) Effacement generally is well ahead of dilation in women giving birth for the first time; they are more together in subsequent pregnancies c) Scarring of the cervix caused by a previous infection or surgery may make the delivery a bit more painful, but it should not slow or inhibit dilation d) Pushing in the second stage of labor is more effective if the woman can breathe deeply and control some of her involuntary needs to push, as the nurse directs

A (Primary powers are responsible for effacement and dilation of the cervix)

When teaching a group of nursing students about uterine contractions, which of the following would the instructor include as a typical feature? a) Retracting of the lower segment b) Pulling down of the cervical portion c) Thinning out of the upper segment d) Contracting stronger in the lower segment

A (Retracting of the lower segment Rationale: The nurse should identify retraction of the lower segment of the uterus as a feature of typical uterine contractions. As labor progresses the uterine contractions become stronger. The upper segment of the uterus contracts more actively than the lower segment. The lower segment retracts, pulling up the cervix. The upper segment thickens with time and the lower segment thins.)

Which condition in a pregnant patient with severe preeclampsia is an indication for administering magnesium sulfate? A. Seizure activity B. Renal dysfunction C. Pulmonary edema D. Low blood pressure (BP)

A (Severe preeclampsia may cause seizure activity or eclampsia in the patient, which is treated with magnesium sulfate. Magnesium sulfate is not administered for renal dysfunction and can cause magnesium toxicity in the patient. Pulmonary enema can be prevented by restricting the patient's fluid intake to 125 mL/hr. Increasing magnesium toxicity can cause low BP in the patient.)

A nurse is teaching a group of nursing students about the role of progesterone in labor. Which of the following should the nurse explain as the function of progesterone? a) Suppresses the uterine irritability throughout pregnancy b) Promotes oxytocin production from the posterior pituitary c) Sensitizes the uterus to effects of oxytocin on the myometrium d) Stimulates smooth muscle contraction in the uterus

A (Suppresses the uterine irritability throughout pregnancy Rationale: The function of progesterone is to suppress uterine irritability throughout pregnancy. The function of estrogen is to promote oxytocin production and to sensitize the uterus to the effects of oxytocin. Prostaglandin, and not progesterone, stimulates the smooth muscle contractions in the uterus.)

The nurse is caring for a pregnant patient who is receiving antibiotic therapy to treat a urinary tract infection (UTI). Which dietary changes does the nurse suggest for the pregnant patient who is receiving antibiotic therapy for UTI? A. "Include yogurt, cheese, and milk in your diet." B. "Avoid folic acid supplements until the end of therapy." C. "Include vitamins C and E supplementation in your diet." D. "Reduce your dietary fat intake by 40 to 50 g per day.

A (The antibiotic therapy kills normal flora in the genitourinary tract, as well as pathologic organisms. Therefore the nurse instructs the patient to include yogurt, cheese, and milk in daily diet because they contain active acidophilus cultures. Folic acid should not be avoided, because it may affect the fetal development. Vitamins C and E supplementation is usually included in the diet to treat preeclampsia in a patient. Dietary fat is reduced in patients with cholecystitis or cholelithiasis, because it may cause epigastric pain.)

To care for a laboring woman adequately, the nurse understands that the __________ stage of labor varies the most in length? a. First b. Third c. Second d. Fourth

A (The first stage of labor is considered to last from the onset of regular uterine contractions to full dilation of the cervix. The first stage is much longer than the second and third stages combined. In a first-time pregnancy the first stage of labor can take up to 20 hours. The second stage of labor lasts from the time the cervix is fully dilated to the birth of the fetus. The average length is 20 minutes for a multiparous woman and 50 minutes for a nulliparous woman. The third stage of labor lasts from the birth of the fetus until the placenta is delivered. This stage may be as short as 3 minutes or as long as 1 hour. The fourth stage of labor, recovery, lasts about 2 hours after delivery of the placenta.)

In relation to primary and secondary powers, the maternity nurse comprehends that: a. Primary powers are responsible for effacement and dilation of the cervix. b. Effacement generally is well ahead of dilation in women giving birth for the first time; they are closer together in subsequent pregnancies. c. Scarring of the cervix caused by a previous infection or surgery may make the delivery a bit more painful, but it should not slow or inhibit dilation. d. Pushing in the second stage of labor is more effective if the woman can breathe deeply and control some of her involuntary needs to push, as the nurse directs.

A (The primary powers are responsible for dilation and effacement; secondary powers are concerned with expulsion of the fetus. Effacement generally is well ahead of dilation in first-timers; they are closer together in subsequent pregnancies. Scarring of the cervix may slow dilation. Pushing is more effective and less fatiguing when the woman begins to push only after she has the urge to do so.)

The woman has just delivered a healthy baby boy, but the placenta has not yet delivered. What stage of labor does this scenario represent? a) Third b) Second c) Fourth d) First

A (Third Rationale: Stage 3 begins with the birth of the baby and ends with delivery of the placenta.)

A nurse is assisting a client who is in the first stage of labor. Which of the following principles should the nurse keep in mind to help make this client's labor and birth as natural as possible? a) Women should be able to move about freely throughout labor b) The support person's access to the client should be limited to prevent the client from becoming overwhelmed c) Routine intravenous fluid should be implemented d) A woman should be allowed to assume a supine position

A (Women should be able to move about freely throughout labor Rationale: Six major concepts that make labor and birth as natural as possible are as follows: 1) labor should begin on its own, not be artificially induced; 2) women should be able to move about freely throughout labor, not be confined to bed; 3) women should receive continuous support from a caring other during labor; 4) no interventions such as intravenous fluid should be used routinely; 5) women should be allowed to assume a nonsupine position such as upright and side-lying for birth; and 6) mother and baby should be housed together after the birth, with unlimited opportunity for breast-feeding.)

A woman diagnosed with marginal placenta previa gave birth vaginally 15 minutes ago. At the present time she is at the greatest risk for: A. hemorrhage. B. infection. C. urinary retention. D. thrombophlebitis.

A (hemorrhage. Hemorrhage is the most immediate risk because the lower uterine segment has limited ability to contract to reduce blood loss. Infection is a risk because of the location of the placental attachment site; however, it is not a priority concern at this time. Placenta previa poses no greater risk for urinary retention than with a normally implanted placenta. There is no greater risk for thrombophlebitis than with a normally implanted placenta.)

A woman with severe preeclampsia has been receiving magnesium sulfate by IV infusion for 8 hours. The nurse assesses the woman and documents the following findings: temperature 37.1° C, pulse rate 96 beats/min, respiratory rate 24 breaths/min, blood pressure 155/112 mm Hg, 3+ deep tendon reflexes, and no ankle clonus. The nurse calls the physician, anticipating an order for: A. hydralazine. B. magnesium sulfate bolus . C. diazepam. D. calcium gluconate.

A (hydralazine. Hydralazine is an antihypertensive commonly used to treat hypertension in severe preeclampsia. An additional bolus of magnesium sulfate may be ordered for increasing signs of central nervous system irritability related to severe preeclampsia (e.g., clonus) or if eclampsia develops. Diazepam sometimes is used to stop or shorten eclamptic seizures. Calcium gluconate is used as the antidote for magnesium sulfate toxicity. The client is not currently displaying any signs or symptoms of magnesium toxicity.)

The nurse providing care for the laboring woman should understand that amnioinfusion is used to treat: a. Variable decelerations. c. Fetal bradycardia. b. Late decelerations. d. Fetal tachycardia.

A Amnioinfusion is used during labor either to dilute meconium-stained amniotic fluid or to supplement the amount of amniotic fluid to reduce the severity of variable decelerations caused by cord compression. Amnioinfusion has no bearing on late decelerations, fetal bradycardia, or fetal tachycardia alterations in fetal heart rate (FHR) tracings.

Which deceleration of the fetal heart rate would not require the nurse to change the maternal position? a. Early decelerations b. Late decelerations c. Variable decelerations d. It is always a good idea to change the woman's position.

A Early decelerations (and accelerations) generally do not need any nursing intervention. Late decelerations suggest that the nurse should change the maternal position (lateral); variable decelerations also require a maternal position change (side to side). Although changing positions throughout labor is recommended, it is not required in response to early decelerations.

The nurse caring for the laboring woman should understand that early decelerations are caused by: a. Altered fetal cerebral blood flow. c. Uteroplacental insufficiency. b. Umbilical cord compression. d. Spontaneous rupture of membranes.

A Early decelerations are the fetus's response to fetal head compression. Variable decelerations are associated with umbilical cord compression. Late decelerations are associated with uteroplacental insufficiency. Spontaneous rupture of membranes has no bearing on the fetal heart rate unless the umbilical cord prolapses, which would result in variable or prolonged bradycardia.

Fetal well-being during labor is assessed by: a. The response of the fetal heart rate (FHR) to uterine contractions (UCs). b. Maternal pain control. c. Accelerations in the FHR. d. An FHR above 110 beats/min.

A Fetal well-being during labor can be measured by the response of the FHR to UCs. In general, reassuring FHR patterns are characterized by an FHR baseline in the range of 110 to 160 beats/min with no periodic changes, a moderate baseline variability, and accelerations with fetal movement. Maternal pain control is not the measure used to determine fetal well-being in labor. Although FHR accelerations are a reassuring pattern, they are only one component of the criteria by which fetal well-being is assessed. Although an FHR above 110 beats/min may be reassuring, it is only one component of the criteria by which fetal well-being is assessed. More information would be needed to determine fetal well-being.

Which maternal condition is considered a contraindication for the application of internal monitoring devices? a. Unruptured membranes b. External monitors in current use c. Cervix dilated to 4 cm d. Fetus with a known heart defect

A In order to apply internal monitoring devices, the membranes must be ruptured. Cervical dilation of 4 cm permits the insertion of fetal scalp electrodes and intrauterine catheter. The external monitor can be discontinued after the internal ones are applied. A compromised fetus should be monitored with the most accurate monitoring devices.

While evaluating an external monitor tracing of a woman in active labor, the nurse notes that the fetal heart rate (FHR) for five sequential contractions begins to decelerate late in the contraction, with the nadir of the decelerations occurring after the peak of the contraction. The nurse's first priority is to: a. Change the woman's position. c. Assist with amnioinfusion. b. Notify the care provider. d. Insert a scalp electrode.

A Late decelerations may be caused by maternal supine hypotension syndrome. They usually are corrected when the woman turns on her side to displace the weight of the gravid uterus from the vena cava. If the fetus does not respond to primary nursing interventions for late decelerations, the nurse would continue with subsequent intrauterine resuscitation measures, including notifying the care provider. An amnioinfusion may be used to relieve pressure on an umbilical cord that has not prolapsed. The FHR pattern associated with this situation most likely reveals variable deceleration. A fetal scalp electrode would provide accurate data for evaluating the well-being of the fetus; however, this is not a nursing intervention that would alleviate late decelerations, nor is it the nurse's first priority.

The nurse caring for a laboring woman is aware that maternal cardiac output can be increased by: a. Change in position. c. Regional anesthesia. b. Oxytocin administration. d. Intravenous analgesic.

A Maternal supine hypotension syndrome is caused by the weight and pressure of the gravid uterus on the ascending vena cava when the woman is in a supine position. This reduces venous return to the woman's heart, as well as cardiac output, and subsequently reduces her blood pressure. The nurse can encourage the woman to change positions and avoid the supine position. Oxytocin administration, regional anesthesia, and intravenous analgesic may reduce maternal cardiac output.

Perinatal nurses are legally responsible for: a. Correctly interpreting fetal heart rate (FHR) patterns, initiating appropriate nursing interventions, and documenting the outcomes. b. Greeting the client on arrival, assessing her, and starting an intravenous line. c. Applying the external fetal monitor and notifying the care provider. d. Making sure that the woman is comfortable.

A Nurses who care for women during childbirth are legally responsible for correctly interpreting FHR patterns, initiating appropriate nursing interventions based on those patterns, and documenting the outcomes of those interventions. Greeting the client, assessing her, and starting an IV; applying the external fetal monitor and notifying the care provider; and making sure the woman is comfortable may be activities that a nurse performs, but they are not activities for which the nurse is legally responsible.

When using intermittent auscultation (IA) to assess uterine activity, the nurse should be cognizant that: a. The examiner's hand should be placed over the fundus before, during, and after contractions. b. The frequency and duration of contractions is measured in seconds for consistency. c. Contraction intensity is given a judgment number of 1 to 7 by the nurse and client together. d. The resting tone between contractions is described as either placid or turbulent.

A The assessment is done by palpation; duration, frequency, intensity, and resting tone must be assessed. The duration of contractions is measured in seconds; the frequency is measured in minutes. The intensity of contractions usually is described as mild, moderate, or strong. The resting tone usually is characterized as soft or relaxed.

In assisting with the two factors that have an effect on fetal status (i.e., pushing and positioning), nurses should: a. Encourage the woman's cooperation in avoiding the supine position. b. Advise the woman to avoid the semi-Fowler position. c. Encourage the woman to hold her breath and tighten her abdominal muscles to produce a vaginal response. d. Instruct the woman to open her mouth and close her glottis, letting air escape after the push.

A The woman should maintain a side-lying position. The semi-Fowler position is the recommended side-lying position with a lateral tilt to the uterus. The Valsalva maneuver, which encourages the woman to hold her breath and tighten her abdominal muscles, should be avoided. Both the mouth and glottis should be open, letting air escape during the push.

A pregnant patient in the first trimester reports spotting of blood with the cervical os closed and mild uterine cramping. What does the nurse need to assess? Select all that apply. A. Progesterone levels B. Transvaginal ultrasounds C. Human chorionic gonadotropin (hCG) measurement D. Blood pressure E. Kleihauer-Betke (KB) test reports

A, B, C (The spotting of blood with the cervical os closed and mild uterine cramping in the first trimester indicates a threatened miscarriage. Therefore the nurse needs to assess progesterone levels, transvaginal ultrasounds, and measurement of hCG to determine whether the fetus is alive and within the uterus. Blood pressure measurements do not help determine the fetal status. KB assay is prescribed to identify fetal-to-maternal bleeding, usually after a trauma.)

A 24-year-old primipara, who is 18 weeks pregnant, has been having increasing vomiting since she was 8 weeks pregnant. Upon arrival at the emergency department, her skin turgor is diminished, temperature is 99.2F (o), pulse is 102, respiration is 18, blood pressure is 102/68, and she has deep furrows on her tongue. What would the nurse expect to do to care for her? Select all that apply. A. Start an intravenous infusion. B. Check her urine for ketones C. Cross match blood for a transfusion. D. Obtain a complete history. E. Obtain blood for a complete blood count

A, B, D, E (Whenever a pregnant woman has nausea and vomiting, the first priority is a thorough assessment to determine the severity of the problem. In most cases the woman should be told to come immediately to the health care provider's office or the emergency department because the severity of the illness often is difficult to determine by telephone conversation. The assessment should include frequency, severity, and duration of episodes of nausea and vomiting. If the woman reports vomiting, the assessment also should include the approximate amount and color of the vomitus. The woman is asked to report any precipitating factors relating to the onset of her symptoms. Any pharmacologic or nonpharmacologic treatment measures used should be recorded. Prepregnancy weight and documented weight gain or loss during pregnancy are important to note. The woman's weight and vital signs are measured, and a complete physical examination is performed, with attention to signs of fluid and electrolyte imbalance and nutritional status. The most important initial laboratory test to be obtained is a determination of ketonuria. Other laboratory tests that may be prescribed are a urinalysis, a complete blood cell count, electrolytes, liver enzymes, and bilirubin levels. At this time, there is no supportive evidence that a blood transfusion is required. Based on provided objective data that the patient has deep furrows on her tongue, this may suggest a vitamin B deficiency which should be investigated further.)

Signs that precede labor include (Select all that apply): a. Lightening. b. Exhaustion. c. Bloody show. d. Rupture of membranes. e. Decreased fetal movement.

A, C, D (Signs that precede labor may include lightening, urinary frequency, backache, weight loss, surge of energy, bloody show, and rupture of membranes. Many women experience a burst of energy before labor. A decrease in fetal movement is an ominous sign that does not always correlate with labor.)

The nurse is caring for a woman with mitral stenosis who is in the active stage. Which action should the nurse take to promote cardiac function? A. Maintain the woman in a side-lying position with the head and shoulders elevated to facilitate hemodynamics B. Prepare the woman for delivery by cesarean section since this is the recommended delivery method to sustain hemodynamics C. Encourage the woman to avoid the use of narcotics or epidural regional analgesia since this alters cardiac function D. Promote the use of the Valsalva maneuver during pushing in the second stage to improve diastolic ventricular filling

A. Maintain the woman in a side-lying position with the head and shoulders elevated to facilitate hemodynamics The side-lying position with the head and shoulders elevated helps to facilitate hemodynamics during labor. A vaginal delivery is the preferred method of delivery for a woman with cardiac disease as it sustains hemodynamics better than a cesarean section. The use of supportive care, medication, and narcotics or epidural regional analgesia is not contraindicated with a woman with heart disease. The use of the Valsalva maneuver during pushing in the second stage should be avoided because it reduces diastolic ventricular filling and obstructs left ventricular outflow.

A nurse is caring for a woman with mitral stenosis who is in the active stage. Which action should the nurse take to promote cardiac function? A. Maintain the woman in a side-lying position with the head and shoulders elevated to facilitate hemodynamics. B. Prepare the woman for delivery by cesarean section because this is the recommended delivery method to sustain hemodynamics. C. Encourage the woman to avoid the use of narcotics or epidural regional analgesia because this alters cardiac function. D. Promote the use of the Valsalva maneuver during pushing in the second stage to improve diastolic ventricular filling.

A. Maintain the woman in a side-lying position with the head and shoulders elevated to facilitate hemodynamics. The side-lying position with the head and shoulders elevated helps facilitate hemodynamics during labor. A vaginal delivery is the preferred method for a woman with cardiac disease because it sustains hemodynamics better than a cesarean section. The use of supportive care, medication, and narcotics or epidural regional analgesia is not contraindicated with a woman with heart disease. Epidural anesthesia for labor is preferred. (Easterling and Stout, 2012). Using the Valsalva maneuver during pushing in the second stage should be avoided because it reduces diastolic ventricular filling and obstructs left ventricular outflow.

A pregnant woman is being examined by the nurse in the outpatient obstetric clinic. The nurse suspects systemic lupus erythematosus (SLE) after revealing which symptoms? (Select all that apply.) A. Muscle aches B. Hyperactivity C. Weight changes D. Fever E. Hypotension

A. Muscle aches C. Weight changes D. Fever Fatigue, rather than hyperactivity is a common sign of SLE. Hypotension is not a characteristic sign of SLE. Common symptoms, including myalgias, fatigue, weight change, and fevers, occur in nearly all women with SLE at some time during the course of the disease. Although a diagnosis of SLE is suspected based on clinical signs and symptoms, it is confirmed by laboratory testing that demonstrates the presence of circulating autoantibodies. As is the case with other autoimmune diseases, SLE is characterized by a series of exacerbations (flares) and remissions (Chin and Branch, 2012).

16. A woman asks the nurse, "What protects my baby's umbilical cord from being squashed while the baby's inside of me?" The nurse's best response is: A. "Your baby's umbilical cord is surrounded by connective tissue called Wharton jelly, which prevents compression of the blood vessels and ensures continued nourishment of your baby." B. "Your baby's umbilical floats around in blood anyway." C. "You don't need to worry about things like that." D. "The umbilical cord is a group of blood vessels that are very well protected by the placenta."

ANS: A "Your baby's umbilical cord is surrounded by connective tissue called Wharton jelly, which prevents compression of the blood vessels and ensures continued nourishment of your baby" is the most appropriate response. "Your baby's umbilical floats around in blood anyway" is inaccurate. "You don't need to worry about things like that" is an inappropriate response. It negates the patient's need for teaching and discounts her feelings. The placenta does not protect the umbilical cord. The cord is protected by the surrounding Wharton jelly.

33. Identify the goal of a patient with the following nursing diagnosis: Imbalanced Nutrition: Less Than Body Requirements related to diet choices inadequate to meet nutrient requirements of pregnancy. A. Gain a total of 30 lbs. B. Take daily supplements consistently. C. Decrease intake of snack foods. D. Increase intake of complex carbohydrates.

ANS: A A weight gain of 30 lbs is one indication that the patient has gained a sufficient amount for the nutritional needs of pregnancy. A daily supplement is not the best goal for this patient. It does not meet the basic need of proper nutrition during pregnancy. Decreasing snack foods may be a problem and should be assessed; however, assessing weight gain is the best method of monitoring nutritional intake for this pregnant patient. Increasing the intake of complex carbohydrates is important for this patient, but monitoring the weight gain should be the end goal.

33. As the United States and Canada continue to become more culturally diverse, it is increasingly important for the nursing staff to recognize a wide range of varying cultural beliefs and practices. Nurses need to develop respect for these culturally diverse practices and learn to incorporate these into a mutually agreed on plan of care. Although it is common practice in the United States for the father of the baby to be present at the birth, in many societies this is not the case. When implementing care, the nurse would anticipate that a woman from which country would have the father of the baby in attendance? a. Mexico b. China c. Iran d. India

ANS: A A woman from Mexico may be stoic about discomfort until the second stage, at which time she will request pain relief. Fathers and female relatives are usually in attendance during the second stage of labor. The father of the baby is expected to provide encouragement, support, and reassurance that all will be well. Fathers are usually not present in China. The Iranian father will not be present. Female support persons and female care providers are preferred. For many, a male caregiver is unacceptable. The father is usually not present in India, but female relatives are usually present. Natural childbirth methods are preferred.

5. In vitro fertilization-embryo transfer (IVF-ET) is a common approach for women with blocked fallopian tubes or unexplained infertility and for men with very low sperm counts. A husband and wife have arrived for their preprocedural interview. The husband asks the nurse to explain what the procedure entails. The nurse's most appropriate response is: A. "IVF-ET is a type of assisted reproductive therapy that involves collecting eggs from your wife's ovaries, fertilizing them in the laboratory with your sperm, and transferring the embryo to her uterus." B. "A donor embryo will be transferred into your wife's uterus." C. "Donor sperm will be used to inseminate your wife." D. "Don't worry about the technical stuff; that's what we are here for."

ANS: A A woman's eggs are collected from her ovaries, fertilized in the laboratory with sperm, and transferred to her uterus after normal embryonic development has occurred. The statement, "A donor embryo will be transferred into your wife's uterus" describes therapeutic donor insemination. "Donor sperm will be used to inseminate your wife" describes the procedure for a donor embryo. "Don't worry about the technical stuff; that's what we are here for" discredits the patient's need for teaching and is an inappropriate response.

2. Nursing care measures are commonly offered to women in labor. Which nursing measure reflects application of the gate-control theory? a. Massaging the woman's back b. Changing the woman's position c. Giving the prescribed medication d. Encouraging the woman to rest between contractions

ANS: A According to the gate-control theory, pain sensations travel along sensory nerve pathways to the brain, but only a limited number of sensations, or messages, can travel through these nerve pathways at one time. Distraction techniques such as massage or stroking, music, focal points, and imagery reduce or completely block the capacity of nerve pathways to transmit pain. These distractions are thought to work by closing down a hypothetic gate in the spinal cord and thus preventing pain signals from reaching the brain. The perception of pain is thereby diminished. Changing the woman's position, giving prescribed medication, and encouraging rest do not reduce or block the capacity of nerve pathways to transmit pain using the gate-control theory.

6. The nurse providing care for the laboring woman should understand that amnioinfusion is used to treat: a. variable decelerations. b. late decelerations. c. fetal bradycardia. d. fetal tachycardia.

ANS: A Amnioinfusion is used during labor either to dilute meconium-stained amniotic fluid or to supplement the amount of amniotic fluid to reduce the severity of variable decelerations caused by cord compression. Amnioinfusion has no bearing on late decelerations, fetal bradycardia, or fetal tachycardia alterations in fetal heart rate (FHR) tracings.

22. A maternity nurse should be aware of which fact about the amniotic fluid? A. It serves as a source of oral fluid and a repository for waste from the fetus. B. The volume remains about the same throughout the term of a healthy pregnancy. C. A volume of less than 300 mL is associated with gastrointestinal malformations. D. A volume of more than 2 L is associated with fetal renal abnormalities.

ANS: A Amniotic fluid serves as a source of oral fluid, serves as a repository for waste from the fetus, cushions the fetus, and helps maintain a constant body temperature. The volume of amniotic fluid changes constantly. Too little amniotic fluid (oligohydramnios) is associated with renal abnormalities. Too much amniotic fluid (hydramnios) is associated with gastrointestinal and other abnormalities.

11. Nurses should be aware that the biophysical profile (BPP): A. is an accurate indicator of impending fetal death. B. is a compilation of health risk factors of the mother during the later stages of pregnancy. C. consists of a Doppler blood flow analysis and an amniotic fluid index. D. involves an invasive form of ultrasound examination.

ANS: A An abnormal BPP score is an indication that labor should be induced. The BPP evaluates the health of the fetus, requires many different measures, and is a noninvasive procedure.

6. A patient asks her nurse, "My doctor told me that he is concerned with the grade of my placenta because I am overdue. What does that mean?" The best response by the nurse is: A. "Your placenta changes as your pregnancy progresses, and it is given a score that indicates the amount of calcium deposits it has. The more calcium deposits, the higher the grade, or number, that is assigned to the placenta. It also means that less blood and oxygen can be delivered to your baby." B. "Your placenta isn't working properly, and your baby is in danger." C. "This means that we will need to perform an amniocentesis to detect if you have any placental damage." D. "Don't worry about it. Everything is fine."

ANS: A An accurate and appropriate response is, "Your placenta changes as your pregnancy progresses, and it is given a score that indicates the amount of calcium deposits it has. The more calcium deposits, the higher the grade, or number, that is assigned to the placenta. It also means that less blood and oxygen can be delivered to your baby." Although "Your placenta isn't working properly, and your baby is in danger" may be valid, it does not reflect therapeutic communication techniques and is likely to alarm the patient. An ultrasound, not an amniocentesis, is the method of assessment used to determine placental maturation. The response "Don't worry about it. Everything is fine" is not appropriate and discredits the patient's concerns.

14. As relates to dysfunctional uterine bleeding (DUB), the nurse should be aware that: A. it is most commonly caused by anovulation. B. it most often occurs in middle age. C. the diagnosis of DUB should be first considered for abnormal menstrual bleeding. D. the most effective medical treatment is steroids.

ANS: A Anovulation may occur because of hypothalamic dysfunction or polycystic ovary syndrome. DUB most often occurs when the menstrual cycle is being established or when it draws to a close at menopause. A diagnosis of DUB is made only after all other causes of abnormal menstrual bleeding have been ruled out. The most effective medical treatment is oral or intravenous estrogen.

20. With regard to nerve block analgesia and anesthesia, nurses should be aware that: a. most local agents are related chemically to cocaine and end in the suffix -caine. b. local perineal infiltration anesthesia is effective when epinephrine is added, but it can be injected only once. c. a pudendal nerve block is designed to relieve the pain from uterine contractions. d. a pudendal nerve block, if done correctly, does not significantly lessen the bearing-down reflex.

ANS: A Common agents include lidocaine and chloroprocaine. Injections can be repeated to prolong the anesthesia. A pudendal nerve block relieves pain in the vagina, vulva, and perineum but not the pain from uterine contractions, and it lessens or shuts down the bearing-down reflex.

7. A woman is experiencing back labor and complains of intense pain in her lower back. An effective relief measure would be to use: a. counterpressure against the sacrum. b. pant-blow (breaths and puffs) breathing techniques. c. effleurage. d. conscious relaxation or guided imagery.

ANS: A Counterpressure is a steady pressure applied by a support person to the sacral area with the fist or heel of the hand. This technique helps the woman cope with the sensations of internal pressure and pain in the lower back. The pain-management techniques of pant-blow, effleurage, and conscious relaxation or guided imagery are usually helpful for contractions per the gate-control theory.

3. The nurse sees a woman for the first time when she is 30 weeks pregnant. The woman has smoked throughout the pregnancy, and fundal height measurements now are suggestive of growth restriction in the fetus. In addition to ultrasound to measure fetal size, what other tool would be useful in confirming the diagnosis? A. Doppler blood flow analysis B. Contraction stress test (CST) C. Amniocentesis D. Daily fetal movement counts

ANS: A Doppler blood flow analysis allows the examiner to study the blood flow noninvasively in the fetus and the placenta. It is a helpful tool in the management of high risk pregnancies because of intrauterine growth restriction (IUGR), diabetes mellitus, multiple fetuses, or preterm labor. Because of the potential risk of inducing labor and causing fetal distress, CST is not performed on a woman whose fetus is preterm. Indications for amniocentesis include diagnosis of genetic disorders or congenital anomalies, assessment of pulmonary maturity, and diagnosis of fetal hemolytic disease, not IUGR. Fetal kick count monitoring is performed to monitor the fetus in pregnancies complicated by conditions that may affect fetal oxygenation. Although this may be a useful tool at some point later in this woman's pregnancy, it is not used to diagnose IUGR.

8. The nurse expects which maternal cardiovascular finding during labor? a. Increased cardiac output b. Decreased pulse rate c. Decreased white blood cell (WBC) count d. Decreased blood pressure

ANS: A During each contraction, 400 ml of blood is emptied from the uterus into the maternal vascular system, which increases cardiac output by approximately 10% to 15% during the first stage of labor and by approximately 30% to 50% in the second stage of labor. The heart rate increases slightly during labor. The WBC count can increase during labor. During the first stage of labor, uterine contractions cause systolic readings to increase by approximately 10 mm Hg. During the second stage, contractions may cause systolic pressures to increase by 30 mm Hg and diastolic readings to increase by 25 mm Hg.

26. Which deceleration of the fetal heart rate would not require the nurse to change the maternal position? a. Early decelerations b. Late decelerations c. Variable decelerations d. It is always a good idea to change the woman's position.

ANS: A Early decelerations (and accelerations) generally do not need any nursing intervention. Late decelerations suggest that the nurse should change the maternal position (lateral); variable decelerations also require a maternal position change (side to side). Although changing positions throughout labor is recommended, it is not required in response to early decelerations.

2. The nurse caring for the laboring woman should understand that early decelerations are caused by: a. altered fetal cerebral blood flow. b. umbilical cord compression. c. uteroplacental insufficiency. d. spontaneous rupture of membranes.

ANS: A Early decelerations are the fetus's response to fetal head compression. Variable decelerations are associated with umbilical cord compression. Late decelerations are associated with uteroplacental insufficiency. Spontaneous rupture of membranes has no bearing on the fetal heart rate unless the umbilical cord prolapses, which would result in variable or prolonged bradycardia.

17. Which nutritional recommendation about fluids is accurate? A. A woman's daily intake should be 8 to 10 glasses (2.3 L) of water, milk, or juice. B. Coffee should be limited to no more than two cups, but tea and cocoa can be consumed without worry. C. Of the artificial sweeteners, only aspartame has been not associated with any maternity health concerns. D. Water with fluoride is especially encouraged because it reduces the child's risk of tooth decay.

ANS: A Eight to ten glasses is the standard for fluids; however, they should be the right fluids. All beverages containing caffeine, including tea, cocoa, and some soft drinks should be avoided or drunk only in limited amounts. Artificial sweeteners, including aspartame, have no ill effects on the normal mother or fetus; however, mothers with phenylketonuria should avoid aspartame. No evidence indicates that prenatal fluoride consumption reduces childhood tooth decay.

10. A woman is in her seventh month of pregnancy. She has been reporting nasal congestion and occasional epistaxis. The nurse suspects that: A. this is a normal respiratory change in pregnancy caused by elevated levels of estrogen. B. this is an abnormal cardiovascular change, and the nosebleeds are an ominous sign. C. the woman is a victim of domestic violence and is being hit in the face by her partner. D. the woman has been using cocaine intranasally.

ANS: A Elevated levels of estrogen cause capillaries to become engorged in the respiratory tract. This may result in edema in the nose, larynx, trachea, and bronchi. This congestion may cause nasal stuffiness and epistaxis. Cardiovascular changes in pregnancy may cause edema in lower extremities. Determining that the woman is a victim of domestic violence and was hit in the face cannot be made on the basis of the sparse facts provided. If the woman had been hit in the face, she most likely would have additional physical findings. Determination of the use of cocaine by the woman cannot be made on the basis of the sparse facts provided.

12. Fetal well-being during labor is assessed by: A. the response of the fetal heart rate (FHR) to uterine contractions (UCs). B. maternal pain control. C. accelerations in the FHR. D. an FHR above 110 beats/min.

ANS: A Fetal well-being during labor can be measured by the response of the FHR to UCs. In general, reassuring FHR patterns are characterized by an FHR baseline in the range of 110 to 160 beats/min with no periodic changes, a moderate baseline variability, and accelerations with fetal movement. Maternal pain control is not the measure used to determine fetal well-being in labor. Although FHR accelerations are a reassuring pattern, they are only one component of the criteria by which fetal well-being is assessed. Although an FHR above 110 beats/min may be reassuring, it is only one component of the criteria by which fetal well-being is assessed. More information would be needed to determine fetal well-being.

17. With regard to breathing techniques during labor, maternity nurses should understand that: a. breathing techniques in the first stage of labor are designed to increase the size of the abdominal cavity to reduce friction. b. by the time labor has begun, it is too late for instruction in breathing and relaxation. c. controlled breathing techniques are most difficult near the end of the second stage of labor. d. the patterned-paced breathing technique can help prevent hyperventilation.

ANS: A First-stage techniques promote relaxation of abdominal muscles, thereby increasing the size of the abdominal cavity. Instruction in simple breathing and relaxation techniques early in labor is possible and effective. Controlled breathing techniques are most difficult in the transition phase at the end of the first stage of labor when the cervix is dilated 8 to 10 cm. Patterned-paced breathing sometimes can lead to hyperventilation.

5. A pregnant woman reports that she is still playing tennis at 32 weeks of gestation. The nurse would be most concerned that during and after tennis matches this woman consumes: A. several glasses of fluid. B. extra protein sources such as peanut butter. C. salty foods to replace lost sodium. D. easily digested sources of carbohydrate.

ANS: A If no medical or obstetric problems contraindicate physical activity, pregnant women should get 30 minutes of moderate physical exercise daily. Liberal amounts of fluid should be consumed before, during, and after exercise because dehydration can trigger premature labor. The woman's calorie intake should be sufficient to meet the increased needs of pregnancy and the demands of exercise.

25. Many parents-to-be have questions about multiple births. Maternity nurses should be able to tell them that: A. twinning and other multiple births are increasing because of the use of fertility drugs and delayed childbearing. B. dizygotic twins (two fertilized ova) have the potential to be conjoined twins. C. identical twins are more common in white families. D. fraternal twins are same gender, usually male.

ANS: A If the parents-to-be are older and have taken fertility drugs, they would be very interested to know about twinning and other multiple births. Conjoined twins are monozygotic; they are from a single fertilized ovum in which division occurred very late. Identical twins show no racial or ethnic preference; fraternal twins are more common among African-American women. Fraternal twins can be different genders or the same gender. Identical twins are the same gender.

10. Which presentation is accurately described in terms of both the resenting part and the frequency of occurrence? a. Cephalic: occiput, at least 96% b. Breech: sacrum, 10% to 15% c. Shoulder: scapula, 10% to 15% d. Cephalic: cranial, 80% to 85%

ANS: A In cephalic presentations (head first), the presenting part is the occiput; this presentation occurs in 96% of births. In a breech birth, the sacrum emerges first; this presentation occurs in approximately 3% of births. In shoulder presentations, the scapula emerges first; this presentation occurs in only 1% of births. In a cephalic presentation, the part of the head or cranium that emerges first is the occiput; cephalic presentations occur in 96% of births.

30. Which maternal condition is considered a contraindication for the application of internal monitoring devices? a. Unruptured membranes b. Cervix dilated to 4 cm c. External monitors in current use d. Fetus with a known heart defect

ANS: A In order to apply internal monitoring devices, the membranes must be ruptured. Cervical dilation of 4 cm permits the insertion of fetal scalp electrodes and intrauterine catheter. The external monitor can be discontinued after the internal ones are applied. A compromised fetus should be monitored with the most accurate monitoring devices.

A. 1. In assessing the knowledge of a pregestational woman with type 1 diabetes concerning changing insulin needs during pregnancy, the nurse recognizes that further teaching is warranted when the patient states: A. "I will need to increase my insulin dosage during the first 3 months of pregnancy." B. "Insulin dosage will likely need to be increased during the second and third trimesters." C. "Episodes of hypoglycemia are more likely to occur during the first 3 months." D. "Insulin needs should return to normal within 7 to 10 days after birth if I am bottle-feeding."

ANS: A Insulin needs are reduced in the first trimester because of increased insulin production by the pancreas and increased peripheral sensitivity to insulin. "Insulin dosage will likely need to be increased during the second and third trimesters," "Episodes of hypoglycemia are more likely to occur during the first 3 months," and "Insulin needs should return to normal within 7 to 10 days after birth if I am bottle-feeding" are accurate statements and signify that the woman has understood the teachings regarding control of her diabetes during pregnancy.

1. While evaluating an external monitor tracing of a woman in active labor, the nurse notes that the fetal heart rate (FHR) for five sequential contractions begins to decelerate late in the contraction, with the nadir of the decelerations occurring afte r the peak of the contraction. The nurse's first priority is to: a. change the woman's position. b. notify the care provider. c. assist with amnioinfusion. d. insert a scalp electrode.

ANS: A Late decelerations may be caused by maternal supine hypotension syndrome. They usually are corrected when the woman turns on her side to displace the weight of the gravid uterus from the vena cava. If the fetus does not respond to primary nursing interventions for late decelerations, the nurse would continue with subsequent intrauterine resuscitation measures, including notifying the care provider. An amnioinfusion may be used to relieve pressure on an umbilical cord that has not prolapsed. The FHR pattern associated with this situation most likely reveals variable deceleration. A fetal scalp electrode would provide accurate data for evaluating the well-being of the fetus; however, this is not a nursing intervention that would alleviate late decelerations, nor is it the nurse's first priority.

3. The nurse is assessing the knowledge of new parents with a child born with maple syrup urine disease (MSUD). This is an autosomal recessive inherited disorder, which means that: A. both genes of a pair must be abnormal for the disorder to be expressed. B. only one copy of the abnormal gene is required for the disorder to be expressed. C. the disorder occurs in males and heterozygous females. D. the disorder is carried on the X chromosome.

ANS: A MSUD is a type of autosomal recessive inheritance disorder in which both genes of a pair must be abnormal for the disorder to be expressed. MSUD is not an X-linked dominant or recessive disorder or an autosomal dominant inheritance disorder.

16. Which analysis of maternal serum may predict chromosomal abnormalities in the fetus? A. Multiple-marker screening B. Lecithin/sphingomyelin [L/S] ratio C. Biophysical profile D. Type and crossmatch of maternal and fetal serum

ANS: A Maternal serum can be analyzed for abnormal levels of alpha-fetoprotein, human chorionic gonadotropin, and estriol. The multiple-marker screening may predict chromosomal defects in the fetus. The L/S ratio is used to determine fetal lung maturity. A biophysical profile is used for evaluating fetal status during the antepartum period. Five variables are used, but none is concerned with chromosomal problems. The blood type and crossmatch would not predict chromosomal defects in the fetus.

8. With regard to prenatal genetic testing, nurses should be aware that: A. maternal serum screening can determine whether a pregnant woman is at risk of carrying a fetus with Down syndrome. B. carrier screening tests look for gene mutations of people already showing symptoms of a disease. C. predisposition testing predicts with near certainty that symptoms will appear. D. presymptomatic testing is used to predict the likelihood of breast cancer.

ANS: A Maternal serum screening identifies the risk for the neural tube defect and the specific chromosome abnormality involved in Down syndrome. Carriers of some diseases, such as sickle cell disease, do not display symptoms. Predisposition testing determines susceptibility, such as for breast cancer. presymptomatic testing indicates that symptoms are certain to appear if the gene is present.

8. The nurse caring for a laboring woman is aware that maternal cardiac output can be increased by: a. change in position. b. oxytocin administration. c. regional anesthesia. d. intravenous analgesic.

ANS: A Maternal supine hypotension syndrome is caused by the weight and pressure of the gravid uterus on the ascending vena cava when the woman is in a supine position. This reduces venous return to the woman's heart, as well as cardiac output, and subsequently reduces her blood pressure. The nurse can encourage the woman to change positions and avoid the supine position. Oxytocin administration, regional anesthesia, and intravenous analgesic may reduce maternal cardiac output.

23. With one exception, the safest pregnancy is one in which the woman is drug and alcohol free. For women addicted to opioids, ________________________ treatment is the current standard of care during pregnancy. A. methadone maintenance B. detoxification C. smoking cessation D. 4 Ps Plus

ANS: A Methadone maintenance treatment (MMT) is currently considered the standard of care for pregnant women who are dependent on heroin or other narcotics. Buprenorphine is another medication approved for opioid addiction treatment that is increasingly being used during pregnancy. Opioid replacement therapy has been shown to decrease opioid and other drug use, reduce criminal activity, improve individual functioning, and decrease rates of infections such as hepatitis B and C, HIV, and other sexually transmitted infections. Detoxification is the treatment used for alcohol addiction. Pregnant women requiring withdrawal from alcohol should be admitted for inpatient management. Women are more likely to stop smoking during pregnancy than at any other time in their lives. A smoking cessation program can assist in achieving this goal. The 4 Ps Plus is a screening tool designed specifically to identify pregnant women who need in-depth assessment related to substance abuse.

20. A woman has a thick, white, lumpy, cottage cheese-like discharge, with patches on her labia and in her vagina. She complains of intense pruritus. The nurse practitioner would order which preparation for treatment? a. Miconazole b. Tetracycline c. Clindamycin d. Acyclovir

ANS: A Miconazole and clotrimazole are the drugs of choice to treat candidiasis. Tetracycline is used to treat syphilis. Clindamycin is used to treat bacterial vaginosis. Acyclovir is used to treat genital herpes.

23. On vaginal examination of a 30-year-old woman, the nurse documents the following findings: profuse, thin, grayish white vaginal discharge with a "fishy" odor; complaint of pruritus. On the basis of these findings, the nurse suspects that this woman has: A. bacterial vaginosis (BV). B. candidiasis. C. trichomoniasis. D. gonorrhea.

ANS: A Most women with BV complain of a characteristic "fishy" odor. The discharge usually is profuse; thin; and white, gray, or milky in color. Some women also may have mild irritation or pruritus. The discharge associated with candidiasis is thick, white, and lumpy and resembles cottage cheese. Trichomoniasis may be asymptomatic, but women commonly have a characteristic yellowish-to-greenish, frothy, mucopurulent, copious, and malodorous discharge. Women with gonorrhea are often asymptomatic. They may have a purulent endocervical discharge, but discharge usually is minimal or absent.

15. Perinatal nurses are legally responsible for: a. correctly interpreting fetal heart rate (FHR) patterns, initiating appropriate nursing interventions, and documenting the outcomes. b. greeting the patient on arrival, assessing her, and starting an intravenous line. c. applying the external fetal monitor and notifying the care provider. d. making sure that the woman is comfortable.

ANS: A Nurses who care for women during childbirth are legally responsible for correctly interpreting FHR patterns, initiating appropriate nursing interventions based on those patterns, and documenting the outcomes of those interventions. Greeting the patient, assessing her, and starting an IV; applying the external fetal monitor and notifying the care provider; and making sure the woman is comfortable may be activities that a nurse performs, but they are not activities for which the nurse is legally responsible.

26. The nurse caring for a pregnant patient knows that her health teaching regarding fetal circulation has been effective when the patient reports that she has been sleeping: A. in a side-lying position. B. on her back with a pillow under her knees. C. with the head of the bed elevated. D. on her abdomen.

ANS: A Optimal circulation is achieved when the woman is lying at rest on her side. Decreased uterine circulation may lead to intrauterine growth restriction. Previously it was believed that the left lateral position promoted maternal cardiac output, enhancing blood flow to the fetus. However, it is now known that the side-lying position enhances uteroplacental blood flow. If a woman lies on her back with the pressure of the uterus compressing the vena cava, blood return to the right atrium is diminished. Although having the head of the bed elevated is recommended and ideal for later in pregnancy, the woman still must maintain a lateral tilt to the pelvis to avoid compression of the vena cava. Many women find lying on her abdomen uncomfortable as pregnancy advances. Side-lying is the ideal position to promote blood flow to the fetus.

14. When assessing a multiparous woman who has just given birth to an 8-lb boy, the nurse notes that the woman's fundus is firm and has become globular in shape. A gush of dark red blood comes from her vagina. The nurse concludes that: a. the placenta has separated. b. a cervical tear occurred during the birth. c. the woman is beginning to hemorrhage. d. clots have formed in the upper uterine segment.

ANS: A Placental separation is indicated by a firmly contracting uterus, a change in the uterus from a discoid to a globular ovoid shape, a sudden gush of dark red blood from the introitus, an apparent lengthening of the umbilical cord, and a finding of vaginal fullness. Cervical tears that do not extend to the vagina result in minimal blood loss. Signs of hemorrhage are a boggy uterus, bright red vaginal bleeding, alterations in vital signs, pallor, lightheadedness, restlessness, decreased urinary output, and alter ation in the level of consciousness. If clots have formed in the upper uterine segment, the nurse would expect to find the uterus boggy and displaced to the side.

4. Screening at 24 weeks of gestation reveals that a pregnant woman has gestational diabetes mellitus (GDM). In planning her care, the nurse and the woman mutually agree that an expected outcome is to prevent injury to the fetus as a result of GDM. The nurse identifies that the fetus is at greatest risk for: A. macrosomia. B. congenital anomalies of the central nervous system. C. preterm birth. D. low birth weight.

ANS: A Poor glycemic control later in pregnancy increases the rate of fetal macrosomia. Poor glycemic control during the preconception time frame and into the early weeks of the pregnancy is associated with congenital anomalies. Preterm labor or birth is more likely to occur with severe diabetes and is the greatest risk in women with pregestational diabetes. Increased weight, or macrosomia, is the greatest risk factor for this woman.

28. A new mother asks the nurse about the "white substance" covering her infant. The nurse explains that the purpose of vernix caseosa is to: A. protect the fetal skin from amniotic fluid. B. promote normal peripheral nervous system development. C. allow transport of oxygen and nutrients across the amnion. D. regulate fetal temperature.

ANS: A Prolonged exposure to amniotic fluid during the fetal period could result in breakdown of the skin without the protection of the vernix caseosa. Normal development of the peripheral nervous system is dependent on nutritional intake of the mother. The amnion is the inner membrane that surrounds the fetus. It is not involved in the oxygen and nutrient exchange. The amniotic fluid aids in maintaining fetal temperature.

13. After you complete your nutritional counseling for a pregnant woman, you ask her to repeat your instructions so you can assess her understanding of the instructions given. Which statement indicates that she understands the role of protein in her pregnancy? A. "Protein will help my baby grow." B. "Eating protein will prevent me from becoming anemic." C. "Eating protein will make my baby have strong teeth after he/she is born." D. "Eating protein will prevent me from being diabetic."

ANS: A Protein is the nutritional element basic to growth. An adequate protein intake is essential to meeting the increasing demands of pregnancy. These demands arise from the rapid growth of the fetus; the enlargement of the uterus, mammary glands, and placenta; the increase in the maternal blood volume; and the formation of amniotic fluid. Iron intake prevents anemia. Calcium intake is needed for fetal bone and tooth development. Glycemic control is needed in diabetics; protein is one nutritional factor to consider, but this is not the primary role of protein intake.

33. Despite warnings, prenatal exposure to alcohol continues to exceed by far exposure to illicit drugs. A diagnosis of fetal alcohol syndrome (FAS) is made when there are visible markers in each of three categories. Which category is not associated with the diagnosis of FAS? A. Respiratory conditions B. Impaired growth C. CNS abnormality D. Craniofacial dysmorphologies

ANS: A Respiratory difficulties are not a category of conditions that are related to FAS. Abnormalities related to FAS include organ deformities, genital malformations, and kidney and urinary defects. Impaired growth is a visible marker for FAS. CNS abnormalities with neurologic and intellectual impairments are categories used to assist in the diagnosis of FAS. An infant with FAS manifests at least two craniofacial abnormalities, such as microcephaly, short palpebral fissures, poorly developed philtrum, thin upper lip, or flattening of the maxilla.

15. Nurses should be aware of the differences experience can make in labor pain such as: a. sensory pain for nulliparous women often is greater than for multiparous women during early labor. b. affective pain for nulliparous women usually is less than for multiparous women throughout the first stage of labor. c. women with a history of substance abuse experience more pain during labor. d. multiparous women have more fatigue from labor and therefore experience more pain.

ANS: A Sensory pain is greater for nulliparous women because their reproductive tract structures are less supple. Affective pain is higher for nulliparous women during the first stage but decreases for both nulliparous and multiparous during the second stage. Women with a history of substance abuse experience the same amount of pain as those without such a history. Nulliparous women have longer labors and therefore experience more fatigue.

23. To help a woman reduce the severity of nausea caused by morning sickness, the nurse might suggest that she: A. try a tart food or drink such as lemonade or salty foods such as potato chips. B. drink plenty of fluids early in the day. C. brush her teeth immediately after eating. D. never snack before bedtime.

ANS: A Some women can tolerate tart or salty foods when they are nauseous. The woman should avoid drinking too much when nausea is most likely, but she should make up the fluid levels later in the day when she feels better. The woman should avoid brushing her teeth immediately after eating. A small snack of cereal and milk or yogurt before bedtime may help the stomach in the morning.

4. A woman complains of severe abdominal and pelvic pain around the time of menstruation that has gotten worse over the last 5 years. She also complains of pain during intercourse and has tried unsuccessfully to get pregnant for the past 18 months. These symptoms are most likely related to: A. endometriosis. B. PMS. C. primary dysmenorrhea. D. secondary dysmenorrhea.

ANS: A Symptoms of endometriosis can change over time and may not reflect the extent of the disease. Major symptoms include dysmenorrhea and deep pelvic dyspareunia (painful intercourse). Impaired fertility may result from adhesions caused by endometriosis. Although endometriosis may be associated with secondary dysmenorrhea, it is not a cause of primary dysmenorrhea or PMS. In addition, this woman is complaining of dyspareunia and infertility, which are associated with endometriosis not with PMS or primary or secondary dysmenorrhea.

2. Prenatal testing for human immunodeficiency virus (HIV) is recommended for: A. all women, regardless of risk factors. B. a woman who has had more than one sexual partner. C. a woman who has had a sexually transmitted infection. D. a woman who is monogamous with her partner.

ANS: A Testing for the antibody to HIV is strongly recommended for all pregnant women. A HIV test is recommended for all women, regardless of risk factors. Women who test positive for HIV can be treated, reducing the risk of transmission to the fetus.

29. A means of controlling the birth of the fetal head with a vertex presentation is: a. the Ritgen maneuver. b. fundal pressure. c. the lithotomy position. d. the De Lee apparatus.

ANS: A The Ritgen maneuver extends the head during the actual birth and protects the perineum. Gentle, steady pressure against the fundus of the uterus facilitates vaginal birth. The lithotomy position has been commonly used in Western cultures, partly because it is convenient for the health care provider. The De Lee apparatus is used to suction fluid from the infant's mouth.

34. What important, immediate postoperative care practice should the nurse remember when caring for a woman who has had a mastectomy? A. The blood pressure (BP) cuff should not be applied to the affected arm. B. Venipuncture for blood work should be performed on the affected arm. C. The affected arm should be used for intravenous (IV) therapy. D. The affected arm should be held down close to the woman's side.

ANS: A The affected arm should not be used for BP readings, IV therapy, or venipuncture. The affected arm should be elevated with pillows above the level of the right atrium.

21. When using intermittent auscultation (IA) to assess uterine activity, the nurse should be cognizant that: a. the examiner's hand should be placed over the fundus before, during, and after contractions. b. the frequency and duration of contractions is measured in seconds for consistency. c. contraction intensity is given a judgment number of 1 to 7 by the nurse and patient together. d. the resting tone between contractions is described as either placid or turbulent.

ANS: A The assessment is done by palpation; duration, frequency, intensity, and resting tone must be assessed. The duration of contractions is measured in seconds; the frequency is measured in minutes. The intensity of contractions usually is described as mild, moderate, or strong. The resting tone usually is characterized as soft or relaxed.

9. A woman has chosen the calendar method of conception control. During the assessment process, it is most important that the nurse: A. obtain a history of menstrual cycle lengths for the past 6 months. B. determine the patient's weight gain and loss pattern for the previous year. C. examine skin pigmentation and hair texture for hormonal changes. D. explore the patient's previous experiences with conception control.

ANS: A The calendar method of conception control is based on the number of days in each cycle, counting from the first day of menses. The fertile period is determined after the lengths of menstrual cycles have been accurately recorded for 6 months. Weight gain or loss may be partly related to hormonal fluctuations, but it has no bearing on use of the calendar method. Integumentary changes may be related to hormonal changes, but they are not indicators for use of the calendar method. Exploring previous experiences with conception control may demonstrate patient understanding and compliancy, but it is not the most important aspect to assess for discussion of the calendar method.

13. The most critical nursing action in caring for the newborn immediately after birth is: a. keeping the newborn's airway clear. b. fostering parent-newborn attachment. c. drying the newborn and wrapping the infant in a blanket. d. administering eyedrops and vitamin K.

ANS: A The care given immediately after the birth focuses on assessing and stabilizing the newborn. Although fostering parent-infant attachment is an important task for the nurse, it is not the most critical nursing action in caring for the newborn immediately after birth. The nursing activities would be (in order of importance) to maintain a patent airway, support respiratory effort, and prevent cold stress by drying the newborn and covering the infant with a warmed blanket or placing the newborn under a radiant warmer. After the newborn has been stabilized, the nurse assesses the newborn's physical condition, weighs and measures the newborn, administers prophylactic eye ointment and a vitamin K injection, affixes an identification bracelet, wraps the newborn in warm blankets, and then gives the infant to the partner or mother when he or she is ready.

15. The transition phase during which ovarian function and hormone production decline is called: A. the climacteric. B. menarche. C. menopause. D. puberty.

ANS: A The climacteric is a transitional phase during which ovarian function and hormone production decline. Menarche is the term that denotes the first menstruation. Menopause refers only to the last menstrual period. Puberty is a broad term that denotes the entire transitional stage between childhood and sexual maturity.

7. Which stage of labor varies the most in length? a. First b. Second c. Third d. Fourth

ANS: A The first stage of labor is considered to last from the onset of regular uterine contractions to the full dilation of the cervix. The first stage is significantly longer than the second and third stages combined. In a first-time pregnancy, the first stage of labor can take up to 20 hours. The second stage of labor lasts from the time the cervix is fully dilated to the birth of the fetus. The average length is 20 minutes for a multiparous woman and 50 minutes for a nulliparous woman. The third stage of labor lasts from the birth of the fetus until the placenta is delivered. This stage may be as short as 3 minutes or as long as 1 hour. The fourth stage of labor, recovery, lasts approximately 2 hours after the delivery of the placenta.

10. The nurse must be cognizant that an individual's genetic makeup is known as his or her: a. genotype. b. phenotype. c. karyotype. d. chromotype.

ANS: A The genotype comprises all the genes the individual can pass on to a future generation. The phenotype is the observable expression of an individual's genotype. The karyotype is a pictorial analysis of the number, form, and size of an individual's chromosomes. Genotype refers to an individual's genetic makeup.

9. In evaluating the effectiveness of oxytocin induction, the nurse would expect: a. contractions lasting 80 to 90 seconds, 2 to 3 minutes apart. b. the intensity of contractions to be at least 110 to 130 mm Hg. c. labor to progress at least 2 cm/hr dilation. d. At least 30 mU/min of oxytocin will be needed to achieve cervical dilation.

ANS: A The goal of induction of labor would be to produce contractions that occur every 2 to 3 minutes and last 60 to 90 seconds. The intensity of the contractions should be 80 to 90 mm Hg by intrauterine pressure catheter. Cervical dilation of 1 cm/hr in the active phase of labor would be the goal in an oxytocin induction. The dose is increased by 1 to 2 mU/min at intervals of 30 to 60 minutes until the desired contraction pattern is achieved. Doses are increased up to a maximum of 20 to 40 mU/min.

29. A physician prescribes clomiphene citrate (Clomid, Serophene) for a woman experiencing infertility. She is very concerned about the risk of multiple births. The nurse's most appropriate response is: A. "This is a legitimate concern. Would you like to discuss this further before your treatment begins?" B. "No one has ever had more than triplets with Clomid." C. "Ovulation will be monitored with ultrasound so that this will not happen." D. "Ten percent is a very low risk, so you don't need to worry too much."

ANS: A The incidence of multiple pregnancies with the use of these medications is significantly increased. The patient's concern is legitimate and should be discussed so that she can make an informed decision. Stating that no one has ever had "more than triplets" is inaccurate and negates the patient's concerns. Ultrasound cannot ensure that a multiple pregnancy will not occur. The percentage quoted in this statement is inaccurate. The comment "don't worry" discredits the patient's concern.

28. Immediately after the forceps-assisted birth of an infant, the nurse should: a. assess the infant for signs of trauma. b. give the infant prophylactic antibiotics. c. apply a cold pack to the infant's scalp. d. measure the circumference of the infant's head.

ANS: A The infant should be assessed for bruising or abrasions at the site of application, facial palsy, and subdural hematoma. Prophylactic antibiotics are not necessary with a forceps delivery. A cold pack would put the infant at risk for cold stress and is contraindicated. Measuring the circumference of the head is part of the initial nursing assessment.

21. The exact cause of preterm labor is unknown and believed to be multifactorial. Infection is thought to be a major factor in many preterm labors. Select the type of infection that has not been linked to preterm births. a. Viral b. Periodontal c. Cervical d. Urinary tract

ANS: A The infections that increase the risk of preterm labor and birth are all bacterial. They include cervical, urinary tract, periodontal, and other bacterial infections. Therefore, it is important for the patient to participate in early, continual, and comprehensive prenatal care. Evidence has shown a link between periodontal infections and preterm labor. Researchers recommend regular dental care before and during pregnancy, oral assessment as a routine part of prenatal care, and scrupulous oral hygiene to prevent infection. Cervical infections of a bacterial nature have been linked to preterm labor and birth. The presence of urinary tract infections increases the risk of preterm labor and birth.

21. While teaching the expectant mother about personal hygiene during pregnancy, maternity nurses should be aware that: A. tub bathing is permitted even in late pregnancy unless membranes have ruptured. B. the perineum should be wiped from back to front. C. bubble bath and bath oils are permissible because they add an extra soothing and cleansing action to the bath. D. expectant mothers should use specially treated soap to cleanse the nipples.

ANS: A The main danger from taking baths is falling in the tub. The perineum should be wiped from front to back. Bubble baths and bath oils should be avoided because they may irritate the urethra. Soap, alcohol, ointments, and tinctures should not be used to cleanse the nipples because they remove protective oils. Warm water is sufficient.

5. The multiple marker test is used to assess the fetus for which condition? A. Down syndrome B. Diaphragmatic hernia C. Congenital cardiac abnormality D. Anencephaly

ANS: A The maternal serum level of alpha-fetoprotein is used to screen for Down syndrome, neural tube defects, and other chromosome anomalies. The multiple marker test would not detect diaphragmatic hernia, congenital cardiac abnormality, or anencephaly. Additional testing, such as ultrasonography and amniocentesis, would be required to diagnose these conditions.

18. A woman is 16 weeks pregnant and has elected to terminate her pregnancy. The nurse knows that the most common technique used for medical termination of a pregnancy in the second trimester is: A. dilation and evacuation (D&E). B. instillation of hypertonic saline into the uterine cavity. C. intravenous administration of Pitocin. D. vacuum aspiration.

ANS: A The most common technique for medical termination of a pregnancy in the second trimester is D&E. It is usually performed between 13 and 16 weeks. Hypertonic solutions injected directly into the uterus account for less than 1% of all abortions because other methods are safer and easier to use. Intravenous administration of Pitocin is used to induce labor in a woman with a third-trimester fetal demise. Vacuum aspiration is used for abortions in the first trimester.

24. Which major neonatal complication is carefully monitored after the birth of the infant of a diabetic mother? a. Hypoglycemia b. Hypercalcemiac. c. Hypobilirubinemia d. Hypoinsulinemia

ANS: A The neonate is at highest risk for hypoglycemia because fetal insulin production is accelerated during pregnancy to metabolize excessive glucose from the mother. At birth, the maternal glucose supply stops and the neonatal insulin exceeds the available glucose, thus leading to hypoglycemia. Hypocalcemia is associated with preterm birth, birth trauma, and asphyxia, all common problems of the infant of a diabetic mother. Excess erythrocytes are broken down after birth and release large amounts of bilirubin into the neonate's circulation, with resulting hyperbilirubinemia. Because fetal insulin production is accelerated during pregnancy, the neonate presents with hyperinsulinemia.

15. A pregnant woman's biophysical profile score is 8. She asks the nurse to explain the results. The nurse's best response is: A. "The test results are within normal limits." B. "Immediate delivery by cesarean birth is being considered." C. "Further testing will be performed to determine the meaning of this score." D. "An obstetric specialist will evaluate the results of this profile and, within the next week, will inform you of your options regarding delivery."

ANS: A The normal biophysical score ranges from 8 to 10 points if the amniotic fluid volume is adequate. A normal score allows conservative treatment of high risk patients. Delivery can be delayed if fetal well-being is indicated. Scores less than 4 should be investigated, and delivery could be initiated sooner than planned. This score is within normal range, and no further testing is required at this time. The results of the biophysical profile are usually available immediately after the procedure is performed.

19. The mucous plug that forms in the endocervical canal is called the: A. operculum. B. leukorrhea. C. funic souffle. D. ballottement.

ANS: A The operculum protects against bacterial invasion. Leukorrhea is the mucus that forms the endocervical plug (the operculum). The funic souffle is the sound of blood flowing through the umbilical vessels. Ballottement is a technique for palpating the fetus.

5. Because of the effect of cyclic ovarian changes on the breast, the best time for breast self-examination (BSE) is: A. 5 to 7 days after menses ceases. B. Day 1 of the endometrial cycle. C. mid-menstrual cycle. D. any time during a shower or bath.

ANS: A The physiologic alterations in breast size and activity reach their minimal level about 5 to 7 days after menstruation stops. All women should perform BSE during this phase of the menstrual cycle.

18. What is the nurse's understanding of the appropriate role of primary and secondary powers? a. Primary powers are responsible for the effacement and dilation of the cervix. b. Effacement is generally well ahead of dilation in women giving birth for the first time; they are closer together in subsequent pregnancies. c. Scarring of the cervix caused by a previous infection or surgery may make the delivery a bit more painful, but it should not slow or inhibit dilation. d. Pushing in the second stage of labor is more effective if the woman can breathe deeply and control some of her involuntary needs to push, as the nurse directs.

ANS: A The primary powers are responsible for dilation and effacement; secondary powers are concerned with expulsion of the fetus. Effacement is generally well ahead of dilation in first-time pregnancies; they are closer together in subsequent pregnancies. Scarring of the cervix may slow dilation. Pushing is more effective and less fatiguing when the woman begins to push only after she has the urge to do so.

31. The nurse knows that proper placement of the tocotransducer for electronic fetal monitoring is located: a. over the uterine fundus. b. on the fetal scalp. c. inside the uterus. d. over the mother's lower abdomen.

ANS: A The tocotransducer monitors uterine activity and should be placed over the fundus, where the most intensive uterine contractions occur. The tocotransducer is for external use.

2. The uterus is a muscular, pear-shaped organ that is responsible for: A. cyclic menstruation. B. sex hormone production. C. fertilization. D. sexual arousal.

ANS: A The uterus is an organ for reception, implantation, retention, and nutrition of the fertilized ovum; it also is responsible for cyclic menstruation. Hormone production and fertilization occur in the ovaries. Sexual arousal is a feedback mechanism involving the hypothalamus, the pituitary gland, and the ovaries.

17. The female reproductive organ(s) responsible for cyclic menstruation is/are the: A. uterus. B. ovaries. C. vaginal vestibule. D. urethra.

ANS: A The uterus is responsible for cyclic menstruation. It also houses and nourishes the fertilized ovum and the fetus. The ovaries are responsible for ovulation and production of estrogen; the uterus is responsible for cyclic menstruation. The vaginal vestibule is an external organ that has openings to the urethra and vagina; the uterus is responsible for cyclic menstruation. The urethra is not a reproductive organ, although it is found in the area.

6. When assessing a woman in the first stage of labor, the nurse recognizes that the most conclusive sign that uterine contractions are effective would be: a. dilation of the cervix. b. descent of the fetus. c. rupture of the amniotic membranes. d. increase in bloody show.

ANS: A The vaginal examination reveals whether the woman is in true labor. Cervical change, especially dilation, in the presence of adequate labor indicates that the woman is in true labor. Descent of the fetus, or engagement, may occur before labor. Rupture of membranes may occur with or without the presence of labor. Bloody show may indicate slow, progressive cervical change (e.g., effacement) in both true and false labor.

10. A pregnant woman's amniotic membranes rupture. Prolapsed umbilical cord is suspected. What intervention would be the top priority? a. Placing the woman in the knee-chest position. b. Covering the cord in sterile gauze soaked in saline. c. Preparing the woman for a cesarean birth. d. Starting oxygen by face mask.

ANS: A The woman is assisted into a position (e.g., modified Sims position, Trendelenburg position, or the knee-chest position) in which gravity keeps the pressure of the presenting part off the cord. Although covering the cord in sterile gauze soaked saline, preparing the woman for a cesarean, and starting oxygen by face mark are appropriate nursing interventions in the event of a prolapsed cord, the intervention of top priority would be positioning the mother to relieve cord compression.

29. In assisting with the two factors that have an effect on fetal status (i.e., pushing and positioning), nurses should: a. encourage the woman's cooperation in avoiding the supine position. b. advise the woman to avoid the semi-Fowler position. c. encourage the woman to hold her breath and tighten her abdominal muscles to produce a vaginal response. d. instruct the woman to open her mouth and close her glottis, letting air escape after the push.

ANS: A The woman should maintain a side-lying position. The semi-Fowler position is the recommended side-lying position with a lateral tilt to the uterus. The Valsalva maneuver, which encourages the woman to hold her breath and tighten her abdominal muscles, should be avoided. Both the mouth and glottis should be open, letting air escape during the push.

38. A laboring woman is lying in the supine position. The most appropriate nursing action at this time is to: a. ask her to turn to one side. b. elevate her feet and legs. c. take her blood pressure. d. determine whether fetal tachycardia is present.

ANS: A The woman's supine position may cause the heavy uterus to compress her inferior vena cava, thus reducing blood return to her heart and reducing placental blood flow. Elevating her legs will not relieve the pressure from the inferior vena cava. If the woman is allowed to stay in the supine position and blood flow to the placental is reduced significantly, fetal tachycardia may occur. The most appropriate nursing action is to prevent this from occurring by turning the woman to her side. Blood pressure readings may be obtained when the patient is in the appropriate and safest position.

11. To help patients manage discomfort and pain during labor, nurses should be aware that: a. the predominant pain of the first stage of labor is the visceral pain located in the lower portion of the abdomen. b. referred pain is the extreme discomfort between contractions. c. the somatic pain of the second stage of labor is more generalized and related to fatigue. d. pain during the third stage is a somewhat milder version of the second stage.

ANS: A This pain comes from cervical changes, distention of the lower uterine segment, and uterine ischemia. Referred pain occurs when the pain that originates in the uterus radiates to the abdominal wall, lumbosacral area of the back, iliac crests, and gluteal area. Second-stage labor pain is intense, sharp, burning, and localized. Third-stage labor pain is similar to that of the first stage.

17. A pregnant woman is in her third trimester. She asks the nurse to explain how she can tell true labor from false labor. The nurse would explain that "true" labor contractions: a. increase with activity such as ambulation. b. decrease with activity. c. are always accompanied by the rupture of the bag of waters. d. alternate between a regular and an irregular pattern.

ANS: A True labor contractions become more intense with walking. False labor contractions often stop with walking or position changes. Rupture of membranes may occur before or during labor. True labor contractions are regular.

11. When managing the care of a woman in the second stage of labor, the nurse uses various measures to enhance the progress of fetal descent. These measures include: a. encouraging the woman to try various upright positions, including squatting and standing. b. telling the woman to start pushing as soon as her cervix is fully dilated. c. continuing an epidural anesthetic so pain is reduced and the woman can relax. d. coaching the woman to use sustained, 10- to 15-second, closed-glottis bearing-down efforts with each contraction.

ANS: A Upright positions and squatting both may enhance the progress of fetal descent. Many factors dictate when a woman will begin pushing. Complete cervical dilation is necessary, but it is only one factor. If the fetal head is still in a higher pelvic station, the physician or midwife may allow the woman to "labor down" (allowing more time for fetal descent, thereby reducing the amount of pushing needed) if the woman is able. The epidural may mask the sensations and muscle control needed for the woman to push effectively. Closed glottic breathing may trigger the Valsalva maneuver, which increases intrathoracic and cardiovascular pressures, reducing cardiac output and inhibiting perfusion of the uterus and placenta. In addition, holding the breath for longer than 5 to 7 seconds diminishes the perfusion of oxygen across the placenta and results in fetal hypoxia.

26. A first-time mother at 18 weeks of gestation comes for her regularly scheduled prenatal visit. The patient tells the nurse that she is afraid that she is going into premature labor because she is beginning to have regular contractions. The nurse explains that this is the Braxton Hicks sign and teaches the patient that this type of contraction: A. is painless. B. increases with walking. C. causes cervical dilation. D. impedes oxygen flow to the fetus.

ANS: A Uterine contractions can be felt through the abdominal wall soon after the fourth month of gestation. Braxton Hicks contractions are regular and painless and continue throughout the pregnancy. Although they are not painful, some women complain that they are annoying. Braxton Hicks contractions usually cease with walking or exercise. They can be mistaken for true labor; however, they do not increase in intensity or frequency or cause cervical dilation. In addition, they facilitate uterine blood flow through the intervillous spaces of the placenta and promote oxygen delivery to the fetus.

4. When planning care for a laboring woman whose membranes have ruptured, the nurse recognizes that the woman's risk for _________________________ has increased. a. intrauterine infection b. hemorrhage c. precipitous labor d. supine hypotension

ANS: A When the membranes rupture, microorganisms from the vagina can ascend into the amniotic sac and cause chorioamnionitis and placentitis. Rupture of membranes (ROM) is not associated with fetal or maternal bleeding. Although ROM may increase the intensity of contractions and facilitate active labor, it does not result in precipitous labor. ROM has no correlation with supine hypotension.

29. An essential component of counseling women regarding safe sex practices includes discussion regarding avoiding the exchange of body fluids. The physical barrier promoted for the prevention of sexually transmitted infections and human immunodeficiency virus is the condom. Nurses can help motivate patients to use condoms by initiating a discussion related to a number of aspects of condom use. The most important of these is: A. strategies to enhance condom use. B. choice of colors and special features. C. leaving the decision up to the male partner. D. places to carry condoms safely.

ANS: A When the nurse opens discussion on safe sex practices, it gives the woman permission to clear up any concerns or misapprehensions that she may have regarding condom use. The nurse can also suggest ways that the woman can enhance her condom negotiation and communications skills. These include role playing, rehearsal, cultural barriers, and situations that put the patient at risk. Although women can be taught the differences among condoms, such as size ranges, where to purchase, and price, this is not as important as negotiating the use of safe sex practices. Women must address the issue of condom use with every sexual contact. Some men need time to think about this. If they appear reluctant, the woman may want to reconsider the relationship. Although not ideal, women may safely choose to carry condoms in shoes, wallets, or inside their bra. They should be taught to keep the condom away from heat. This information is important; however, it is not germane if the woman cannot even discuss strategies on how to enhance condom use.

25. With regard to a woman's intake and output during labor, nurses should be aware that: a. the tradition of restricting the laboring woman to clear liquids and ice chips is being challenged because regional anesthesia is used more often than general anesthesia. b. intravenous (IV) fluids usually are necessary to ensure that the laboring woman stays hydrated. c. routine use of an enema empties the rectum and is very helpful for producing a clean, clear delivery. d. when a nulliparous woman experiences the urge to defecate, it often means birth will follow quickly.

ANS: A Women are awake with regional anesthesia and are able to protect their own airway, which reduces the worry over aspiration. Routine IV fluids during labor are unlikely to be beneficial and may be harmful. Routine use of an enema is at best ineffective and may be harmful. A multiparous woman may feel the urge to defecate and it may mean birth will follow quickly, but not for a first timer.

16. A woman was treated recently for toxic shock syndrome (TSS). She has intercourse occasionally and uses over-the-counter protection. On the basis of her history, what contraceptive method should she and her partner avoid? a. Cervical cap b. Condom c. Vaginal film d. Vaginal sheath

ANS: A Women with a history of TSS should not use a cervical cap. Condoms, vaginal films, and vaginal sheaths are not contraindicated for a woman with a history of TSS.

1. Congenital anomalies can occur with the use of antiepileptic drugs (AEDs), including: (Select all that apply.) A. cleft lip. B. congenital heart disease. C. neural tube defects. D. gastroschisis. E. diaphragmatic hernia.

ANS: A, B, C Congenital anomalies that can occur with AEDs include cleft lip or palate, congenital heart disease, urogenital defects, and neural tube defects. Gastroschisis and diaphragmatic hernia are not associated with the use of AEDs.

2. Examples of sexual risk behaviors associated with exposure to a sexually transmitted infection (STI) include: (Select all that apply.) A. oral sex. B. unprotected anal intercourse. C. multiple sex partners. D. dry kissing. E. abstinence.

ANS: A, B, C Engaging in these sexual activities increases the exposure risk and the possibility of acquiring an STI. Dry kissing and abstinence are considered "safe" sexual practices.

1. Signs and symptoms that a woman should report immediately to her health care provider include: (Select all that apply.) A. vaginal bleeding. B. rupture of membranes. C. heartburn accompanied by severe headache. D. decreased libido. E. Urinary frequency.

ANS: A, B, C Vaginal bleeding, rupture of membranes, and severe headaches all are signs of potential complications in pregnancy. Patients should be advised to report these signs to the health care provider. Decreased libido and urinary frequency are common discomforts of pregnancy that do not require immediate health care interventions.

1. Most women with uncomplicated pregnancies can use the nurse as their primary source for nutritional information. The nurse or midwife should refer a patient to a registered dietitian for in-depth nutritional counseling in the following situations: (Select all that apply.) A. preexisting or gestational illness such as diabetes. B. ethnic or cultural food patterns. C. obesity. D. vegetarian diet. E. allergy to tree nuts.

ANS: A, B, C, D The nurse should be especially aware that conditions such as diabetes can require in-depth dietary planning and evaluation. To prevent issues with hypoglycemia and hyperglycemia and an increased risk for perinatal morbidity and mortality, this patient would benefit from a referral to a dietitian. Consultation with a dietitian may ensure that cultural food beliefs are congruent with modern knowledge of fetal development and that adjustments can be made to ensure that all nutritional needs are met. The obese pregnant patient may be under the misapprehension that because of her excess weight little or no weight gain is necessary. According to the Institute of Medicine, a patient with a body mass index in the obese range should gain at least 7 kg to ensure a healthy outcome. This patient may require in-depth counseling on optimal food choices. The vegetarian patient needs to have her dietary intake carefully assessed to ensure that the optimal combination of amino acids and protein intake is achieved. Very strict vegetarians (vegans) who consume only plant products may also require vitamin B and mineral supplementation. A patient with a food allergy would not alter that component of her diet during pregnancy; therefore, no additional consultation is necessary.

3. While developing an intrapartum care plan for the patient in early labor, it is important that the nurse recognize that psychosocial factors may influence a woman's experience of pain. These include: (Select all that apply.) a. culture. b. anxiety and fear. c. previous experiences with pain. d. intervention of caregivers. e. support systems.

ANS: A, B, C, E Culture: A woman's sociocultural roots influence how she perceives, interprets, and responds to pain during childbirth. Some cultures encourage loud and vigorous expressions of pain, whereas others value self-control. The nurse should avoid praising some behaviors (stoicism) while belittling others (noisy expression). Anxiety and fear: Extreme anxiety and fear magnify sensitivity to pain and impair a woman's ability to tolerate it. Anxiety and fear increase muscle tension in the pelvic area, which counters the expulsive forces of uterine contractions and pushing efforts. Previous experiences with pain: Fear and withdrawal are a natural response to pain during labor. Learning about these normal sensations ahead of time helps a woman suppress her natural reactions of fear regarding the impending birth. If a woman previously had a long and difficult labor, she is likely to be anxious. She may also have learned ways to cope and may use these skills to adapt to the present labor experience. Support systems: An anxious partner is less able to provide help and support to a woman during labor. A woman's family and friends can be an important source of support if they convey realistic and positive information about labor and delivery. Although the intervention of caregivers may be necessary for the well-being of the woman and her fetus, some interventions add discomfort to the natural pain of labor (i.e., fetal monitor straps, intravenous lines).

3. At least five factors affect the process of labor and birth. These are easily remembered as the five Ps. Which factors are included in this process? (Select all that apply.) a. Passenger b. Passageway c. Powers d. Pressure e. Psychologic response

ANS: A, B, C, E The five Ps are passenger (fetus and placenta), passageway (birth canal), powers (contractions), position of the mother, and psychologic response. Pressure is not one of the five Ps.

2. Transvaginal ultrasonography is often performed during the first trimester. While preparing your 6-week gestation patient for this procedure, she expresses concerns over the necessity for this test. The nurse should explain that this diagnostic test may be indicated for a number of situations. (Select all that apply) A. Establish gestational age B. Obesity C. Fetal abnormalities D. Amniotic fluid volume E. Ectopic pregnancy

ANS: A, B, C, E Transvaginal ultrasound is useful in obese women whose thick abdominal layers cannot be penetrated with traditional abdominal ultrasound. This procedure is also used for identifying ectopic pregnancy, estimating gestational age, confirming fetal viability, and identifying fetal abnormalities. Amniotic fluid volume is assessed during the second and third trimester. Conventional ultrasound would be used.

2. Which factors influence cervical dilation? (Select all that apply.) a. Strong uterine contractions b. Force of the presenting fetal part against the cervix c. Size of the woman d. Pressure applied by the amniotic sac e. Scarring of the cervix

ANS: A, B, D, E Dilation of the cervix occurs by the drawing upward of the musculofibrous components of the cervix, which is caused by strong uterine contractions. Pressure exerted by the amniotic fluid while the membranes are intact or by the force applied by the presenting part can also promote cervical dilation. Scarring of the cervix as a result of a previous infection or surgery may slow cervical dilation. Pelvic size or the size of the woman does not affect cervical dilation.

1. There is little consensus on the management of premenstrual dysphoric disorder (PMDD). However, nurses can advise women on several self-help modalities that often improve symptoms. The nurse knows that health teaching has been effective when the patient reports that she has adopted a number of lifestyle changes, including: (Select all that apply.) A. regular exercise. B. improved nutrition. C. a daily glass of wine. D. smoking cessation. E. oil of evening primrose.

ANS: A, B, D, E These modalities may provide significant symptom relief in 1 to 2 months. If there is no improvement after these changes have been made, the patient may need to begin pharmacologic therapy. Women should decrease both their alcohol and caffeinated beverage consumption if they have PMDD.

2. Diabetes refers to a group of metabolic diseases characterized by hyperglycemia resulting from defects in insulin action, insulin secretion, or both. Over time, diabetes causes significant changes in the microvascular and macrovascular circulations. These complications include: (Select all that apply.) a. atherosclerosis. b. retinopathy. c. IUFD. d. nephropathy. e. neuropathy.

ANS: A, B, D, E These structural changes are most likely to affect a variety of systems, including the heart, eyes, kidneys, and nerves. Intrauterine fetal death (stillbirth) remains a major complication of diabetes in pregnancy; however, this is a fetal complication.

3. Along with gas exchange and nutrient transfer, the placenta produces many hormones necessary for normal pregnancy. These include: (Select all that apply.) A. human chorionic gonadotropin (hCG). B. insulin. C. estrogen. D. progesterone. E. testosterone.

ANS: A, C, D hCG causes the corpus luteum to persist and produce the necessary estrogens and progesterone for the first 6 to 8 weeks. Estrogens cause enlargement of the woman's uterus and breasts; cause growth of the ductal system in the breasts; and, as term approaches, play a role in the initiation of labor. Progesterone causes the endometrium to change, providing early nourishment. Progesterone also protects against spontaneous abortion by suppressing maternal reactions to fetal antigens and reduces unnecessary uterine contractions. Other hormones produced by the placenta include hCT, hCA, and numerous growth factors. Human placental lactogen promotes normal nutrition and growth of the fetus and maternal breast development for lactation. This hormone decreases maternal insulin sensitivity and utilization of glucose, making more glucose available for fetal growth. If a Y chromosome is present in the male fetus, hCG causes the fetal testes to secrete testosterone necessary for the normal development of male reproductive structures.

2. Induction of labor is considered an acceptable obstetric procedure if it is in the best interest to deliver the fetus. The charge nurse in the labor and delivery unit is often asked to schedule patients for this procedure and therefore must be cognizant of the specific conditions appropriate for labor induction. These include: (Select all that apply.) a. rupture of membranes at or near term. b. convenience of the woman or her physician. c. chorioamnionitis (inflammation of the amniotic sac). d. postterm pregnancy. e. fetal death.

ANS: A, C, D, E These are all acceptable indications for induction. Other conditions include intrauterine growth retardation (IUGR), maternal-fetal blood incompatibility, hypertension, and placental abruption. Elective inductions for the convenience of the woman or her provider are not recommended; however, they have become commonplace. Factors such as rapid labors and living a long distance from a health care facility may be valid reasons in such a circumstance. Elective delivery should not occur before 39 weeks' completed gestation.

2. The class of drugs known as opioid analgesics (butorphanol, nalbuphine) is not suitable for administration to women with known opioid dependence. The antagonistic activity could precipitate withdrawal symptoms (abstinence syndrome) in both mothers and newborns. Signs of opioid/narcotic withdrawal in the mother would include: (Select all that apply.) a. yawning, runny nose. b. increase in appetite. c. chills and hot flashes. d. constipation. e. irritability, restlessness.

ANS: A, C, E The woman experiencing maternal opioid withdrawal syndrome will exhibit yawning, runny nose, sneezing, anorexia, chills or hot flashes, vomiting, diarrhea, abdominal pain, irritability, restlessness, muscle spasms, weakness, and drowsiness. It is important for the nurse to assess both mother and baby and to plan care accordingly.

3. The exact cause of breast cancer remains undetermined. Researchers have found that there are many common risk factors that increase a woman's chance of developing a malignancy. It is essential for the nurse who provides care to women of any age to be aware of which of the following risk factors? (Select all that apply.) A. Family history B. Late menarche C. Early menopause D. Race E. Nulliparity or first pregnancy after age 30

ANS: A, D, E Family history, race, and nulliparity are known risk factors for the development of breast cancer. Other risk factors include age, personal history of cancer, high socioeconomic status, sedentary lifestyle, hormone replacement therapy, recent use of oral contraceptives, never having breastfed a child, and drinking more than one alcoholic beverage per day. Early menarche and late menopause are risk factors for breast malignancy, not late menarche and early menopause.

1. You (the nurse) are reviewing the educational packet provided to a patient about tubal ligation. What is an important fact you should point out? (Select all that apply.) A. "It is highly unlikely that you will become pregnant after the procedure." B. "This is an effective form of 100% permanent sterilization. You won't be able to get pregnant." C. "Sterilization offers some form of protection against sexually transmitted infections (STIs)." D. "Sterilization offers no protection against STIs." E. "Your menstrual cycle will greatly increase after your sterilization."

ANS: A,D A woman is unlikely to become pregnant after tubal ligation, although it is not 100% effective. Sterilization offers no protection against STIs. The menstrual cycle typically remains the same after a tubal ligation.

3. Which symptom described by a patient is characteristic of premenstrual syndrome (PMS)? A. "I feel irritable and moody a week before my period is supposed to start." B. "I have lower abdominal pain beginning the third day of my menstrual period." C. "I have nausea and headaches after my period starts, and they last 2 to 3 days." D. "I have abdominal bloating and breast pain after a couple days of my period."

ANS: APMS is a cluster of physical, psychologic, and behavioral symptoms that begin in the luteal phase of the menstrual cycle and resolve within a couple of days of the onset of menses. Complaints of lower abdominal pain, nausea and headaches, and abdominal bloating all are associated with PMS. However, the timing reflected is inaccurate.

18. A woman who is 39 weeks pregnant expresses fear about her impending labor and how she will manage. The nurse's best response is: a. "Don't worry about it. You'll do fine." b. "It's normal to be anxious about labor. Let's discuss what makes you afraid." c. "Labor is scary to think about, but the actual experience isn't." d. "You can have an epidural. You won't feel anything."

ANS: B "It's normal to be anxious about labor. Let's discuss what makes you afraid" allows the woman to share her concerns with the nurse and is a therapeutic communication tool. "Don't worry about it. You'll do fine" negates the woman's fears and is not therapeutic. "Labor is scary to think about, but the actual experience isn't" negates the woman's fears and offers a false sense of security. It is not true that every woman may have an epidural. A number of criteria must be met for use of an epidural. Furthermore, many women still experience the feeling of pressure with an epidural.

17. A new patient and her partner arrive in the labor, delivery, recovery, and after birth unit for the birth of their first child. You apply the electronic fetal monitor (EFM) to the woman. Her partner asks you to explain what is printing on the graph, referring to the EFM strip. He wants to know what the baby's heart rate should be. Your best response is: a. "Don't worry about that machine; that's my job." b. "The top line graphs the baby's heart rate. Generally the heart rate is between 110 and 160. The heart rate will fluctuate in response to what is happening during labor." c. "The top line graphs the baby's heart rate, and the bottom line lets me know how strong the contractions are." d. "Your doctor will explain all of that later."

ANS: B "The top line graphs the baby's heart rate. Generally the heart rate is between 110 and 160. The heart rate will fluctuate in response to what is happening during labor" educates the partner about fetal monitoring and provides support and information to alleviate his fears. "Don't worry about that machine; that's my job" discredits the partner's feelings and does not provide the teaching he is requesting. "The top line graphs the baby's heart rate, and the bottom line lets me know how strong the contractions are" provides inaccurate information and does not address the partner's concerns about the fetal heart rate. The EFM graphs the frequency and duration of the contractions, not the intensity. Nurses should take every opportunity to provide patient and family teaching, especially when information is requested.

15. What type of cultural concern is the most likely deterrent to many women seeking prenatal care? A. Religion B. Modesty C. Ignorance D. Belief that physicians are evil

ANS: B A concern for modesty is a deterrent to many women seeking prenatal care. For some women, exposing body parts, especially to a man, is considered a major violation of their modesty. Many cultural variations are found in prenatal care. Even if the prenatal care described is familiar to a woman, some practices may conflict with the beliefs and practices of a subculture group to which she belongs.

20. With regard to nutritional needs during lactation, a maternity nurse should be aware that: A. the mother's intake of vitamin C, zinc, and protein now can be lower than during pregnancy. B. caffeine consumed by the mother accumulates in the infant, who may be unusually active and wakeful. C. critical iron and folic acid levels must be maintained. D. lactating women can go back to their prepregnant calorie intake.

ANS: B A lactating woman needs to avoid consuming too much caffeine. Vitamin C, zinc, and protein levels need to be moderately higher during lactation than during pregnancy. The recommendations for iron and folic acid are lower during lactation. Lactating women should consume about 500 kcal more than their prepregnancy intake, at least 1800 kcal daily overall.

25. A maternal indication for the use of forceps is: a. a wide pelvic outlet. b. maternal exhaustion. c. a history of rapid deliveries. d. failure to progress past 0 station.

ANS: B A mother who is exhausted may be unable to assist with the expulsion of the fetus.The patient with a wide pelvic outlet will likely not require vacuum extraction. With a rapid delivery, vacuum extraction is not necessary. A station of 0 is too high for a vacuum extraction.

26. The nerve block used in labor that provides anesthesia to the lower vagina and perineum is called: a. an epidural. b. a pudendal. c. a local. d. a spinal block.

ANS: B A pudendal block anesthetizes the lower vagina and perineum to provide anesthesia for an episiotomy and use of low forceps if needed. An epidural provides anesthesia for the uterus, perineum, and legs. A local provides anesthesia for the perineum at the site of the episiotomy. A spinal block provides anesthesia for the uterus, perineum, and down the legs.

22. Care management of a woman diagnosed with acute pelvic inflammatory disease (PID) most likely would include: A. oral antiviral therapy. B. bed rest in a semi-Fowler position. C. antibiotic regimen continued until symptoms subside. D. frequent pelvic examination to monitor the progress of healing.

ANS: B A woman with acute PID should be on bed rest in a semi-Fowler position. Broad-spectrum antibiotics are used. Antibiotics must be taken as prescribed, even if symptoms subside. Few pelvic examinations should be conducted during the acute phase of the disease.

6. Which statement made by a lactating woman would lead the nurse to believe that the woman might have lactose intolerance? A. "I always have heartburn after I drink milk." B. "If I drink more than a cup of milk, I usually have abdominal cramps and bloating." C. "Drinking milk usually makes me break out in hives." D. "Sometimes I notice that I have bad breath after I drink a cup of milk."

ANS: B Abdominal cramps and bloating are consistent with lactose intolerance. One problem that can interfere with milk consumption is lactose intolerance, which is the inability to digest milk sugar because of a lack of the enzyme lactase in the small intestine. Milk consumption may cause abdominal cramping, bloating, and diarrhea in people who are lactose intolerant, although many affected individuals can tolerate small amounts of milk without symptoms.

31. Concerning the third stage of labor, nurses should be aware that: a. the placenta eventually detaches itself from a flaccid uterus. b. an expectant or active approach to managing this stage of labor reduces the risk of complications. c. it is important that the dark, roughened maternal surface of the placenta appear before the shiny fetal surface. d. the major risk for women during the third stage is a rapid heart rate.

ANS: B Active management facilitates placental separation and expulsion, thus reducing the risk of complications. The placenta cannot detach itself from a flaccid (relaxed) uterus. Which surface of the placenta comes out first is not clinically important. The major risk for women during the third stage of labor is after birth hemorrhage.

36. A woman who is gravida 3 para 2 enters the intrapartum unit. The most important nursing assessments are: a. contraction pattern, amount of discomfort, and pregnancy history. b. fetal heart rate, maternal vital signs, and the woman's nearness to birth. c. identification of ruptured membranes, the woman's gravida and para, and her support person. d. last food intake, when labor began, and cultural practices the couple desires.

ANS: B All options describe relevant intrapartum nursing assessments; however, this focused assessment has priority. If the maternal and fetal conditions are normal and birth is not imminent, other assessments can be performed in an unhurried manner. This includes: gravida, para, support person, pregnancy history, pain assessment, last food intake, and cultural practices.

6. A woman who is 32 weeks' pregnant is informed by the nurse that a danger sign of pregnancy could be: A. constipation. B. alteration in the pattern of fetal movement. C. heart palpitations. D. edema in the ankles and feet at the end of the day.

ANS: B An alteration in the pattern or amount of fetal movement may indicate fetal jeopardy. Constipation, heart palpitations, and ankle and foot edema are normal discomforts of pregnancy that occur in the second and third trimesters.

11. With regard to chromosome abnormalities, nurses should be aware that: A. they occur in approximately 10% of newborns. B. abnormalities of number are the leading cause of pregnancy loss. C. Down syndrome is a result of an abnormal chromosome structure. D. unbalanced translocation results in a mild abnormality that the child will outgrow.

ANS: B Aneuploidy is an abnormality of number that also is the leading genetic cause of mental retardation. Chromosome abnormalities occur in less than 1% of newborns. Down syndrome is the most common form of trisomal abnormality, an abnormality of chromosome number (47 chromosomes). Unbalanced translocation is an abnormality of chromosome structure that often has serious clinical effects.

9. A 31-year-old woman believes that she may be pregnant. She took an OTC pregnancy test 1 week ago after missing her period; the test was positive. During her assessment interview, the nurse enquires about the woman's last menstrual period and asks whether she is taking any medications. The woman states that she takes medicine for epilepsy. She has been under considerable stress lately at work and has not been sleeping well. She also has a history of irregular periods. Her physical examination does not indicate that she is pregnant. She has an ultrasound scan that reveals she is not pregnant. What is the most likely cause of the false-positive pregnancy test result? A. She took the pregnancy test too early. B. She takes anticonvulsants. C. She has a fibroid tumor. D. She has been under considerable stress and has a hormone imbalance.

ANS: B Anticonvulsants may cause false-positive pregnancy test results. OTC pregnancy tests use enzyme-linked immunosorbent assay technology, which can yield positive results 4 days after implantation. Implantation occurs 6 to 10 days after conception. If the woman were pregnant, she would be into her third week at this point (having missed her period 1 week ago). Fibroid tumors do not produce hormones and have no bearing on hCG pregnancy tests. Although stress may interrupt normal hormone cycles (menstrual cycles), it does not affect human chorionic gonadotropin levels or produce positive pregnancy test results.

14. With regard to a pregnant woman's anxiety and pain experience, nurses should be aware that: a. even mild anxiety must be treated. b. severe anxiety increases tension, which increases pain, which in turn increases fear and anxiety, and so on. c. anxiety may increase the perception of pain, but it does not affect the mechanism of labor. d. women who have had a painful labor will have learned from the experience and have less anxiety the second time because of increased familiarity.

ANS: B Anxiety and pain reinforce each other in a negative cycle. Mild anxiety is normal for a woman in labor and likely needs no special treatment other than the standard reassurances. Anxiety increases muscle tension and ultimately can build sufficiently to slow the progress of labor. Unfortunately, an anxious, painful first labor is likely to carry over, through expectations and memories, into an anxious and painful experience in the second pregnancy.

24. Three servings of milk, yogurt, or cheese plus two servings of meat, poultry, or fish adequately supply the recommended amount of protein for a pregnant woman. Many patients are concerned about the increased levels of mercury in fish and may be afraid to include this source of nutrients in their diet. Sound advice by the nurse to assist the patient in determining which fish is safe to consume would include: A. white tuna is a preferred choice. B. avoid shark, swordfish, and mackerel. C. fish caught in local waterways are the safest. D. salmon and shrimp contain high levels of mercury.

ANS: B As a precaution, the pregnant patient should avoid eating all of these and the less common tilefish. High levels of mercury can harm the developing nervous system of the fetus. It is essential for the nurse to assist the patient in understanding the differences between numerous sources of this product. A pregnant patient can take 12 ounces a week of canned light tuna; however, canned white, albacore, or tuna steaks contain higher levels of mercury and should be limited to no more than 6 ounces per week. It is a common misconception that fish caught in local waterways are the safest. Pregnant women and mothers of young children should check with local advisories about the safety of fish caught by families and friends in nearby bodies of water. If no information is available, these fish sources should be avoided, limited to less than 6 ounces, or the only fish consumed that week. Commercially caught fish that are low in mercury include salmon, shrimp, pollock, or catfish.

23. After change-of-shift report the nurse assumes care of a multiparous patient in labor. The woman is complaining of pain that radiates to her abdominal wall, lower back, and buttocks and down her thighs. Before implementing a plan of care, the nurse should understand that this type of pain is: a. visceral. b. referred. c. somatic. d. afterpain.

ANS: B As labor progresses the woman often experiences referred pain. This occurs when pain that originates in the uterus radiates to the abdominal wall, the lumbosacral area of the back, the gluteal area, and thighs. The woman usually has pain only during a contraction and is free from pain between contractions. Visceral pain is that which predominates in the first stage of labor. This pain originates from cervical changes, distention of the lower uterine segment, and uterine ischemia. Visceral pain is located over the lower portion of the abdomen. Somatic pain is described as intense, sharp, burning, and well localized. This results from stretching of the perineal tissues and the pelvic floor. This occurs during the second stage of labor. Pain experienced during the third stage of labor or afterward during the early after birth period is uterine. This pain is very similar to that experienced in the first stage of labor. P

21. To reassure and educate pregnant patients about changes in their cardiovascular system, maternity nurses should be aware that: A. a pregnant woman experiencing disturbed cardiac rhythm, such as sinus arrhythmia requires close medical and obstetric observation, no matter how healthy she otherwise may appear. B. changes in heart size and position and increases in blood volume create auditory changes from 20 weeks to term. C. palpitations are twice as likely to occur in twin gestations. D. all of the above changes will likely occur.

ANS: B Auscultatory changes should be discernible after 20 weeks of gestation. A healthy woman with no underlying heart disease does not need any therapy. The maternal heart rate increases in the third trimester, but palpitations may not occur. Auditory changes are discernible at 20 weeks.

19. While caring for the patient who requires an induction of labor, the nurse should be cognizant that: a. ripening the cervix usually results in a decreased success rate for induction. b. labor sometimes can be induced with balloon catheters or laminaria tents. c. oxytocin is less expensive than prostaglandins and more effective but creates greater health risks. d. amniotomy can be used to make the cervix more favorable for labor.

ANS: B Balloon catheters or laminaria tents are mechanical means of ripening the cervix. Ripening the cervix, making it softer and thinner, increases the success rate of induced labor. Prostaglandin E1 is less expensive and more effective than oxytocin but carries a greater risk. Amniotomy is the artificial rupture of membranes, which is used to induce labor only when the cervix is already ripe.

23. Which contraceptive method best protects against sexually transmitted infections (STIs) and human immunodeficiency virus (HIV)? A. Periodic abstinence B. Barrier methods C. Hormonal methods D. They all offer about the same protection.

ANS: B Barrier methods such as condoms best protect against STIs and HIV. Periodic abstinence and hormonal methods (the pill) offer no protection against STIs or HIV.

24. It is important for the nurse to develop a realistic birth plan with the pregnant woman in her care. The nurse can explain that a major advantage of nonpharmacologic pain management is: a. greater and more complete pain relief is possible. b. no side effects or risks to the fetus are involved. c. the woman remains fully alert at all times. d. a more rapid labor is likely.

ANS: B Because nonpharmacologic pain management does not include analgesics, adjunct drugs, or anesthesia, it is harmless to the mother and the fetus. There is less pain relief with nonpharmacologic pain management during childbirth. The woman's alertness is not altered by medication; however, the increase in pain will decrease alertness. Pain management may or may not alter the length of

13. A woman who has a seizure disorder and takes barbiturates and phenytoin sodium daily asks the nurse about the pill as a contraceptive choice. The nurse's most appropriate response would be: A. "This is a highly effective method, but it has some side effects." B. "Your current medications will reduce the effectiveness of the pill." C. "The pill will reduce the effectiveness of your seizure medication." D. "This is a good choice for a woman of your age and personal history."

ANS: B Because the liver metabolizes oral contraceptives, their effectiveness is reduced when they are taken simultaneously with anticonvulsants. The statement "Your current medications will reduce the effectiveness of the pill" is true, but it is not the most appropriate response. The anticonvulsant will reduce the effectiveness of the pill, not the other way around. The statement "This is a good choice for a woman of your age and personal history" does not teach the patient that the effectiveness of the pill may be reduced because of her anticonvulsant therapy.

14. While providing care in an obstetric setting, the nurse should understand that after birth care of the woman with cardiac disease: A. is the same as that for any pregnant woman. B. includes rest, stool softeners, and monitoring of the effect of activity. C. includes ambulating frequently, alternating with active range of motion. D. includes limiting visits with the infant to once per day.

ANS: B Bed rest may be ordered, with or without bathroom privileges. Bowel movements without stress or strain for the woman are promoted with stool softeners, diet, and fluid. Care of the woman with cardiac disease in the after birth period is tailored to the woman's functional capacity. The woman will be on bed rest to conserve energy and reduce the strain on the heart. Although the woman may need help caring for the infant, breastfeeding and infant visits are not contraindicated.

13. Nurses should know some basic definitions concerning preterm birth, preterm labor, and low birth weight. For instance: a. the terms preterm birth and low birth weight can be used interchangeably. b. preterm labor is defined as cervical changes and uterine contractions occurring between 20 and 37 weeks of pregnancy. c. low birth weight is anything below 3.7 lbs. d. in the United States early in this century, preterm birth accounted for 18% to 20% of all births.

ANS: B Before 20 weeks, it is not viable (miscarriage); after 37 weeks, it can be considered term. Although these terms are used interchangeably, they have different meanings: preterm birth describes the length of gestation (37 weeks) regardless of weight; low birth weight describes weight only (2500 g or less) at the time of birth, whenever it occurs. Low birth weight is anything less than 2500 g, or about 5.5 lbs. In 2003 the preterm birth rate in the United States was 12.3%, but it is increasing in frequency.

6. Cardiovascular system changes occur during pregnancy. Which finding would be considered normal for a woman in her second trimester? A. Less audible heart sounds (S1, S2) B. Increased pulse rate C. Increased blood pressure D. Decreased red blood cell (RBC) production

ANS: B Between 14 and 20 weeks of gestation, the pulse increases about 10 to 15 beats/min, which persists to term. Splitting of S1 and S2 is more audible. In the first trimester, blood pressure usually remains the same as at the prepregnancy level, but it gradually decreases up to about 20 weeks of gestation. During the second trimester, both the systolic and the diastolic pressures decrease by about 5 to 10 mm Hg. Production of RBCs accelerates during pregnancy.

12. Women with an inadequate weight gain during pregnancy are at higher risk of giving birth to an infant with: A. spina bifida. B. intrauterine growth restriction. C. diabetes mellitus. D. Down syndrome.

ANS: B Both normal-weight and underweight women with inadequate weight gain have an increased risk of giving birth to an infant with intrauterine growth restriction. Spina bifida, diabetes mellitus, and Down syndrome are not associated with inadequate maternal weight gain.

13. A 3-year-old girl's mother is 6 months pregnant. What concern is this child likely to verbalize? A. How the baby will "get out"? B. What the baby will eat? C. Whether her mother will die? D. What color eyes the baby has?

ANS: B By age 3 or 4, children like to be told the story of their own beginning and accept its comparison with the present pregnancy. They like to listen to the fetal heartbeat and feel the baby move. Sometimes they worry about how the baby is being fed and what it wears. School-age children take a more clinical interest in their mother's pregnancy and may want to know, "How did the baby get in there?" and "How will it get out?" Whether her mother will die does not tend to be the focus of a child's questions about the impending birth of a sibling. The baby's eye color does not tend to be the focus of children's questions about the impending birth of a sibling.

6. Maternal phenylketonuria (PKU) is an important health concern during pregnancy because: A. it is a recognized cause of preterm labor. B. the fetus may develop neurologic problems. C. a pregnant woman is more likely to die without dietary control. D. women with PKU are usually retarded and should not reproduce.

ANS: B Children born to women with untreated PKU are more likely to be born with mental retardation, microcephaly, congenital heart disease, and low birth weight. Maternal PKU has no effect on labor. Women without dietary control of PKU are more likely to miscarry or bear a child with congenital anomalies. Screening for undiagnosed maternal PKU at the first prenatal visit may be warranted, especially in individuals with a family history of the disorder, with low intelligence of uncertain etiology, or who have given birth to microcephalic infants.

20. Nurses caring for antepartum women with cardiac conditions should be aware that: A. stress on the heart is greatest in the first trimester and the last 2 weeks before labor. B. women with Class II cardiac disease should avoid heavy exertion and any activity that causes even minor symptoms. C. women with Class III cardiac disease should have 8 to 10 hours of sleep every day and limit housework, shopping, and exercise. D. Women with Class I cardiac disease need bed rest through most of the pregnancy and face the possibility of hospitalization near term.

ANS: B Class II cardiac disease is symptomatic with ordinary activity. Women in this category need to avoid heavy exertion and limit regular activities as symptoms dictate. Stress is greatest between weeks 28 and 32, when homodynamic changes reach their maximum. Class III cardiac disease is symptomatic with less than ordinary activity. These women need bed rest most of the day and face the possibility of hospitalization near term. Class I cardiac disease is asymptomatic at normal levels of activity. These women can carry on limited normal activities with discretion, although they still need a good amount of sleep.

28. Which of the following statements about the various forms of hepatitis is accurate? A. A vaccine exists for hepatitis C but not for hepatitis B. B. Hepatitis A is acquired by eating contaminated food or drinking polluted water. C. Hepatitis B is less contagious than human immunodeficiency virus (HIV). D. The incidence of hepatitis C is decreasing.

ANS: B Contaminated milk and shellfish are common sources of infection with hepatitis A. A vaccine exists for hepatitis B but not for hepatitis C. Hepatitis B is more contagious than HIV. The incidence of hepatitis C is increasing.

4. In presenting to obstetric nurses interested in genetics, the genetic nurse identifies the primary risk(s) associated with genetic testing as: A. anxiety and altered family relationships. B. denial of insurance benefits. C. high false-positive results associated with genetic testing. D. ethnic and socioeconomic disparity associated with genetic testing.

ANS: B Decisions about genetic testing are shaped by socioeconomic status and the ability to pay for the testing. Some types of genetic testing are expensive and are not covered by insurance benefits. Anxiety and altered family relationships, high false-positive results, and ethnic and socioeconomic disparity are factors that may be difficulties associated with genetic testing, but they are not risks associated with testing.

17. An unmarried young woman describes her sex life as "active" and involving "many" partners. She wants a contraceptive method that is reliable and does not interfere with sex. She requests an intrauterine device (IUD). The nurse's most appropriate response is: A. "The IUD does not interfere with sex." B. "The risk of pelvic inflammatory disease (PID) will be higher for you." C. "The IUD will protect you from sexually transmitted infections (STIs)." D. "Pregnancy rates are high with IUDs."

ANS: B Disadvantages of IUDs include an increased risk of PID in the first 20 days after insertion and the risks of bacterial vaginosis and uterine perforation. The IUD offers no protection against STIs or human immunodeficiency virus. Because this woman has multiple sex partners, she is at higher risk of developing a STI. The IUD does not protect against infection, as does a barrier method. Although the statement "The IUD does not interfere with sex" may be correct, it is not the most appropriate response. The IUD offers no protection from STIs. The typical failure rate of the IUD in the first year of use is 0.8%.

13. During a health history interview, a woman states that she thinks that she has "bumps" on her labia. She also states that she is not sure how to check herself. The correct response would be to: A. reassure the woman that the examination will not reveal any problems. B. explain the process of vulvar self-examination to the woman and reassure her that she should become familiar with normal and abnormal findings during the examination. C. reassure the woman that "bumps" can be treated. D. reassure her that most women have "bumps" on their labia.

ANS: B During the assessment and evaluation, the responsibility for self-care, health promotion, and enhancement of wellness is emphasized. The pelvic examination provides a good opportunity for the practitioner to emphasize the need for regular vulvar self-examination. Providing reassurance to the woman concerning the "bumps" would not be an accurate response.

10. In the first trimester, ultrasonography can be used to gain information on: A. amniotic fluid volume. B. location of gestational sacs. C. placental location and maturity. D. cervical length.

ANS: B During the first trimester, ultrasound examination is performed to obtain information regarding the number, size, and location of gestational sacs; the presence or absence of fetal cardiac and body movements; the presences or absence of uterine abnormalities (e.g., bicornuate uterus or fibroids) or adnexal masses (e.g., ovarian cysts or an ectopic pregnancy); and pregnancy dating.

24. During which phase of the cycle of violence does the batterer become contrite and remorseful? A. Battering phase B. Honeymoon phase C. Tension-building phase D. Increased drug-taking phase

ANS: B During the tension-building phase, the batterer becomes increasingly hostile, swears, threatens, and throws things. This is followed by the battering phase where violence actually occurs, and the victim feels powerless. During the honeymoon phase, the victim of IPV wants to believe that the battering will never happen again, and the batterer will promise anything to get back into the home. Often the batterer increases the use of drugs during the tension-building phase.

21. Which statement is the best rationale for assessing the maternal vital signs between uterine contractions? a. During a contraction, assessing the fetal heart rate is the priority. b. Maternal circulating blood volume temporarily increases during contractions. c. Maternal blood flow to the heart is reduced during contractions. d. Vital signs taken during contractions are not accurate.

ANS: B During uterine contractions, blood flow to the placenta temporarily stops, causing a relative increase in the mother's blood volume, which, in turn, temporarily increases blood pressure and slows the pulse. Monitoring fetal responses to the contractions is important; however, this question concerns the maternal vital signs. Maternal blood flow is increased during a contraction. Vital signs are altered by contractions but are considered accurate for that period.

9. During her gynecologic checkup, a 17-year-old girl states that recently she has been experiencing cramping and pain during her menstrual periods. The nurse would document this complaint as: A. amenorrhea. B. dysmenorrhea. C. dyspareunia. D. premenstrual syndrome (PMS).

ANS: B Dysmenorrhea is pain during or shortly before menstruation. Amenorrhea is the absence of menstrual flow. Dyspareunia is pain during intercourse. PMS is a cluster of physical, psychologic, and behavioral symptoms that begin in the luteal phase of the menstrual cycle and resolve within a couple of days of the onset of menses.

4. A pregnant woman experiencing nausea and vomiting should: A. drink a glass of water with a fat-free carbohydrate before getting out of bed in the morning. B. eat small, frequent meals (every 2 to 3 hours). C. increase her intake of high-fat foods to keep the stomach full and coated. D. limit fluid intake throughout the day.

ANS: B Eating small, frequent meals is the correct suggestion for a woman experiencing nausea and vomiting. A pregnant woman experiencing nausea and vomiting should avoid consuming fluids early in the day or when nauseated, but should compensate by drinking fluids at other times. A pregnant woman experiencing nausea and vomiting should reduce her intake of fried and other fatty foods.

19. With regard to systemic analgesics administered during labor, nurses should be aware that: a. systemic analgesics cross the maternal blood-brain barrier as easily as they do the fetal blood-brain barrier. b. effects on the fetus and newborn can include decreased alertness and delayed sucking. c. intramuscular (IM) administration is preferred over intravenous (IV) administration. d. IV patient-controlled analgesia (PCA) results in increased use of an analgesic.

ANS: B Effects depend on the specific drug given, the dosage, and the timing. Systemic analgesics cross the fetal blood-brain barrier more readily than the maternal blood-brain barrier. IV administration is preferred over IM administration because the drug acts faster and more predictably. PCA results in decreased use of an analgesic.

11. A married couple is discussing alternatives for pregnancy prevention and has asked about fertility awareness methods (FAMs). The nurse's most appropriate reply is: A. "They're not very effective, and it's very likely you'll get pregnant." B. "They can be effective for many couples, but they require motivation." C. "These methods have a few advantages and several health risks." D. "You would be much safer going on the pill and not having to worry."

ANS: B FAMs are effective with proper vigilance about ovulatory changes in the body and adherence to coitus intervals. They are effective if used correctly by a woman with a regular menstrual cycle. The typical failure rate for all FAMs is 25% during the first year of use. FAMs have no associated health risks. The use of birth control has associated health risks. In addition, taking a pill daily requires compliance on the patient's part.

6. With regard to the assessment of female, male, and couple infertility, nurses should be aware that: A. the couple's religious, cultural, and ethnic backgrounds provide emotional clutter that does not affect the clinical scientific diagnosis. B. the investigation takes 3 to 4 months and a significant financial investment. C. the woman is assessed first; if she is not the problem, the male partner is analyzed. D. semen analysis is for men; the postcoital test is for women.

ANS: B Fertility assessment and diagnosis take time, money, and commitment from the couple. Religious, cultural, and ethnic-bred attitudes about fertility and related issues always have an impact on diagnosis and assessment. Both partners are assessed systematically and simultaneously, as individuals and as a couple. Semen analysis is for men, but the postcoital test is for the couple.

24. To reassure and educate pregnant patients about the functioning of their kidneys in eliminating waste products, maternity nurses should be aware that: A. increased urinary output makes pregnant women less susceptible to urinary infection. B. increased bladder sensitivity and then compression of the bladder by the enlarging uterus results in the urge to urinate even if the bladder is almost empty. C. renal (kidney) function is more efficient when the woman assumes a supine position. D. using diuretics during pregnancy can help keep kidney function regular.

ANS: B First bladder sensitivity and then compression of the bladder by the uterus result in the urge to urinate more often. Numerous anatomic changes make a pregnant woman more susceptible to urinary tract infection. Renal function is more efficient when the woman lies in the lateral recumbent position and less efficient when she is supine. Diuretic use during pregnancy can overstress the system and cause problems.

15. Which time-based description of a stage of development in pregnancy is accurate? A. Viability—22 to 37 weeks since the last menstrual period (LMP) (assuming a fetal weight >500 g). B. Full Term—Pregnancy from the beginning of week 39 of gestation to the end of week 40. C. Preterm—Pregnancy from 20 to 28 weeks. D. Postdate—Pregnancy that extends beyond 38 weeks.

ANS: B Full Term is 39 to 40 weeks of gestation. Viability is the ability of the fetus to live outside the uterus before coming to term, or 22 to 24 weeks since LMP. Preterm is 20 to 37 weeks of gestation. Postdate or postterm is a pregnancy that extends beyond 42 weeks or what is considered the limit of full term.

18. The viral sexually transmitted infection (STI) that affects most people in the United States today is: A. herpes simplex virus type 2 (HSV-2). B. human papillomavirus (HPV). C. human immunodeficiency virus (HIV). D. cytomegalovirus (CMV).

ANS: B HPV infection is the most prevalent viral STI seen in ambulatory health care settings. HSV-2, HIV, and CMV all are viral STIs but are not the most prevalent viral STIs.

21. With regard to spinal and epidural (block) anesthesia, nurses should know that: a. this type of anesthesia is commonly used for cesarean births but is not suitable for vaginal births. b. a high incidence of after-birth headache is seen with spinal blocks. c. epidural blocks allow the woman to move freely. d. spinal and epidural blocks are never used together.

ANS: B Headaches may be prevented or mitigated to some degree by a number of methods. Spinal blocks may be used for vaginal births, but the woman must be assisted through labor. Epidural blocks limit the woman's ability to move freely. Combined use of spinal and epidural blocks is becoming increasingly popular.

4. A woman enquires about herbal alternative methods for improving fertility. Which statement by the nurse is the most appropriate when instructing the patient in which herbal preparations to avoid while trying to conceive? A. "You should avoid nettle leaf, dong quai, and vitamin E while you are trying to get pregnant." B. "You may want to avoid licorice root, lavender, fennel, sage, and thyme while you are trying to conceive." C. "You should not take anything with vitamin E, calcium, or magnesium. They will make you infertile." D. "Herbs have no bearing on fertility."

ANS: B Herbs that a woman should avoid while trying to conceive include licorice root, yarrow, wormwood, ephedra, fennel, golden seal, lavender, juniper, flaxseed, pennyroyal, passionflower, wild cherry, cascara, sage, thyme, and periwinkle. Nettle leaf, dong quai, and vitamin E all promote fertility. Vitamin E, calcium, and magnesium may promote fertility and conception. All supplements and herbs should be purchased from trusted sources.

22. What is the primary difference between the labor of a nullipara and that of a multipara? a. Amount of cervical dilation b. Total duration of labor c. Level of pain experienced d. Sequence of labor mechanisms

ANS: B In a first-time pregnancy, the descent is usually slow but steady; in subsequent pregnancies, the descent is more rapid, resulting in a shorter duration of labor. Cervical dilation is the same for all labors. The level of pain is individual to the woman, not to the number of labors she has experienced. The sequence of labor mechanisms is the same with all labors.

4. A woman is at 14 weeks of gestation. The nurse would expect to palpate the fundus at which level? A. Not palpable above the symphysis at this time B. Slightly above the symphysis pubis C. At the level of the umbilicus D. Slightly above the umbilicus

ANS: B In normal pregnancies, the uterus grows at a predictable rate. It may be palpated above the symphysis pubis sometime between the 12th and 14th weeks of pregnancy. As the uterus grows, it may be palpated above the symphysis pubis sometime between the 12th and 14th weeks of pregnancy. The uterus rises gradually to the level of the umbilicus at 22 to 24 weeks of gestation.

27. A 25-year-old single woman comes to the gynecologist's office for a follow-up visit related to her abnormal Papanicolaou (Pap) smear. The test revealed that the patient has human papillomavirus (HPV). The patient asks, "What is that? Can you get rid of it?" Your best response is: A. "It's just a little lump on your cervix. We can freeze it off." B. "HPV stands for 'human papillomavirus.' It is a sexually transmitted infection (STI) that may lead to cervical cancer. There is no known cure but symptoms are treated." C. "HPV is a type of early human immunodeficiency virus (HIV). You will die from this." D. "You probably caught this from your current boyfriend. He should get tested for this."

ANS: B It is important to inform the patient about STIs and the risks involved with HPV. The health care team has a duty to provide proper information to the patient, including information related to STIs. HPV and HIV are both viruses that can be transmitted sexually, but they are not the same virus. The onset of HPV can be insidious. Often STIs go unnoticed. Abnormal bleeding frequently is the initial symptom. The patient may have had HPV before her current boyfriend. You cannot make any deductions from this limited information.

30. The most important reason for evaluating the pattern of weight gain in pregnancy is to: A. prevent excessive adipose tissue deposits. B. identify potential nutritional problems or complications of pregnancy. C. assess the need to limit caloric intake in obese women. D. determine cultural influences on the woman's diet.

ANS: B Maternal and fetal risks in pregnancy are increased when the mother is significantly overweight. Excessive adipose tissue may occur with excess weight gain; however, this is not the reason for monitoring the weight gain pattern. It is important to monitor the pattern of weight gain to identify complications. The pattern of weight gain is not influenced by cultural influences.

27. A woman is 15 weeks pregnant with her first baby. She asks how long it will be before she feels the baby move. The best answer is: A. "You should have felt the baby move by now." B. "Within the next month, you should start to feel fluttering sensations." C. "The baby is moving; however, you can't feel it yet." D. "Some babies are quiet, and you don't feel them move."

ANS: B Maternal perception of fetal movement usually begins 16 to 20 weeks after conception. Because this is her first pregnancy, movement is felt toward the later part of the 16- to 20-week time period. Stating that "you should have felt the baby move by now" is incorrect and may be alarming to the patient. Fetal movement should be felt by 16 to 20 weeks. If movement is not felt by the end of that time, further assessment will be necessary.

11. During the first trimester, a woman can expect which of the following changes in her sexual desire? A. An increase, because of enlarging breasts B. A decrease, because of nausea and fatigue C. No change D. An increase, because of increased levels of female hormones

ANS: B Maternal physiologic changes such as breast enlargement, nausea, fatigue, abdominal changes, perineal enlargement, leukorrhea, pelvic vasocongestion, and orgasmic responses may affect sexuality and sexual expression. Libido may be depressed in the first trimester but often increases during the second and third trimesters. During pregnancy, the breasts may become enlarged and tender; this tends to interfere with coitus, decreasing the desire to engage in sexual activity.

6. While interviewing a 31-year-old woman before her routine gynecologic examination, the nurse collects data about the patient's recent menstrual cycles. The nurse should collect additional information associated with which patient statement? TA. he woman says her menstrual flow lasts 5 to 6 days. B. She describes her flow as very heavy. C. She reports that she has had a small amount of spotting midway between her periods for the past 2 months. D. She says the length of her menstrual cycle varies from 26 to 29 days.

ANS: B Menorrhagia is defined as excessive menstrual bleeding, in either duration or amount. Heavy bleeding can have many causes. The amount of bleeding and its effect on daily activities should be evaluated. A menstrual flow lasting 5 to 6 days is a normal finding. Mittlestaining, a small amount of bleeding or spotting that occurs at the time of ovulation (14 days before onset of the next menses), is considered normal. During her reproductive years, a woman may have physiologic variations in her menstrual cycle. Variations in the length of a menstrual cycle are considered normal.

16. The use of methamphetamine (meth) has been described as a significant drug problem in the United States. In order to provide adequate nursing care to this patient population the nurse must be cognizant that methamphetamine: A. is similar to opiates. B. is a stimulant with vasoconstrictive characteristics. C. should not be discontinued during pregnancy. D. is associated with a low rate of relapse.

ANS: B Methamphetamines are stimulants with vasoconstrictive characteristics similar to cocaine and are used similarly. As is the case with cocaine users, methamphetamine users are urged to immediately stop all use during pregnancy. Unfortunately, because methamphetamine users are extremely psychologically addicted, the rate of relapse is very high.

9. What is the correct term describing the slight overlapping of cranial bones or shaping of the fetal head during labor? a. Lightening b. Molding c. Ferguson reflex d. Valsalva maneuver

ANS: B Molding also permits adaptation to various diameters of the maternal pelvis. Lightening is the mother's sensation of decreased abdominal distention, which usually occurs the week before labor. The Ferguson reflex is the contraction urge of the uterus after the stimulation of the cervix. The Valsalva maneuver describes conscious pushing during the second stage of labor.

35. The primary difference between the labor of a nullipara and that of a multipara is the: a. amount of cervical dilation. b. total duration of labor. c. level of pain experienced. d. sequence of labor mechanisms.

ANS: B Multiparas usually labor more quickly than nulliparas, thus making the total duration of their labor shorter. Cervical dilation is the same for all labors. The level of pain is individual to the woman, not to the number of labors she has experienced. The sequence of labor mechanisms remains the same with all labors.

30. A patient in her first trimester complains of nausea and vomiting. She asks, "Why does this happen?" The nurse's best response is: A. "It is due to an increase in gastric motility." B. "It may be due to changes in hormones." C. "It is related to an increase in glucose levels." D. "It is caused by a decrease in gastric secretions."

ANS: B Nausea and vomiting are believed to be caused by increased levels of hormones, decreased gastric motility, and hypoglycemia. Gastric motility decreases during pregnancy. Glucose levels decrease in the first trimester. Although gastric secretions decrease, this is not the main cause of nausea and vomiting.

9. The role of the nurse with regard to informed consent is to: a. inform the patient about the procedure and have her sign the consent form. b. act as a patient advocate and help clarify the procedure and the options. c. call the physician to see the patient. d. witness the signing of the consent form.

ANS: B Nurses play a part in the informed consent process by clarifying and describing procedures or by acting as the woman's advocate and asking the primary health care provider for further explanations. The physician is responsible for informing the woman of her options, explaining the procedure, and advising the patient about potential risk factors. The physician must be present to explain the procedure to the patient. However, the nurse's responsibilities go further than simply asking the physician to see the patient. The nurse may witness the signing of the consent form. However, depending on the state's guidelines, the woman's husband or another hospital health care employee may sign as witness.

15. The nurse expects to administer an oxytocic (e.g., Pitocin, Methergine) to a woman after expulsion of her placenta to: a. relieve pain. b. stimulate uterine contraction. c. prevent infection. d. facilitate rest and relaxation.

ANS: B Oxytocics stimulate uterine contractions, which reduce blood loss after the third stage of labor. Oxytocics are not used to treat pain or prevent infection. They cause the uterus to contract, which reduces blood loss. Oxytocics do not facilitate rest and relaxation.

21. Certain fatty acids classified as hormones that are found in many body tissues and that have roles in many reproductive functions are known as: A. gonadotropin-releasing hormone (GnRH). B. prostaglandins (PGs). C. follicle-stimulating hormone (FSH). D. luteinizing hormone (LH).

ANS: B PGs affect smooth muscle contraction and changes in the cervix. GnRH, FSH, and LH are part of the hypothalamic-pituitary cycle, which responds to the rise and fall of estrogen and progesterone.

12. A new mother with which of these thyroid disorders would be strongly discouraged from breastfeeding? A. Hyperthyroidism B. Phenylketonuria (PKU) C. Hypothyroidism D. Thyroid storm

ANS: B PKU is a cause of mental retardation in infants; mothers with PKU pass on phenylalanine. A woman with hyperthyroidism or hypothyroidism would have no particular reason not to breastfeed. A thyroid storm is a complication of hyperthyroidism.

8. A pregnant woman at 18 weeks of gestation calls the clinic to report that she has been experiencing occasional backaches of mild-to-moderate intensity. The nurse would recommend that she: A. do Kegel exercises. B. do pelvic rock exercises. C. use a softer mattress. D. stay in bed for 24 hours.

ANS: B Pelvic rock exercises may help stretch and strengthen the abdominal and lower back muscles and relieve low back pain. Kegel exercises increase the tone of the pelvic area, not the back. A softer mattress may not provide the support needed to maintain proper alignment of the spine and may contribute to back pain. Stretching and other exercises to relieve back pain should be performed several times a day.

9. Physiologically, sexual response can be characterized by: A. coitus, masturbation, and fantasy. B. myotonia and vasocongestion. C. erection and orgasm. D. excitement, plateau, and orgasm.

ANS: B Physiologically, according to Masters (1992), sexual response can be analyzed in terms of two processes: vasocongestion and myotonia. Coitus, masturbation, and fantasy are forms of stimulation for the physical manifestation of the sexual response. Erection and orgasm occur in two of the four phases of the sexual response cycle. Excitement, plateau, and orgasm are three of the four phases of the sexual response cycle.

14. Risk factors tend to be interrelated and cumulative in their effect. While planning the care for a laboring patient with diabetes mellitus, the nurse is aware that she is at a greater risk for: A. oligohydramnios. B. polyhydramnios. C. postterm pregnancy. D. Chromosomal abnormalities.

ANS: B Polyhydramnios (amniotic fluid >2000 mL) is 10 times more likely to occur in diabetic compared with nondiabetic pregnancies. Polyhydramnios puts the mother at risk for premature rupture of membranes, premature labor, and after birth hemorrhage. Prolonged rupture of membranes, intrauterine growth restriction, intrauterine fetal death, and renal agenesis (Potter syndrome) all put the patient at risk for developing oligohydramnios. Anencephaly, placental insufficiency, and perinatal hypoxia all contribute to the risk for postterm pregnancy. Maternal age older than 35 years and balanced translocation (maternal and paternal) are risk factors for chromosome abnormalities.

16. With regard to dysfunctional labor, nurses should be aware that: a. women who are underweight are more at risk. b. women experiencing precipitous labor are about the only "dysfunctionals" not to be exhausted. c. hypertonic uterine dysfunction is more common than hypotonic dysfunction. d. abnormal labor patterns are most common in older women.

ANS: B Precipitous labor lasts less than 3 hours. Short women more than 30 lbs overweight are more at risk for dysfunctional labor. Hypotonic uterine dysfunction, in which the contractions become weaker, is more common. Abnormal labor patterns are more common in women less than 20 years of age.

2. Preconception counseling is critical to the outcome of diabetic pregnancies because poor glycemic control before and during early pregnancy is associated with: A. frequent episodes of maternal hypoglycemia. B. congenital anomalies in the fetus. C. polyhydramnios. D. hyperemesis gravidarum.

ANS: B Preconception counseling is particularly important because strict metabolic control before conception and in the early weeks of gestation is instrumental in decreasing the risks of congenital anomalies. Frequent episodes of maternal hypoglycemia may occur during the first trimester (not before conception) as a result of hormone changes and the effects on insulin production and usage. Hydramnios occurs about 10 times more often in diabetic pregnancies than in nondiabetic pregnancies. Typically it is seen in the third trimester of pregnancy. Hyperemesis gravidarum may exacerbate hypoglycemic events because the decreased food intake by the mother and glucose transfer to the fetus contributes to hypoglycemia.

1. The nurse caring for a newly pregnant woman would advise her that ideally prenatal care should begin: A. before the first missed menstrual period. B. after the first missed menstrual period. C. after the second missed menstrual period. D. after the third missed menstrual period.

ANS: B Prenatal care ideally should begin soon after the first missed menstrual period. Regular prenatal visits offer opportunities to ensure the health of the expectant mother and her infant.

4. A 41-week pregnant multigravida presents in the labor and delivery unit after a nonstress test indicated that her fetus could be experiencing some difficulties in utero. Which diagnostic tool would yield more detailed information about the fetus? A. Ultrasound for fetal anomalies B. Biophysical profile (BPP) C. Maternal serum alpha-fetoprotein (MSAFP) screening D. Percutaneous umbilical blood sampling (PUBS)

ANS: B Real-time ultrasound permits detailed assessment of the physical and physiologic characteristics of the developing fetus and cataloging of normal and abnormal biophysical responses to stimuli. BPP is a noninvasive, dynamic assessment of a fetus that is based on acute and chronic markers of fetal disease. An ultrasound for fetal anomalies would most likely have been performed earlier in the pregnancy. It is too late in the pregnancy to perform MSAFP screening. Also, MSAFP screening does not provide information related to fetal well-being. Indications for PUBS include prenatal diagnosis or inherited blood disorders, karyotyping of malformed fetuses, detection of fetal infection, determination of the acid-base status of a fetus with IUGR, and assessment and treatment of isoimmunization and thrombocytopenia in the fetus.

14. What three measures should the nurse implement to provide intrauterine resuscitation? Select the response that best indicates the priority of actions that should be taken. a. Call the provider, reposition the mother, and perform a vaginal examination. b. Reposition the mother, increase intravenous (IV) fluid, and provide oxygen via face mask. c. Administer oxygen to the mother, increase IV fluid, and notify the care provider. d. Perform a vaginal examination, reposition the mother, and provide oxygen via face mask.

ANS: B Repositioning the mother, increasing intravenous (IV) fluid, and providing oxygen via face mask are correct nursing actions for intrauterine resuscitation. The nurse should initiate intrauterine resuscitation in an ABC manner, similar to basic life support. The first priority is to open the maternal and fetal vascular systems by repositioning the mother for improved perfusion. The second priority is to increase blood volume by increasing the IV fluid. The third priority is to optimize oxygenation of the circulatory volume by providing oxygen via face mask. If these interventions do not resolve the fetal heart rate issue quickly, the primary provider should be notified immediately.

10. A first-time mother is concerned about the type of medications she will receive during labor. She is in a fair amount of pain and is nauseous. In addition, she appears to be very anxious. You explain that opioid analgesics are often used with sedatives because: a. "The two together work the best for you and your baby." b. "Sedatives help the opioid work better, and they also will assist you to relax and relieve your nausea." c. "They work better together so you can sleep until you have the baby." d. "This is what the doctor has ordered for you."

ANS: B Sedatives can be used to reduce the nausea and vomiting that often accompany opioid use. In addition, some ataractics reduce anxiety and apprehension and potentiate the opioid analgesic affects. A potentiator may cause the two drugs to work together more effectively, but it does not ensure maternal or fetal complications will not occur. Sedation may be a related effect of some ataractics, but it is not the goal. Furthermore, a woman is unlikely to be able to sleep through transitional labor and birth. "This is what the doctor has ordered for you" may be true, but it is not an acceptable comment for the nurse to make.

27. Which finding in the urine analysis of a pregnant woman is considered a variation of normal? A. Proteinuria B. Glycosuria C. Bacteria in the urine D. Ketones in the urine

ANS: B Small amounts of glucose may indicate "physiologic spilling." The presence of protein could indicate kidney disease or preeclampsia. Urinary tract infections are associated with bacteria in the urine. An increase in ketones indicates that the patient is exercising too strenuously or has an inadequate fluid and food intake.

16. After an emergency birth, the nurse encourages the woman to breastfeed her newborn. The primary purpose of this activity is to: a. facilitate maternal-newborn interaction. b. stimulate the uterus to contract. c. prevent neonatal hypoglycemia. d. initiate the lactation cycle.

ANS: B Stimulation of the nipples through breastfeeding or manual stimulation causes the release of oxytocin and prevents maternal hemorrhage. Breastfeeding facilitates maternal-newborn interaction, but it is not the primary reason a woman is encouraged to breastfeed after an emergency birth. The primary intervention for preventing neonatal hypoglycemia is thermoregulation. Cold stress can result in hypoglycemia. The woman is encouraged to breastfeed after an emergency birth to stimulate the release of oxytocin, which prevents hemorrhage. Breastfeeding is encouraged to initiate the lactation cycle, but it is not the primary reason for this activity after an emergency birth.

24. Nurses can help their clients by keeping them informed about the distinctive stages of labor. Which description of the phases of the first stage of labor is accurate? a. Latent: Mild, regular contractions; no dilation; bloody show; duration of 2 to 4 hours b. Active: Moderate, regular contractions; 4- to 7-cm dilation; duration of 3 to 6 hours c. Lull: No contractions; dilation stable; duration of 20 to 60 minutes d. Transition: Very strong but irregular contractions; 8- to 10-cm dilation; duration of 1 to 2 hours

ANS: B The active phase is characterized by moderate, regular contractions; 4- to 7-cm dilation; and a duration of 3 to 6 hours. The latent phase is characterized by mild-to-moderate and irregular contractions; dilation up to 3 cm; brownish-to-pale pink mucus, and a duration of 6 to 8 hours. No official "lull" phase exists in the first stage. The transition phase is characterized by strong- to-very strong and regular contractions; 8- to 10-cm dilation; and a duration of 20 to 40 minutes.

20. Nurses can help their patients by keeping them informed about the distinctive stages of labor. Which description of the phases of the first stage of labor is accurate? .a Latent: Mild, regular contractions; no dilation; bloody show; duration of 2 to 4 hours b. Active: Moderate, regular contractions; 4- to 7-cm dilation; duration of 3 to 6 hours c. Lull: No contractions; dilation stable; duration of 20 to 60 minutes d. Transition: Very strong but irregular contractions; 8- to 10-cm dilation; duration of 1 to 2 hours

ANS: B The active phase is characterized by moderate, regular contractions; 4- to 7-cm dilation; and a duration of 3 to 6 hours. The latent phase is characterized by mild-to-moderate, irregular contractions; dilation up to 3 cm; brownish-to-pale pink mucus, and a duration of 6 to 8 hours. No official "lull" phase exists in the first stage. The transition phase is characterized by strong to very strong, regular contractions; 8- to 10-cm dilation; and a duration of 20 to 40 minutes.

24. During labor a fetus with an average heart rate of 135 beats/min over a 10-minute period would be considered to have: a. bradycardia. b. a normal baseline heart rate. c. tachycardia. d. hypoxia.

ANS: B The baseline heart rate is measured over 10 minutes; a normal range is 110 to 160 beats/min. Bradycardia is a fetal heart rate (FHR) below 110 beats/min for 10 minutes or longer. Tachycardia is an FHR over 160 beats/min for 10 minutes or longer. Hypoxia is an inadequate supply of oxygen; no indication of this condition exists with a baseline heart rate in the normal range.

18. The body part that both protects the pelvic structures and accommodates the growing fetus during pregnancy is the: A. perineum. B. bony pelvis. C. vaginal vestibule. D. fourchette.

ANS: B The bony pelvis protects and accommodates the growing fetus. The perineum covers the pelvic structures. The vaginal vestibule contains openings to the urethra and vagina. The fourchette is formed by the labia minor.

1. A woman's obstetric history indicates that she is pregnant for the fourth time and all of her children from previous pregnancies are living. One was born at 39 weeks of gestation, twins were born at 34 weeks of gestation, and another child was born at 35 weeks of gestation. What is her gravidity and parity using the GTPAL system? a. 3-1-1-1-3 b. 4-1-2-0-4 c. 3-0-3-0-3 d. 4-2-1-0-3

ANS: B The correct calculation of this woman's gravidity and parity is 4-1-2-0-4. The numbers reflect the woman's gravidity and parity information. Using the GPTAL system, her information is calculated as:G: The first number reflects the total number of times the woman has been pregnant; she is pregnant for the fourth time.T: This number indicates the number of pregnancies carried to term, not the number of deliveries at term; only one of her pregnancies has resulted in a fetus at term.P: This is the number of pregnancies that resulted in a preterm birth; the woman has had two pregnancies in which she delivered preterm.A: This number signifies whether the woman has had any abortions or miscarriages before the period of viability; she has not.L: This number signifies the number of children born who are currently living; the woman has four children.

5. A man's wife is pregnant for the third time. One child was born with cystic fibrosis, and the other child is healthy. The man wonders what the chance is that this child will have cystic fibrosis. This type of testing is known as: A. occurrence risk. B. recurrence risk. C. predictive testing. D. predisposition testing.

ANS: B The couple already has a child with a genetic disease so they will be given a recurrence risk test. If a couple has not yet had children but are known to be at risk for having children with a genetic disease, they are given an occurrence risk test. Predictive testing is used to clarify the genetic status of an asymptomatic family member. Predisposition testing differs from presymptomatic testing in that a positive result does not indicate 100% risk of a condition developing.

12. Through vaginal examination the nurse determines that a woman is 4 cm dilated, and the external fetal monitor shows uterine contractions every 3.5 to 4 minutes. The nurse would report this as: a. first stage, latent phase. b. first stage, active phase. c. first stage, transition phase. d. second stage, latent phase.

ANS: B The first stage, active phase of maternal progress indicates that the woman is in the active phase of the first stage of labor. During the latent phase of the first stage of labor, the expected maternal progress would be 0 to 3 cm dilation with contractions every 5 to 30 minutes. During the transition phase of the first stage of labor, the expected maternal progress is 8 to 10 cm dilation with contractions every 2 to 3 minutes. During the latent phase of the second stage of labor, the woman is completely dilated and experiences a restful period of "laboring down."

3. Unique muscle fibers make the uterine myometrium ideally suited for: A. menstruation. B. the birth process. C. ovulation. D. fertilization.

ANS: B The myometrium is made up of layers of smooth muscles that extend in three directions. These muscles assist in the birth process by expelling the fetus, ligating blood vessels after birth, and controlling the opening of the cervical os.

32. For women who have a history of sexual abuse, a number of traumatic memories may be triggered during labor. The woman may fight the labor process and react with pain or anger. Alternately, she may become a passive player and emotionally absent herself from the process. The nurse is in a unique position of being able to assist the patient to associate the sensations of labor with the process of childbirth and not the past abuse. The nurse can implement a number of care measures to help the patient view the childbirth experience in a positive manner. Which intervention would be key for the nurse to use while providing care? a. Telling the patient to relax and that it won't hurt much. b. Limiting the number of procedures that invade her body. c. Reassuring the patient that as the nurse you know what is best. d. Allowing unlimited care providers to be with the patient.

ANS: B The number of invasive procedures such as vaginal examinations, internal monitoring, and intravenous therapy should be limited as much as possible. The nurse should always avoid words and phrases that may result in the patient's recalling the phrases of her abuser (e.g., "Relax, this won't hurt" or "Just open your legs.") The woman's sense of control should be maintained at all times. The nurse should explain procedures at the patient's pace and wait for permission to proceed. Protecting the patient's environment by providing privacy and limiting the number of staff who observe the patient will help to make her feel safe.

12. A 62-year-old woman has not been to the clinic for an annual examination for 5 years. The recent death of her husband reminded her that she should come for a visit. Her family doctor has retired, and she is going to see the women's health nurse practitioner for her visit. To facilitate a positive health care experience, the nurse should: A. remind the woman that she is long overdue for her examination and that she should come in annually. B. listen carefully and allow extra time for this woman's health history interview. C. reassure the woman that a nurse practitioner is just as good as her old doctor. D. encourage the woman to talk about the death of her husband and her fears about her own death.

ANS: B The nurse has an opportunity to use reflection and empathy while listening and to ensure open and caring communication. Scheduling a longer appointment time may be necessary because older women may have longer histories or may need to talk. A respectful and reassuring approach to caring for women older than age 50 can help ensure that they continue to seek health care. Reminding the woman about her overdue examination, reassuring the woman that she has a good practitioner, and encouraging conversation about the death of her husband and her own death are not the best approaches with women in this age-group.

23. As relates to the structure and function of the placenta, the maternity nurse should be aware that: A. as the placenta widens, it gradually thins to allow easier passage of air and nutrients. B. as one of its early functions, the placenta acts as an endocrine gland. C. the placenta is able to keep out most potentially toxic substances such as cigarette smoke to which the mother is exposed. D. optimal blood circulation is achieved through the placenta when the woman is lying on her back or standing.

ANS: B The placenta produces four hormones necessary to maintain the pregnancy. The placenta widens until week 20 and continues to grow thicker. Toxic substances such as nicotine and carbon monoxide readily cross the placenta into the fetus. Optimal circulation occurs when the woman is lying on her side.

28. The maternity nurse understands that vascular volume increases 40% to 45% during pregnancy to: A. compensate for decreased renal plasma flow. B. provide adequate perfusion of the placenta. C. eliminate metabolic wastes of the mother. D. prevent maternal and fetal dehydration.

ANS: B The primary function of increased vascular volume is to transport oxygen and nutrients to the fetus via the placenta. Renal plasma flow increases during pregnancy. Assisting with pulling metabolic wastes from the fetus for maternal excretion is one purpose of the increased vascular volume.

21. To detect human immunodeficiency virus (HIV), most laboratory tests focus on the: A. virus. B. HIV antibodies. C. CD4 counts. D. CD8 counts.

ANS: B The screening tool used to detect HIV is the enzyme immunoassay, which tests for the presence of antibodies to the virus. CD4 counts are associated with the incidence of acquired immunodeficiency syndrome (AIDS) in HIV-infected individuals.

17. To reassure and educate pregnant patients about changes in the uterus, nurses should be aware that: A. lightening occurs near the end of the second trimester as the uterus rises into a different position. B. the woman's increased urinary frequency in the first trimester is the result of exaggerated uterine anteflexion caused by softening. C. Braxton Hicks contractions become more painful in the third trimester, particularly if the woman tries to exercise. D. the uterine souffle is the movement of the fetus.

ANS: B The softening of the lower uterine segment is called Hegar's sign. Lightening occurs in the last 2 weeks of pregnancy, when the fetus descends. Braxton Hicks contractions become more defined in the final trimester but are not painful. Walking or exercise usually causes them to stop. The uterine souffle is the sound made by blood in the uterine arteries; it can be heard with a fetal stethoscope.

12. A male patient asks the nurse why it is better to purchase condoms that are not lubricated with nonoxynol-9 (a common spermicide). The nurse's most appropriate response is: A. "The lubricant prevents vaginal irritation." B. "It has also been linked to an increase in the transmission of human immunodeficiency virus." C. "The additional lubrication improves sex." D. "Nonoxynol-9 improves penile sensitivity."

ANS: B The statement "Nonoxynol-9 does not provide protection against sexually transmitted infections, as originally thought; it has also been linked to an increase in the transmission of human immunodeficiency virus and can cause genital lesions" is true. Nonoxynol-9 may cause vaginal irritation, has no effect on the quality of sexual activity, and has no effect on penile sensitivity.

16. Which statement related to fetal positioning during labor is correct and important for the nurse to understand? a. Position is a measure of the degree of descent of the presenting part of the fetus through the birth canal. b. Birth is imminent when the presenting part is at +4 to +5 cm below the spine. c. The largest transverse diameter of the presenting part is the suboccipitobregmatic diameter. d. Engagement is the term used to describe the beginning of labor.

ANS: B The station of the presenting part should be noted at the beginning of labor to determine the rate of descent. Position is the relationship of the presenting part of the fetus to the four quadrants of the mother's pelvis; station is the measure of degree of descent. The largest diameter is usually the biparietal diameter. The suboccipitobregmatic diameter is the smallest, although one of the most critical. Engagement often occurs in the weeks just before labor in nulliparous women and before or during labor in multiparous women.

5. The nurse has received a report regarding a client in labor. The woman's last vaginal examination was recorded as 3 cm, 30%, and -2. What is the nurse's interpretation of this assessment? a. Cervix is effaced 3 cm and dilated 30%; the presenting part is 2 cm above the ischial spines. b. Cervix is dilated 3 cm and effaced 30%; the presenting part is 2 cm above the ischial spines. c. Cervix is effaced 3 cm and dilated 30%; the presenting part is 2 cm below the ischial spines. d. Cervix is dilated 3 cm and effaced 30%; the presenting part is 2 cm below the ischial spines.

ANS: B The sterile vaginal examination is recorded as centimeters of cervical dilation, percentage of cervical dilation, and the relationship of the presenting part to the ischial spines (either above or below). For this woman, the cervix is dilated 3 cm and effaced 30%, and the presenting part is 2 cm above the ischial spines. The first interpretation of this vaginal examination is incorrect; the cervix is dilated 3 cm and is 30% effaced. However, the presenting part is correct at 2 cm above the ischial spines. The remaining two interpretations of this vaginal examination are incorrect. Although the dilation and effacement are correct at 3 cm and 30%, the presenting part is actually 2 cm above the ischial spines.

25. With regard to the use of intrauterine devices (IUDs), nurses should be aware that: A. return to fertility can take several weeks after the device is removed. B. IUDs containing copper can provide an emergency contraception option if inserted within a few days of unprotected intercourse. C. IUDs offer the same protection against sexually transmitted infections (STIs) as the diaphragm. D. consent forms are not needed for IUD insertion.

ANS: B The woman has up to 5 days to insert the IUD after unprotected sex. Return to fertility is immediate after removal of the IUD. IUDs offer no protection for STIs. A consent form is required for insertion, as is a negative pregnancy test.

8. If an opioid antagonist is administered to a laboring woman, she should be told that: a. her pain will decrease. b. her pain will return. c. she will feel less anxious. d. she will no longer feel the urge to push.

ANS: B The woman should be told that the pain that was relieved by the opioid analgesic will return with administration of the opioid antagonist. Opioid antagonists, such as Narcan, promptly reverse the central nervous system (CNS) depressant effects of opioids. In addition, the antagonist counters the effect of the stress-induced levels of endorphins. An opioid antagonist is especially valuable if labor is more rapid than expected and birth is anticipated when the opioid is at its peak effect.

29. Which statement by the patient indicates that she understands breast self-examination? A. "I will examine both breasts in two different positions." B. "I will perform breast self-examination 1 week after my menstrual period starts." C. "I will examine the outer upper area of the breast only." D. "I will use the palm of the hand to perform the examination."

ANS: B The woman should examine her breasts when hormonal influences are at their lowest level. The patient should be instructed to use four positions: standing with arms at her sides, standing with arms raised above her head, standing with hands pressed against hips, and lying down. The entire breast needs to be examined, including the outer upper area. The patient should use the sensitive pads of the middle three fingers.

12. The nurse, caring for a patient whose labor is being augmented with oxytocin, recognizes that the oxytocin should be discontinued immediately if there is evidence of: a. uterine contractions occurring every 8 to 10 minutes. b. a fetal heart rate (FHR) of 180 with absence of variability. c. the patient's needing to void. d. rupture of the patient's amniotic membranes.

ANS: B This FHR is nonreassuring. The oxytocin should be discontinued immediately, and the physician should be notified. The oxytocin should be discontinued if uterine hyperstimulation occurs. Uterine contractions that are occurring every 8 to 10 minutes do not qualify as hyperstimulation. The patient's needing to void is not an indication to discontinue the oxytocin induction immediately or to call the physician. Unless a change occurs in the FHR pattern that is nonreassuring or the patient experiences uterine hyperstimulation, the oxytocin does not need to be discontinued. The physician should be notified that the patient's membranes have ruptured.

5. A 26-year-old primigravida has come to the clinic for her regular prenatal visit at 12 weeks. She appears thin and somewhat nervous. She reports that she eats a well-balanced diet, although her weight is 5 lbs less than it was at her last visit. The results of laboratory studies confirm that she has a hyperthyroid condition. Based on the available data, the nurse formulates a plan of care. What nursing diagnosis is most appropriate for the woman at this time? A. Deficient fluid volume B. Imbalanced nutrition: less than body requirements C. Imbalanced nutrition: more than body requirements D. Disturbed sleep pattern

ANS: B This patient's clinical cues include weight loss, which would support the nursing diagnosis of Imbalanced nutrition: less than body requirements. No clinical signs or symptoms support the nursing diagnosis of Deficient fluid volume. This patient reports weight loss, not weight gain. Imbalanced nutrition: more than body requirements is not an appropriate nursing diagnosis. Although the patient reports nervousness based on the patient's other clinical symptoms the most appropriate nursing diagnosis would be Imbalanced nutrition: less than body requirements.

25. Which statement about multifetal pregnancy is inaccurate? A. The expectant mother often develops anemia because the fetuses have a greater demand for iron. B. Twin pregnancies come to term with the same frequency as single pregnancies. C. The mother should be counseled to increase her nutritional intake and gain more weight. D. Backache and varicose veins often are more pronounced.

ANS: B Twin pregnancies often end in prematurity. Serious efforts should be made to bring the pregnancy to term. A woman with a multifetal pregnancy often develops anemia, suffers more or worse backache, and needs to gain more weight. Counseling is needed to help her adjust to these conditions.

7. In terms of the incidence and classification of diabetes, maternity nurses should know that: A. type 1 diabetes is most common. B. type 2 diabetes often goes undiagnosed. C. gestational diabetes mellitus (GDM) means that the woman will be receiving insulin treatment until 6 weeks after birth. D. type 1 diabetes may become type 2 during pregnancy.

ANS: B Type 2 diabetes often goes undiagnosed because hyperglycemia develops gradually and often is not severe. Type 2 diabetes, sometimes called adult onset diabetes, is the most common. GDM refers to any degree of glucose intolerance first recognized during pregnancy. Insulin may or may not be needed. People do not go back and forth between type 1 and 2 diabetes.

4. The nurse providing care for the laboring woman realizes that variable fetal heart rate (FHR) decelerations are caused by: a. altered fetal cerebral blood flow. b. umbilical cord compression. c. uteroplacental insufficiency. d. fetal hypoxemia.

ANS: B Variable decelerations can occur any time during the uterine contracting phase and are caused by compression of the umbilical cord. Altered fetal cerebral blood flow would result in early decelerations in the FHR. Uteroplacental insufficiency would result in late decelerations in the FHR. Fetal hypoxemia would result in tachycardia initially and then bradycardia if hypoxia continues.

29. Physiologic anemia often occurs during pregnancy as a result of: A. inadequate intake of iron. B. dilution of hemoglobin concentration. C. the fetus establishing iron stores. D. decreased production of erythrocytes.

ANS: B When blood volume expansion is more pronounced and occurs earlier than the increase in red blood cells, the woman has physiologic anemia, which is the result of dilution of hemoglobin concentration rather than inadequate hemoglobin. Inadequate intake of iron may lead to true anemia. There is an increased production of erythrocytes during pregnancy.

18. Which nursing intervention is necessary before a second-trimester transabdominal ultrasound? A. Place the woman NPO for 12 hours. B. Instruct the woman not void until after the test. C. Administer an enema. D. Perform an abdominal preparation.

ANS: B When the uterus is still in the pelvis, visualization may be difficult. It is necessary to perform the test when the woman has a full bladder, which provides a "window" through which the uterus and its contents can be viewed. The woman needs a full bladder to elevate the uterus; therefore being NPO is not appropriate. Neither an enema nor an abdominal preparation is necessary for this procedure.

1. A tiered system of categorizing FHR has been recommended by regulatory agencies. Nurses, midwives, and physicians who care for women in labor must have a working knowledge of fetal monitoring standards and understand the significance of each category. These categories include: (Select all that apply.) a. reassuring. b. Category I. c. Category II. d. nonreassuring. e. Category III.

ANS: B, C, E The three tiered system of FHR tracings include Category I, II, and III. Category I is a normal tracing requiring no action. Category II FHR tracings are indeterminate. This category includes tracings that do not meet Category I or III criteria. Category III tracings are abnormal and require immediate intervention.

13. A pregnant woman at 25 weeks' gestation tells the nurse that she dropped a pan last week and her baby jumped at the noise. Which response by the nurse is most accurate? A. "That must have been a coincidence; babies can't respond like that." B. "The fetus is demonstrating the aural reflex." C. "Babies respond to sound starting at about 24 weeks of gestation." D. "Let me know if it happens again; we need to report that to your midwife."

ANS: C "Babies respond to sound starting at about 24 weeks of gestation" is an accurate statement. "That must have been a coincidence; babies can't respond like that" is inaccurate. Acoustic stimulations can evoke a fetal heart rate response. There is no such thing as an aural reflex. The statement, "Let me know if it happens again; we need to report that to your midwife" is not appropriate; it gives the impression that something is wrong.

39. Which nursing assessment indicates that a woman who is in second-stage labor is almost ready to give birth? a. The fetal head is felt at 0 station during vaginal examination. b. Bloody mucus discharge increases. c. The vulva bulges and encircles the fetal head. d. The membranes rupture during a contraction.

ANS: C A bulging vulva that encircles the fetal head describes crowning, which occurs shortly before birth. Birth of the head occurs when the station is +4. A 0 station indicates engagement. Bloody show occurs throughout the labor process and is not an indication of an imminent birth. Rupture of membranes can occur at any time during the labor process and does not indicate an imminent birth.

24. With regard to the development of the respiratory system, maternity nurses should understand that: A. the respiratory system does not begin developing until after the embryonic stage. B. the infant's lungs are considered mature when the lecithin/sphingomyelin [L/S] ratio is 1:1, at about 32 weeks. C. maternal hypertension can reduce maternal-placental blood flow, accelerating lung maturity. D. fetal respiratory movements are not visible on ultrasound scans until at least 16 weeks.

ANS: C A reduction in placental blood flow stresses the fetus, increases blood levels of corticosteroids, and accelerates lung maturity. Development of the respiratory system begins during the embryonic phase and continues into childhood. The infant's lungs are mature when the L/S ratio is 2:1, at about 35 weeks. Lung movements have been seen on ultrasound scans at 11 weeks.

21. It is paramount for the obstetric nurse to understand the regulatory procedures and criteria for admitting a woman to the hospital labor unit. Which guideline is an important legal requirement of maternity care? a. The patient is not considered to be in true labor (according to the Emergency Medical Treatment and Active Labor Act [EMTALA]) until a qualified health care provider says she is. b. The woman can have only her male partner or predesignated "doula" with her at assessment. c. The patient's weight gain is calculated to determine whether she is at greater risk for cephalopelvic disproportion (CPD) and cesarean birth. d. The nurse may exchange information about the patient with family members.

ANS: C According to EMTALA, a woman is entitled to active labor care and is presumed to be in "true" labor until a qualified health care provider certifies otherwise. A woman can have anyone she wishes present for her support. The risk for CPD is especially great for petite women or those who have gained 16 kg or more. All patients should have their weight and BMI calculated on admission. This is part of standard nursing care on a maternity unit and not a regulatory concern. According to the Health Insurance Portability and Accountability Act (HIPAA), the patient must give consent for others to receive any information related to her condition.

14. Injectable progestins (DMPA, Depo-Provera) are a good contraceptive choice for women who: A. want menstrual regularity and predictability. B. have a history of thrombotic problems or breast cancer. C. have difficulty remembering to take oral contraceptives daily. D. are homeless or mobile and rarely receive health care.

ANS: C Advantages of DMPA include a contraceptive effectiveness comparable to that of combined oral contraceptives with the requirement of only four injections a year. Disadvantages of injectable progestins are prolonged amenorrhea and uterine bleeding. Use of injectable progestin carries an increased risk of venous thrombosis and thromboembolism. To be effective, DMPA injections must be administered every 11 to 13 weeks. Access to health care is necessary to prevent pregnancy or potential complications.

7. When evaluating a patient whose primary complaint is amenorrhea, the nurse must be aware that lack of menstruation is most often the result of: A. stress. B. excessive exercise. C. pregnancy. D. eating disorders.

ANS: C Amenorrhea, or the absence of menstrual flow, is most often a result of pregnancy. Although stress, excessive exercise, and eating disorders all may be contributing factors, none is the most common factor associated with amenorrhea.

14. A woman arrives at the clinic for her annual examination. She tells the nurse that she thinks she has a vaginal infection and has been using an over-the-counter cream for the past 2 days to treat it. The nurse's initial response should be to: A. inform the woman that vaginal creams may interfere with the Papanicolaou (Pap) test for which she is scheduled. B. reassure the woman that using vaginal cream is not a problem for the examination. C. ask the woman to describe the symptoms that indicate to her that she has a vaginal infection. D. ask the woman to reschedule the appointment for the examination.

ANS: C An important element of the history and physical examination is the patient's description of any symptoms she may be experiencing. Although vaginal creams may interfere with the Pap test, the best response is for the nurse to inquire about the symptoms the patient is experiencing. Women should not douche, use vaginal medications, or have sexual intercourse for 24 to 48 hours before obtaining a Pap test. Although the woman may need to reschedule a visit for her Pap test, her current symptoms should still be addressed.

2. The nurse is performing an initial assessment of a client in labor. What is the appropriate terminology for the relationship of the fetal body parts to one another? a. Lie b. Presentation c. Attitude d. Position

ANS: C Attitude is the relationship of the fetal body parts to one another. Lie is the relationship of the long axis (spine) of the fetus to the long axis (spine) of the mother. Presentation refers to the part of the fetus that enters the pelvic inlet first and leads through the birth canal during labor at term. Position is the relationship of the presenting part of the fetus to the four quadrants of the mother's pelvis.

27. The priority nursing care associated with an oxytocin (Pitocin) infusion is: a. measuring urinary output. b. increasing infusion rate every 30 minutes. c. monitoring uterine response. d. evaluating cervical dilation.

ANS: C Because of the risk of hyperstimulation, which could result in decreased placental perfusion and uterine rupture, the nurse's priority intervention is monitoring uterine response. Monitoring urinary output is also important; however, it is not the top priority during the administration of Pitocin. The infusion rate may be increased after proper assessment that it is an appropriate interval to do so. Monitoring labor progression is the standard of care for all labor patients.

19. With regard to the initial visit with a patient who is beginning prenatal care, nurses should be aware that: A. the first interview is a relaxed, get-acquainted affair in which nurses gather some general impressions. B. if nurses observe handicapping conditions, they should be sensitive and not enquire about them because the patient will do that in her own time. C. nurses should be alert to the appearance of potential parenting problems, such as depression or lack of family support. D. because of legal complications, nurses should not ask about illegal drug use; that is left to physicians.

ANS: C Besides these potential problems, nurses need to be alert to the woman's attitude toward health care. The initial interview needs to be planned, purposeful, and focused on specific content. A lot of ground must be covered. Nurses must be sensitive to special problems, but they do need to inquire because discovering individual needs is important. People with chronic or handicapping conditions forget to mention them because they have adapted to them. Getting information on drug use is important and can be done confidentially. Actual testing for drug use requires the patient's consent.

17. When evaluating a patient for sexually transmitted infections (STIs), the nurse should be aware that the most common bacterial STI is: a. gonorrhea. b. syphilis. c. chlamydia. d. candidiasis.

ANS: C Chlamydia is the most common and fastest spreading STI among American women, with an estimated 3 million new cases each year. Gonorrhea and syphilis are bacterial STIs, but they are not the most common ones among American women. Candidiasis is caused by a fungus, not by bacteria.

9. A nulliparous woman who has just begun the second stage of her labor would most likely: a. experience a strong urge to bear down. b. show perineal bulging. c. feel tired yet relieved that the worst is over. d. show an increase in bright red bloody show.

ANS: C Common maternal behaviors during the latent phase of the second stage of labor include feeling a sense of accomplishment and optimism because "the worst is over." During the latent phase of the second stage of labor, the urge to bear down often is absent or only slight during the acme of contractions. Perineal bulging occurs during the transition phase of the second stage of labor, not at the beginning of the second stage. An increase in bright red bloody show occurs during the descent phase of the second stage of labor.

28. As a girl progresses through development, she may be at risk for a number of age-related conditions. While preparing a 21-year-old patient for her first adult physical examination and Papanicolaou (Pap) test, the nurse is aware of excessiveness shyness. The young woman states that she will not remove her bra because, "There is something wrong with my breasts; one is way bigger." What is the best response by the nurse in this situation? A. "Please reschedule your appointment until you are more prepared." B. "It is okay; the provider will not do a breast examination." C. "I will explain normal growth and breast development to you." D. "That is unfortunate; this must be very stressful for you."

ANS: C During adolescence, one breast may grow faster than the other. Discussion regarding this aspect of growth and development with the patient will reassure her that there may be nothing wrong with her breasts. Young women usually enter the health system for screening (Pap tests begin at age 21 or 3 years after first sexual activity). Situations such as these can produce great stress for the young woman, and the nurse and health care provider should treat her carefully. Asking her to reschedule would likely result in the patient's not returning for her appointment at all. A breast examination at her age is part of the complete physical examination. Young women should be taught about normal breast development and begin doing breast self-examinations. Although the last response shows empathy on the part of the nurse and acknowledges the patient's stress, it does not correct the patient's deficient knowledge related to normal growth and development.

10. The nurse knows that the second stage of labor, the descent phase, has begun when: a. the amniotic membranes rupture. b. the cervix cannot be felt during a vaginal examination. c. the woman experiences a strong urge to bear down. d. the presenting part is below the ischial spines.

ANS: C During the descent phase of the second stage of labor, the woman may experience an increase in the urge to bear down. Rupture of membranes has no significance in determining the stage of labor. The second stage of labor begins with full cervical dilation. Many women may have an urge to bear down when the presenting part is below the level of the ischial spines. This can occur during the first stage of labor, as early as 5-cm dilation.

23. Some pregnant patients may complain of changes in their voice and impaired hearing. The nurse can tell these patients that these are common reactions to: A. a decreased estrogen level. B. displacement of the diaphragm, resulting in thoracic breathing. C. congestion and swelling, which occur because the upper respiratory tract has become more vascular. D. increased blood volume.

ANS: C Estrogen levels increase, causing the upper respiratory tract to become more vascular producing swelling and congestion in the nose and ears leading to voice changes and impaired hearing. The diaphragm is displaced, and the volume of blood is increased. However, the main concern is increased estrogen levels.

3. When assessing the fetus using Leopold's maneuvers, the nurse feels a round, firm, and movable fetal part in the fundal portion of the uterus and a long, smooth surface in the mother's right side close to midline. What is the position of the fetus? a. ROA b. LSP c. RSA d. LOA

ANS: C Fetal position is denoted with a three-letter abbreviation. The first letter indicates the presenting part in either the right or the left side of the maternal pelvis. The second letter indicates the anatomic presenting part of the fetus. The third letter stands for the location of the presenting part in relationship to the anterior, posterior, or transverse portion of the maternal pelvis. Palpation of a round, firm fetal part in the fundal portion of the uterus would be the fetal head, indicating that the fetus is in a breech position with the sacrum as the presenting part in the maternal pelvis. Palpation of the fetal spine along the mother's right side denotes the location of the presenting part in the mother's pelvis. The ability to palpate the fetal spine indicates that the fetus is anteriorly positioned in the maternal pelvis. This fetus is anteriorly positioned in the right side of the maternal pelvis with the sacrum as the presenting part. RSA is the correct three-letter abbreviation to indicate this fetal position. ROA denotes a fetus that is anteriorly positioned in the right side of the maternal pelvis with the occiput as the presenting part. LSP describes a fetus that is posteriorly positioned in the left side of the pelvis with the sacrum as the presenting part. A fetus that is LOA would be anteriorly positioned in the left side of the pelvis with the occiput as the presenting part.

40. The nurse providing education regarding breast care should explain to the woman that fibrocystic changes in breasts are: A. a disease of the milk ducts and glands in the breasts. B. a premalignant disorder characterized by lumps found in the breast tissue. C. lumpiness with or without tenderness found in varying degrees in the breast tissue of healthy women during menstrual cycles. D. lumpiness accompanied by tenderness after menses.

ANS: C Fibrocystic changes are palpable thickenings in the breast usually associated with or without tenderness. The pain and tenderness fluctuate with the menstrual cycle. Fibrocystic changes are not premalignant changes.

2. Which meal would provide the most absorbable iron? A. Toasted cheese sandwich, celery sticks, tomato slices, and a grape drink B. Oatmeal, whole wheat toast, jelly, and low-fat milk C. Black bean soup, wheat crackers, orange sections, and prunes D. Red beans and rice, cornbread, mixed greens, and decaffeinated tea

ANS: C Food sources that are rich in iron include liver, meats, whole grain or enriched breads and cereals, deep green leafy vegetables, legumes, and dried fruits. In addition, the vitamin C in orange sections aids absorption. Dairy products and tea are not sources of iron.

12. A woman's position is an important component of the labor progress. Which guidance is important for the nurse to provide to the laboring client? a. The supine position, which is commonly used in the United States, increases blood flow. b. The laboring client positioned on her hands and knees ("all fours" position) is hard on the woman's back. c. Frequent changes in position help relieve fatigue and increase the comfort of the laboring client. d. In a sitting or squatting position, abdominal muscles of the laboring client will have to work harder.

ANS: C Frequent position changes relieve fatigue, increase comfort, and improve circulation. Blood flow can be compromised in the supine position; any upright position benefits cardiac output. The "all fours" position is used to relieve backache in certain situations. In a sitting or squatting position, the abdominal muscles work in greater harmony with uterine contractions.

2. The nurse is providing genetic counseling for an expectant couple who already have a child with trisomy 18. The nurse should: A. tell the couple they need to have an abortion within 2 to 3 weeks. B. explain that the fetus has a 50% chance of having the disorder. C. discuss options with the couple, including amniocentesis to determine whether the fetus is affected. D. refer the couple to a psychologist for emotional support.

ANS: C Genetic testing, including amniocentesis, would need to be performed to determine whether the fetus is affected. The couple should be given information about the likelihood of having another baby with this disorder so that they can make an informed decision. A genetic counselor is the best source for determining genetic probability ratios. The couple eventually may need emotional support, but the status of the pregnancy must be determined first.

27. In response to requests by the U.S. Public Health Service for new models of prenatal care, an innovative new approach to prenatal care known as centering pregnancy was developed. Which statement would accurately apply to the centering model of care? A. Group sessions begin with the first prenatal visit. B. At each visit, blood pressure, weight, and urine dipsticks are obtained by the nurse. C. Eight to twelve women are placed in gestational-age cohort groups. D. Outcomes are similar to those of traditional prenatal care.

ANS: C Gestational-age cohorts comprise the groups with approximately 8 to 12 women in each group. This group remains intact throughout the pregnancy. Individual follow-up visits are scheduled as needed. Group sessions begin at 12 to 16 weeks of gestation and end with an early after birth visit. Before group sessions the patient has an individual assessment, physical examination, and history. At the beginning of each group meeting, patients measure their own blood pressure, weight, and urine dips and enter these in their record. Fetal heart rate assessment and fundal height are obtained by the nurse. Results evaluating this approach have been very promising. In a study of adolescent patients, there was a decrease in low-birth-weight infants and an increase in breastfeeding rates.

41. The nurse thoroughly dries the infant immediately after birth primarily to: a. stimulate crying and lung expansion. b. remove maternal blood from the skin surface. c. reduce heat loss from evaporation. d. increase blood supply to the hands and feet.

ANS: C Infants are wet with amniotic fluid and blood at birth, and this accelerates evaporative heat loss. The primary purpose of drying the infant is to prevent heat loss. Rubbing the infant does stimulate crying; however, it is not the main reason for drying the infant. This process does not remove all the maternal blood.

29. Nursing intervention for the pregnant diabetic patient is based on the knowledge that the need for insulin: A. increases throughout pregnancy and the after birth period. B. decreases throughout pregnancy and the after birth period. C. varies depending on the stage of gestation. D. should not change because the fetus produces its own insulin.

ANS: C Insulin needs decrease during the first trimester, when nausea, vomiting, and anorexia are a factor. They increase during the second and third trimesters, when the hormones of pregnancy create insulin resistance in maternal cells. Insulin needs increase during the second and third trimesters, when the hormones of pregnancy create insulin resistance in maternal cells. The insulin needs change throughout the different stages of pregnancy.

35. A woman has a breast mass that is not well delineated and is nonpalpable, immobile, and nontender. This is most likely: A. fibroadenoma. B. lipoma. C. intraductal papilloma. D. mammary duct ectasia.

ANS: C Intraductal papilloma is the only benign breast mass that is nonpalpable. Fibroadenoma is well delineated, palpable, and movable. Lipoma is palpable and movable. Mammary duct ectasia is not well delineated and is immobile, but it is palpable and painful.

18. Which minerals and vitamins usually are recommended to supplement a pregnant woman's diet? A. Fat-soluble vitamins A and D B. Water-soluble vitamins C and B6 C. Iron and folate D. Calcium and zinc

ANS: C Iron generally should be supplemented, and folic acid supplements often are needed because folate is so important. Fat-soluble vitamins should be supplemented as a medical prescription, as vitamin D might be for lactose-intolerant women. Water-soluble vitamin C sometimes is consumed in excess naturally; vitamin B6 is prescribed only if the woman has a very poor diet. Zinc sometimes is supplemented. Most women obtain enough calcium through their regular diet.

11. Prepidil (prostaglandin gel) has been ordered for a pregnant woman at 43 weeks of gestation. The nurse recognizes that this medication will be administered to: a. enhance uteroplacental perfusion in an aging placenta. b. increase amniotic fluid volume. c. ripen the cervix in preparation for labor induction. d. stimulate the amniotic membranes to rupture.

ANS: C It is accurate to state that Prepidil will be administered to ripen the cervix in preparation for labor induction. It is not administered to enhance uteroplacental perfusion in an aging placenta, increase amniotic fluid volume, or stimulate the amniotic membranes to rupture.

20. To reassure and educate pregnant patients about changes in their breasts, nurses should be aware that: A. the visibility of blood vessels that form an intertwining blue network indicates full function of Montgomery's tubercles and possibly infection of the tubercles. B. the mammary glands do not develop until 2 weeks before labor. C. lactation is inhibited until the estrogen level declines after birth. D. colostrum is the yellowish oily substance used to lubricate the nipples for breastfeeding.

ANS: C Lactation is inhibited until after birth. The visible blue network of blood vessels is a normal outgrowth of a richer blood supply. The mammary glands are functionally complete by midpregnancy. Colostrum is a creamy, white-to-yellow premilk fluid that can be expressed from the nipples before birth.

16. With regard to a woman's reordering of personal relationships during pregnancy, the maternity nurse should understand that: A. because of the special motherhood bond, a woman's relationship with her mother is even more important than with the father of the child. B. nurses need not get involved in any sexual issues the couple has during pregnancy, particularly if they have trouble communicating them to each other. C. women usually express two major relationship needs during pregnancy: feeling loved and valued and having the child accepted by the father. D. the woman's sexual desire is likely to be highest in the first trimester because of the excitement and because intercourse is physically easier.

ANS: C Love and support help a woman feel better about her pregnancy. The most important person to the pregnant woman is usually the father. Nurses can facilitate communication between partners about sexual matters if, as is common, they are nervous about expressing their worries and feelings. The second trimester is the time when a woman's sense of well-being, along with certain physical changes, increases her desire for sex. Desire is decreased in the first and third trimesters.

7. The nurse caring for the woman in labor should understand that maternal hypotension can result in: a. early decelerations. b. fetal dysrhythmias. c. uteroplacental insufficiency. d. spontaneous rupture of membranes.

ANS: C Low maternal blood pressure reduces placental blood flow during uterine contractions and results in fetal hypoxemia. Maternal

8. Prostaglandins are produced in most organs of the body, including the uterus. Other source(s) of prostaglandins is/are: A. ovaries. B. breast milk. C. menstrual blood. D. the vagina.

ANS: C Menstrual blood is a potent source of prostaglandins. Prostaglandins are produced in most organs of the body and in menstrual blood. The ovaries, breast milk, and vagina are neither organs nor a source of prostaglandins.

6. Menstruation is periodic uterine bleeding: A. that occurs every 28 days. B. in which the entire uterine lining is shed. C. that is regulated by ovarian hormones. D. that leads to fertilization.

ANS: C Menstruation is periodic uterine bleeding that is controlled by a feedback system involving three cycles: endometrial, hypothalamic-pituitary, and ovarian. The average length of a menstrual cycle is 28 days, but variations are normal. During the endometrial cycle, the functional two thirds of the endometrium are shed. Lack of fertilization leads to menstruation.

15. Management of primary dysmenorrhea often requires a multifaceted approach. The nurse who provides care for a patient with this condition should be aware that the optimal pharmacologic therapy for pain relief is: A. acetaminophen. B. oral contraceptives (OCPs). C. nonsteroidal anti-inflammatory drugs (NSAIDs). D. aspirin.

ANS: C NSAIDs are prostaglandin inhibitors and show the strongest research results for pain relief. Often if one NSAID is not effective, another one can provide relief. Approximately 80% of women find relief from NSAIDs. Preparations containing acetaminophen are less effective for dysmenorrhea because they lack the antiprostaglandin properties of NSAIDs. OCPs are a reasonable choice for women who also want birth control. The benefit of OCPs is the reduction of menstrual flow and irregularities. OCPs may be contraindicated for some women and have numerous potential side effects. NSAIDs are the drug of choice. If a woman is taking a NSAID, she should avoid taking aspirin.

22. While instructing a couple regarding birth control, the nurse should be aware that the method called natural family planning: A. is the same as coitus interruptus, or "pulling out." B. uses the calendar method to align the woman's cycle with the natural phases of the moon. C. is the only contraceptive practice acceptable to the Roman Catholic Church. D. relies on barrier methods during fertility phases.

ANS: C Natural family planning is another name for periodic abstinence, which is the accepted way to pass safely through the fertility phases without relying on chemical or physical barriers. Natural family planning is the only contraceptive practice acceptable to the Roman Catholic Church. "Pulling out" is not the same as periodic abstinence, another name for natural family planning. The phases of the moon are not part of the calendar method or any method.

13. The nurse providing care for the antepartum woman should understand that contraction stress test (CST): A. sometimes uses vibroacoustic stimulation. B. is an invasive test; however, contractions are stimulated. C. is considered negative if no late decelerations are observed with the contractions. D. is more effective than nonstress test (NST) if the membranes have already been ruptured.

ANS: C No late decelerations are good news. Vibroacoustic stimulation is sometimes used with NST. CST is invasive if stimulation is by intravenous oxytocin but not if by nipple stimulation and is contraindicated if the membranes have ruptured.

8. When nurses help their expectant mothers assess the daily fetal movement counts, they should be aware that: A. alcohol or cigarette smoke can irritate the fetus into greater activity. B. "kick counts" should be taken every half hour and averaged every 6 hours, with every other 6-hour stretch off. C. the fetal alarm signal should go off when fetal movements stop entirely for 12 hours. D. obese mothers familiar with their bodies can assess fetal movement as well as average-size women.

ANS: C No movement in a 12-hour period is cause for investigation and possibly intervention. Alcohol and cigarette smoke temporarily reduce fetal movement. The mother should count fetal activity ("kick counts") two or three times daily for 60 minutes each time. Obese women have a harder time assessing fetal movement.

11. Which statement concerning cyclic perimenstrual pain and discomfort (CPPD) is accurate? A. Premenstrual dysphoric disorder (PMDD) is a milder form of premenstrual syndrome (PMS) and more common in younger women. B. Secondary dysmenorrhea is more intense and medically significant than primary dysmenorrhea. C. Premenstrual syndrome is a complex, poorly understood condition that may include any of a hundred symptoms. D. The causes of PMS have been well established.

ANS: C PMS may manifest with one or more of a hundred or so physical and psychologic symptoms. PMDD is a more severe variant of PMS. Secondary dysmenorrhea is characterized by more muted pain than that seen in primary dysmenorrhea; the medical treatment is much the same. The cause of PMS is unknown. It may be a collection of different problems.

12. A patient at 24 weeks of gestation contacts the nurse at her obstetric provider's office to complain that she has cravings for dirt and gravel. The nurse is aware that this condition is known as ________ and may indicate anemia. A. ptyalism B. pyrosis C. pica D. decreased peristalsis

ANS: C Pica (a desire to eat nonfood substances) is an indication of iron deficiency and should be evaluated. Ptyalism (excessive salivation), pyrosis (heartburn), and decreased peristalsis are normal findings of gastrointestinal change during pregnancy. Food cravings during pregnancy are normal.

29. Surgical, medical, or mechanical methods may be used for labor induction. Which technique is considered a mechanical method of induction? a. Amniotomy b. Intravenous Pitocin c. Transcervical catheter d. Vaginal insertion of prostaglandins

ANS: C Placement of a balloon-tipped Foley catheter into the cervix is a mechanical method of induction. Other methods to expand and gradually dilate the cervix include hydroscopic dilators such as laminaria tents (made from desiccated seaweed), or Lamicel (contains magnesium sulfate). Amniotomy is a surgical method of augmentation and induction.Intravenous Pitocin and insertion of prostaglandins are medical methods of induction.

18. Which heart condition is not a contraindication for pregnancy? A. Peripartum cardiomyopathy B. Eisenmenger syndrome C. Heart transplant D. All of these contraindicate pregnancy.

ANS: C Pregnancy is contraindicated for peripartum cardiomyopathy and Eisenmenger syndrome. Women who have had heart transplants are successfully having babies. However, conception should be postponed for at least 1 year after transplantation.

8. Metabolic changes throughout pregnancy that affect glucose and insulin in the mother and the fetus are complicated but important to understand. Nurses should understand that: A. insulin crosses the placenta to the fetus only in the first trimester, after which the fetus secretes its own. B. women with insulin-dependent diabetes are prone to hyperglycemia during the first trimester because they are consuming more sugar. C. during the second and third trimesters, pregnancy exerts a diabetogenic effect that ensures an abundant supply of glucose for the fetus. D. maternal insulin requirements steadily decline during pregnancy.

ANS: C Pregnant women develop increased insulin resistance during the second and third trimesters. Insulin never crosses the placenta; the fetus starts making its own insulin around the 10th week. As a result of normal metabolic changes during pregnancy, insulin-dependent women are prone to hypoglycemia (low levels). Maternal insulin requirements may double or quadruple by the end of pregnancy.

1. In planning for home care of a woman with preterm labor, which concern must the nurse address? a. Nursing assessments will be different from those done in the hospital setting. b. Restricted activity and medications will be necessary to prevent recurrence of preterm labor. c. Prolonged bed rest may cause negative physiologic effects. d. Home health care providers will be necessary.

ANS: C Prolonged bed rest may cause adverse effects such as weight loss, loss of appetite, muscle wasting, weakness, bone demineralization, decreased cardiac output, risk for thrombophlebitis, alteration in bowel functions, sleep disturbance, and prolonged after birth recovery. Nursing assessments will differ somewhat from those performed in the acute care setting, but this is not the concern that needs to be addressed. Restricted activity and medication may prevent preterm labor, but not in all women. In addition, the plan of care is individualized to meet the needs of each woman. Many women will receive home health nurse visits, but care is individualized for each woman.

4. Which statement by the client would lead the nurse to believe that labor has been established? a. "I passed some thick, pink mucus when I urinated this morning." b. "My bag of waters just broke." c. "The contractions in my uterus are getting stronger and closer together." d. "My baby dropped, and I have to urinate more frequently now."

ANS: C Regular, strong contractions with the presence of cervical change indicate that the woman is experiencing true labor. Although the loss of the mucous plug (operculum) often occurs during the first stage of labor or before the onset of labor, it is not the indicator of true labor. Spontaneous rupture of membranes often occurs during the first stage of labor; however, it is not an indicator of true labor. The presenting part of the fetus typically becomes engaged in the pelvis at the onset of labor but is not the indicator of true labor.

1. The nurse recognizes that a woman is in true labor when she states: a. "I passed some thick, pink mucus when I urinated this morning." b. "My bag of waters just broke." c. "The contractions in my uterus are getting stronger and closer together." d. "My baby dropped, and I have to urinate more frequently now."

ANS: C Regular, strong contractions with the presence of cervical change indicate that the woman is experiencing true labor. Loss of the mucous plug (operculum) often occurs during the first stage of labor or before the onset of labor, but it is not the indicator of true labor. Spontaneous rupture of membranes often occurs during the first stage of labor, but it is not the indicator of true labor. The presenting part of the fetus typically becomes engaged in the pelvis at the onset of labor, but this is not the indicator of true labor.

30. A pregnant woman who abuses cocaine admits to exchanging sex for her drug habit. This behavior places her at a greater risk for: A. depression of the central nervous system. B. hypotension and vasodilation. C. sexually transmitted diseases. D. postmature birth.

ANS: C Sex acts exchanged for drugs places the woman at increased risk for sexually transmitted diseases because of multiple partners and lack of protection. Cocaine is a central nervous system stimulant that causes hypertension and vasoconstriction. Premature delivery of the infant is one of the most common problems associated with cocaine use during pregnancy.

40. At 1 minute after birth, the nurse assesses the newborn to assign an Apgar score. The apical heart rate is 110 bpm, and the infant is crying vigorously with the limbs flexed. The infant's trunk is pink, but the hands and feet are blue. What is the correct Apgar score for this infant? a. 7 b. 8 c. 9 d. 10

ANS: C The Apgar score is 9 because 1 point is deducted from the total score of 10 for the infant's blue hands and feet. The baby received 2 points for each of the categories except color. Because the infant's hands and feet were blue, this category is given a grade of 1.

19. According to standard professional thinking, nurses should auscultate the fetal heart rate (FHR): a. every 15 minutes in the active phase of the first stage of labor in the absence of risk factors. b. every 20 minutes in the second stage, regardless of whether risk factors are present. c. before and after ambulation and rupture of membranes. d. more often in a woman's first pregnancy.

ANS: C The FHR should be auscultated before and after administration of medications and induction of anesthesia. In the active phase of the first stage of labor, the FHR should be auscultated every 30 minutes if no risk factors are involved; with risk factors it should be auscultated every 15 minutes. In the second stage of labor the FHR should be auscultated every 15 minutes if no risk factors are involved; with risk factors it should be auscultated every 5 minutes. The fetus of a first-time mother is automatically at greater risk.

27. The microscopic examination of scrapings from the cervix, endocervix, or other mucous membranes to detect premalignant or malignant cells is called: A. bimanual palpation. B. rectovaginal palpation. C. a Papanicolaou (Pap) test. D. a four As procedure.

ANS: C The Pap test is a microscopic examination for cancer that should be performed regularly, depending on the patient's age. Bimanual palpation is a physical examination of the vagina. Rectovaginal palpation is a physical examination performed through the rectum. The four As is an intervention procedure to help a patient stop smoking.

A. 1. A father and mother are carriers of phenylketonuria (PKU). Their 2-year-old daughter has PKU. The couple tells the nurse that they are planning to have a second baby. Because their daughter has PKU, they are sure that their next baby won't be affected. What response by the nurse is most accurate? A. "Good planning; you need to take advantage of the odds in your favor." B. "I think you'd better check with your doctor first." C. "You are both carriers, so each baby has a 25% chance of being affected." D. "The ultrasound indicates a boy, and boys are not affected by PKU."

ANS: C The chance is one in four that each child produced by this couple will be affected by PKU disorder. This couple still has an increased likelihood of having a child with PKU. Having one child already with PKU does not guarantee that they will not have another. These parents need to discuss their options with their physician. However, an opportune time has presented itself for the couple to receive correct teaching about inherited genetic risks. No correlation exists between gender and inheritance of the disorder because PKU is an autosomal recessive disorder.

27. Which description of the phases of the second stage of labor is accurate? a. Latent phase: Feeling sleepy, fetal station 2+ to 4+, duration 30 to 45 minutes b. Active phase: Overwhelmingly strong contractions, Ferguson reflux activated, duration 5 to 15 minutes c. Descent phase: Significant increase in contractions, Ferguson reflux activated, average duration varied d. Transitional phase: Woman "laboring down," fetal station 0, duration 15 minutes

ANS: C The descent phase begins with a significant increase in contractions; the Ferguson reflex is activated, and the duration varies, depending on a number of factors. The latent phase is the lull, or "laboring down," period at the beginning of the second stage. It lasts 10 to 30 minutes on average. The second stage of labor has no active phase. The transition phase is the final phase in the second stage of labor; contractions are strong and painful.

22. The most common neurologic disorder accompanying pregnancy is: A. eclampsia. B. Bell's palsy. C. epilepsy. D. multiple sclerosis.

ANS: C The effects of pregnancy on epilepsy are unpredictable. Eclampsia sometimes may be confused with epilepsy, which is the most common neurologic disorder accompanying pregnancy. Bell's palsy is a form of facial paralysis. Multiple sclerosis is a patchy demyelinization of the spinal cord that does not affect the normal course of pregnancy or birth.

22. Which statement regarding female sexual response is inaccurate? A. Women and men are more alike than different in their physiologic response to sexual arousal and orgasm. B. Vasocongestion is the congestion of blood vessels. C. The orgasmic phase is the final state of the sexual response cycle. D. Facial grimaces and spasms of hands and feet are often part of arousal.

ANS: C The final state of the sexual response cycle is the resolution phase after orgasm. Men and women are surprisingly alike. Vasocongestion causes vaginal lubrication and engorgement of the genitals. Arousal is characterized by increased muscular tension (myotonia).

17. The least common cause of long, difficult, or abnormal labor (dystocia) is: a. midplane contracture of the pelvis. b. compromised bearing-down efforts as a result of pain medication. c. disproportion of the pelvis. d. low-lying placenta.

ANS: C The least common cause of dystocia is disproportion of the pelvis.

8. A 36-year-old woman has been given a diagnosis of uterine fibroids. When planning care for this patient, the nurse should know that: A. fibroids are malignant tumors of the uterus that require radiation or chemotherapy. B. fibroids increase in size during the perimenopausal period. C. menorrhagia is a common finding. D. the woman is unlikely to become pregnant as long as the fibroids are in her uterus.

ANS: C The major symptoms associated with fibroids are menorrhagia and the physical effects produced by large myomas. Fibroids are benign tumors of the smooth muscle of the uterus, and their etiology is unknown. Fibroids are estrogen sensitive and shrink as levels of estrogen decline. Fibroids occur in 25% of women of reproductive age and are seen in 2% of pregnant women.

4. A woman in labor has just received an epidural block. The most important nursing intervention is to: a. limit parenteral fluids. b. monitor the fetus for possible tachycardia. c. monitor the maternal blood pressure for possible hypotension. d. monitor the maternal pulse for possible bradycardia.

ANS: C The most important nursing intervention for a woman who has received an epidural block is to monitor the maternal blood pressure frequently for signs of hypotension. Intravenous fluids are increased for a woman receiving an epidural, to prevent hypotension. The nurse observes for signs of fetal bradycardia. The nurse monitors for signs of maternal tachycardia secondary to hypotension.

11. A labor and delivery nurse should be cognizant of which information regarding how the fetus moves through the birth canal? a. Fetal attitude describes the angle at which the fetus exits the uterus. b. Of the two primary fetal lies, the horizontal lie is that in which the long axis of the fetus is parallel to the long axis of the mother. c. Normal attitude of the fetus is called general flexion. d. Transverse lie is preferred for vaginal birth.

ANS: C The normal attitude of the fetus is called general flexion. The fetal attitude is the relationship of the fetal body parts to each one another. The horizontal lie is perpendicular to the mother; in the longitudinal (or vertical) lie, the long axes of the fetus and the mother are parallel. Vaginal birth cannot occur if the fetus stays in a transverse lie.

5. Which action is correct when palpation is used to assess the characteristics and pattern of uterine contractions? a. Place the hand on the abdomen below the umbilicus and palpate uterine tone with the fingertips. b. Determine the frequency by timing from the end of one contraction to the end of the next contraction. c. Evaluate the intensity by pressing the fingertips into the uterine fundus. d. Assess uterine contractions every 30 minutes throughout the first stage of labor.

ANS: C The nurse or primary care provider may assess uterine activity by palpating the fundal section of the uterus using the fingertips. Many women may experience labor pain in the lower segment of the uterus that may be unrelated to the firmness of the contraction detectable in the uterine fundus. The frequency of uterine contractions is determined by palpating from the beginning of one contraction to the beginning of the next contraction. Assessment of uterine activity is performed in intervals based on the stage of labor. As labor progresses this assessment is performed more frequently.

20. Sally comes in for her first prenatal examination. This is her first child. She asks you (the nurse), "How does my baby get air inside my uterus?" The correct response is: A. "The baby's lungs work in utero to exchange oxygen and carbon dioxide." B. "The baby absorbs oxygen from your blood system." C. "The placenta provides oxygen to the baby and excretes carbon dioxide into your bloodstream." D. "The placenta delivers oxygen-rich blood through the umbilical artery to the baby's abdomen."

ANS: C The placenta functions by supplying oxygen and excreting carbon dioxide to the maternal bloodstream. The fetal lungs do not function for respiratory gas exchange in utero. The baby does not simply absorb oxygen from a woman's blood system. Blood and gas transport occur through the placenta. The placenta delivers oxygen-rich blood through the umbilical vein and not the artery.

29. When providing care to the prenatal patient, the nurse understands that pica is defined as: A. intolerance of milk products. B. iron deficiency anemia. C. ingestion of nonfood substances. D. episodes of anorexia and vomiting.

ANS: C The practice of eating substances not normally thought of as food is called pica. Clay or dirt and solid laundry starch are the substances most commonly ingested. Intolerance of milk products is referred to as lactose intolerance. Pica may produce iron deficiency anemia if proper nutrition is decreased. Pica is not related to anorexia and vomiting.

22. To provide the patient with accurate information about dental care during pregnancy, maternity nurses should be aware that: A. dental care can be dropped from the priority list because the woman has enough to worry about and is getting a lot of calcium anyway. B. dental surgery, in particular, is contraindicated because of the psychologic stress it engenders. C. if dental treatment is necessary, the woman will be most comfortable with it in the second trimester. D. dental care interferes with the expectant mother's need to practice conscious relaxation.

ANS: C The second trimester is best for dental treatment because that is when the woman will be able to sit most comfortably in the dental chair. Dental care such as brushing with fluoride toothpaste is especially important during pregnancy because nausea during pregnancy may lead to poor oral hygiene. Emergency dental surgery is permissible, but the mother must clearly understand the risks and benefits. Conscious relaxation is useful, and it may even help the woman get through any dental appointments; it is not a reason to avoid them.

19. A fully matured endometrium that has reached the thickness of heavy, soft velvet describes the _____ phase of the endometrial cycle. a. menstrual b. proliferative c. secretory d. ischemic

ANS: C The secretory phase extends from the day of ovulation to approximately 3 days before the next menstrual cycle. During this phase, the endometrium becomes fully mature. During the menstrual phase, the endometrium is being shed; the endometrium is fully mature again during the secretory phase. The proliferative phase is a period of rapid growth, but the endometrium becomes fully mature again during the secretory phase. During the ischemic phase, the blood supply is blocked, and necrosis develops. The endometrium is fully mature during the secretory phase.

15. Nurses should be cognizant of what regarding the mechanism of labor? a. Seven critical movements must progress in a more or less orderly sequence. b. Asynclitism is sometimes achieved by means of the Leopold's maneuver. c. Effects of the forces determining descent are modified by the shape of the woman's pelvis and the size of the fetal head. d. At birth, the baby is said to achieve "restitution"; that is, a return to the C-shape of the womb.

ANS: C The size of the maternal pelvis and the ability of the fetal head to mold also affect the process. The seven identifiable movements of the mechanism of labor simultaneously occur in combinations, not in precise sequences. Asynclitism is the deflection of the baby's head; the Leopold's maneuver is a means of judging descent by palpating the mother's abdomen. Restitution is the rotation of the baby's head after the infant is born.

22. What is an advantage of external electronic fetal monitoring? a. The ultrasound transducer can accurately measure short-term variability and beat-to-beat changes in the fetal heart rate. b. The tocotransducer can measure and record the frequency, regularity, intensity, and approximate duration of uterine contractions (UCs). c. The tocotransducer is especially valuable for measuring uterine activity during the first stage of labor. d. Once correctly applied by the nurse, the transducer need not be repositioned even when the woman changes positions.

ANS: C The tocotransducer is especially valuable for measuring uterine activity during the first stage of labor, particularly when the membranes are intact. Short-term changes cannot be measured with this technology. The tocotransducer cannot measure and record the intensity of UCs. The transducer must be repositioned when the woman or fetus changes position.

2. A couple comes in for an infertility workup, having attempted to get pregnant for 2 years. The woman, 37 years, has always had irregular menstrual cycles but is otherwise healthy. The man has fathered two children from a previous marriage and had a vasectomy reversal 2 years ago. The man has had two normal semen analyses, but the sperm seem to be clumped together. What additional test is needed? A. Testicular biopsy B. Antisperm antibodies C. Follicle-stimulating hormone (FSH) level D. Examination for testicular infection

ANS: C The woman has irregular menstrual cycles. The scenario does not indicate that she has had any testing related to this irregularity. Hormone analysis is performed to assess endocrine function of the hypothalamic-pituitary-ovarian axis when menstrual cycles are absent or irregular. Determination of blood levels of prolactin, FSH, luteinizing hormone (LH), estradiol, progesterone, and thyroid hormones may be necessary to diagnose the cause of irregular menstrual cycles. A testicular biopsy would be indicated only in cases of azoospermia (no sperm cells) or severe oligospermia (low number of sperm cells). Antisperm antibodies are produced by a man against his own sperms. This is unlikely to be the case here because the man has already produced children. Examination for testicular infection would be done before semen analysis. Infection would affect spermatogenesis.

20. With regard to follow-up visits for women receiving prenatal care, nurses should be aware that: A. the interview portions become more intensive as the visits become more frequent over the course of the pregnancy. B. monthly visits are scheduled for the first trimester, every 2 weeks for the second trimester, and weekly for the third trimester. C. during the abdominal examination, the nurse should be alert for supine hypotension. D. for pregnant women, a systolic blood pressure (BP) of 130 and a diastolic BP of 80 is sufficient to be considered hypertensive.

ANS: C The woman lies on her back during the abdominal examination, possibly compressing the vena cava and aorta, which can cause a decrease in blood pressure and a feeling of faintness. The interview portion of follow-up examinations is less extensive than in the initial prenatal visits, during which so much new information must be gathered. Monthly visits are routinely scheduled for the first and second trimesters; visits increase to every 2 weeks at week 28 and to once a week at week 36. For pregnant women hypertension is defined as a systolic BP of 140 or greater and a diastolic BP of 90 or greater.

32. After a mastectomy, a woman should be instructed to perform all of the following except: A. emptying surgical drains twice a day and as needed. B. avoiding lifting more than 4.5 kg (10 lbs) or reaching above her head until given permission by her surgeon. C. wearing clothing with snug sleeves to support the tissue of the arm on the operative side. D. reporting immediately if inflammation develops at the incision site or in the affected arm.

ANS: C The woman should not be advised to wear snug clothing. Rather, she should be advised to avoid tight clothing, tight jewelry, and other causes of decreased circulation in the affected arm. As part of the teaching plan, the woman should be instructed to empty surgical drains, to avoid lifting more than 10 lbs or reaching above her head, and to report the development of incision site inflammation.

25. A patient at 24 weeks of gestation says she has a glass of wine with dinner every evening. The nurse will counsel her to eliminate all alcohol intake because: A. a daily consumption of alcohol indicates a risk for alcoholism. B. she will be at risk for abusing other substances as well. C. the fetus is placed at risk for altered brain growth. D. the fetus is at risk for multiple organ anomalies.

ANS: C There is no period during pregnancy when it is safe to consume alcohol. The documented effects of alcohol consumption during pregnancy include intellectual disability, learning disabilities, high activity level, and short attention span. The bra in grows most rapidly in the third trimester and is vulnerable to alcohol exposure during this time. Abuse of other substances has not been linked to alcohol use.

14. At approximately _____ weeks of gestation, lecithin is forming on the alveolar surfaces, the eyelids open, and the fetus measures approximately 27 cm crown to rump and weighs approximately 1110 g. a. 20 b. 24 c. 28 d. 30

ANS: C These milestones human development occur at approximately 28 weeks.

3. A pregnant woman's diet consists almost entirely of whole grain breads and cereals, fruits, and vegetables. The nurse would be most concerned about this woman's intake of: A. calcium. B. protein. C. vitamin B12. D. folic acid.

ANS: C This diet is consistent with that followed by a strict vegetarian (vegan). Vegans consume only plant products. Because vitamin B12 is found in foods of animal origin, this diet is deficient in vitamin B12.

11. The nurse guides a woman to the examination room and asks her to remove her clothes and put on an examination gown with the front open. The woman states, "I have special undergarments that I do not remove for religious reasons." The most appropriate response from the nurse would be: A. "You can't have an examination without removing all your clothes." B. "I'll ask the doctor to modify the examination." C. "Tell me about your undergarments. I'll explain the examination procedure, and then we can discuss how you can have your examination comfortably." D. "What? I've never heard of such a thing! That sounds different and strange."

ANS: C This statement reflects cultural competence by the nurse and shows respect for the woman's religious practices. The nurse must respect the rich and unique qualities that cultural diversity brings to individuals. In recognizing the value of these differences, the nurse can modify the plan of care to meet the needs of each woman.

8. A 27-year-old pregnant woman had a preconceptual body mass index (BMI) of 18.0. The nurse knows that this woman's total recommended weight gain during pregnancy should be at least: A. 20 kg (44 lbs). B. 16 kg (35 lbs). C. 12.5 kg (27.5 lbs). D. 10 kg (22 lbs).

ANS: C This woman has a normal BMI and should gain 11.5 to 16 kg during pregnancy. A weight gain of 20 kg would be unhealthy for most women. A weight gain 35 lbs is the high end of the range of weight this woman should gain in her pregnancy. A weight gain of 22 lbs would be appropriate for an obese woman.

19. A woman will be taking oral contraceptives using a 28-day pack. The nurse should advise this woman to protect against pregnancy by: A. limiting sexual contact for one cycle after starting the pill. B. using condoms and foam instead of the pill for as long as she takes an antibiotic. C. taking one pill at the same time every day. D. throwing away the pack and using a backup method if she misses two pills during Week 1 of her cycle.

ANS: C To maintain adequate hormone levels for contraception and to enhance compliance, patients should take oral contraceptives at the same time each day. If contraceptives are to be started at any time other than during normal menses or within 3 weeks after birth or abortion, another method of contraception should be used through the first week to prevent the risk of pregnancy. Taken exactly as directed, oral contraceptives prevent ovulation, and pregnancy cannot occur. No strong pharmacokinetic evidence indicates a link between the use of broad-spectrum antibiotics and altered hormone levels in oral contraceptive users. If the patient misses two pills during Week 1, she should take two pills a day for 2 days, finish the package, and use a backup method the next 7 consecutive days.

2. The nurse teaches a pregnant woman about the characteristics of true labor contractions. The nurse evaluates the woman's understanding of the instructions when she states, "True labor contractions will: a. subside when I walk around." b. cause discomfort over the top of my uterus." c. continue and get stronger even if I relax and take a shower." d. remain irregular but become stronger."

ANS: C True labor contractions occur regularly, becoming stronger, lasting longer, and occurring closer together. They may become intense during walking and continue despite comfort measures. Typically true labor contractions are felt in the lower back, radiating to the lower portion of the abdomen. During false labor, contractions tend to be irregular and felt in the abdomen above the navel. Typically the contractions often stop with walking or a change of position.

4. A pregnant woman at 10 weeks of gestation jogs three or four times per week. She is concerned about the effect of exercise on the fetus. The nurse should inform her: A. "You don't need to modify your exercising any time during your pregnancy." B. "Stop exercising because it will harm the fetus." C. "You may find that you need to modify your exercise to walking later in your pregnancy, around the seventh month." D. "Jogging is too hard on your joints; switch to walking now."

ANS: C Typically running should be replaced with walking around the seventh month of pregnancy. The nurse should inform the woman that she may need to reduce her exercise level as the pregnancy progresses. Physical activity promotes a feeling of well-being in pregnant women. It improves circulation, promotes relaxation and rest, and counteracts boredom. Simple measures should be initiated to prevent injuries, such as warm-up and stretching exercises to prepare the joints for more strenuous exercise.

20. Which adaptation of the maternal-fetal exchange of oxygen occurs in response to uterine contraction? a. The maternal-fetal exchange of oxygen and waste products continues except when placental functions are reduced. b. This maternal-fetal exchange increases as the blood pressure decreases. c. It diminishes as the spiral arteries are compressed. d. This exchange of oxygen and waste products is not significantly affected by contractions.

ANS: C Uterine contractions during labor tend to decrease circulation through the spiral electrodes and subsequent perfusion through the intervillous space. The maternal blood supply to the placenta gradually stops with contractions. The exchange of oxygen and waste products decreases. The exchange of oxygen and waste products is affected by contractions.

5. The nurse providing care for the laboring woman should understand that late fetal heart rate (FHR) decelerations are the result of: a. altered cerebral blood flow. b. umbilical cord compression. c. uteroplacental insufficiency. d. meconium fluid.

ANS: C Uteroplacental insufficiency would result in late decelerations in the FHR. Altered fetal cerebral blood flow would result in early decelerations in the FHR. Umbilical cord compression would result in variable decelerations in the FHR. Meconium-stained fluid may or may not produce changes in the fetal heart rate, depending on the gestational age of the fetus and whether other causative factors associated with fetal distress are present.

3. A couple is trying to cope with an infertility problem. They want to know what they can do to preserve their emotional equilibrium. The nurse's most appropriate response is: A. "Tell your friends and family so they can help you." B. "Talk only to other friends who are infertile because only they can help." C. "Get involved with a support group. I'll give you some names." D. "Start adoption proceedings immediately because it is very difficult to obtain an infant."

ANS: C Venting negative feelings may unburden the couple. A support group may provide a safe haven for the couple to share their experiences and gain insight from others' experiences. Although talking about their feelings may unburden them of negative feelings, infertility can be a major stressor that affects the couple's relationships with family and friends. Limiting their interactions to other infertile couples may be a beginning point for addressing psychosocial needs, but depending on where the other couple is in their own recovery process, this may or may not help them. The statement about adoption proceedings is not supportive of the psychosocial needs of this couple and may be detrimental to their well-being.

31. Which diagnostic test is used to confirm a suspected diagnosis of breast cancer? A. Mammogram B. Ultrasound C. Needle aspiration D. CA 15.3

ANS: C When a suspicious mammogram is noted or a lump is detected, diagnosis is confirmed by needle aspiration, core needle biopsy, or needle localization biopsy. Mammography is a clinical screening tool that may aid early detection of breast cancers. Transillumination, thermography, and ultrasound breast imaging are being explored as methods of detecting early breast carcinoma. CA 15.3 is a serum tumor marker that is used to test for residual disease.

8. A woman is having her first child. She has been in labor for 15 hours. Two hours ago her vaginal examination revealed the cervix to be dilated to 5 cm and 100% effaced, and the presenting part was at station 0. Five minutes ago her vaginal examination indicated that there had been no change. What abnormal labor pattern is associated with this description? a. Prolonged latent phase b. Protracted active phase c. Arrest of active phase d. Protracted descent

ANS: C With an arrest of the active phase, the progress of labor has stopped. This patient has not had any anticipated cervical change, thus indicating an arrest of labor. In the nulliparous woman a prolonged latent phase typically would last more than 20 hours. A protracted active phase, the first or second stage of labor, would be prolonged (slow dilation). With protracted descent, the fetus would fail to descend at an anticipated rate during the deceleration phase and second stage of labor.

6. A primigravida at 40 weeks of gestation is having uterine contractions every 1.5 to 2 minutes and says that they are very painful. Her cervix is dilated 2 cm and has not changed in 3 hours. The woman is crying and wants an epidural. What is the likely status of this woman's labor? a. She is exhibiting hypotonic uterine dysfunction. b. She is experiencing a normal latent stage. c. She is exhibiting hypertonic uterine dysfunction. d. She is experiencing pelvic dystocia.

ANS: C Women who experience hypertonic uterine dysfunction, or primary dysfunctional labor, often are anxious first-time mothers who are having painful and frequent contractions that are ineffective at causing cervical dilation or effacement to progress. With hypotonic uterine dysfunction, the woman initially makes normal progress into the active stage of labor; then the contractions become weak and inefficient or stop altogether. The contraction pattern seen in this woman signifies hypertonic uterine activity. Typically uterine activity in this phase occurs at 4- to 5-minute intervals lasting 30 to 45 seconds. Pelvic dystocia can occur whenever contractures of the pelvic diameters reduce the capacity of the bony pelvis, including the inlet, midpelvis, outlet, or any combination of these planes.

30. In caring for a pregnant woman with sickle cell anemia, the nurse is aware that signs and symptoms of sickle cell crisis include: A. anemia. B. endometritis. C. fever and pain. D. urinary tract infection.

ANS: C Women with sickle cell anemia have recurrent attacks (crisis) of fever and pain, most often in the abdomen, joints, and extremities. These attacks are attributed to vascular occlusion when RBCs assume the characteristic sickled shape. Crises are usually triggered by dehydration, hypoxia, or acidosis. Women with sickle cell anemia are not iron deficient. Therefore, routine iron supplementation, even that found in prenatal vitamins, should be avoided in order to prevent iron overload. Women with sickle cell trait usually are at greater risk for after birth endometritis (uterine wall infection); however, this is not likely to occur in pregnancy and is not a sign of crisis. These women are at an increased risk for UTIs; however, this is not an indication of sickle cell crisis.

2. During pregnancy, many changes occur as a direct result of the presence of the fetus. Which of these adaptations meet this criterion? (Select all that apply.) A. Leukorrhea B. Development of the operculum C. Quickening D. Ballottement E. Lightening

ANS: C, D, E Leukorrhea is a white or slightly gray vaginal discharge that develops in response to cervical stimulation by estrogen and progesterone. Quickening is the first recognition of fetal movements or "feeling life." Quickening is often described as a flutter and is felt earlier in multiparous women than in primiparas. Lightening occurs when the fetus begins to descend into the pelvis. This occurs 2 weeks before labor in the nullipara and at the start of labor in the multipara. Mucus fills the cervical canal creating a plug otherwise known as the operculum. The operculum acts as a barrier against bacterial invasion during the pregnancy. Passive movement of the unengaged fetus is referred to as ballottement.

2. The baseline fetal heart rate (FHR) is the average rate during a 10-minute segment. Changes in FHR are categorized as periodic or episodic. These patterns include both accelerations and decelerations. The labor nurse is evaluating the patient's most recent 10-minute segment on the monitor strip and notes a late deceleration. This is likely to be caused by which physiologic alteration? (Select all that apply.) a. Spontaneous fetal movement b. Compression of the fetal head c. Placental abruption d. Cord around the baby's neck e. Maternal supine hypotension

ANS: C,E Late decelerations are almost always caused by uteroplacental insufficiency. Insufficiency is caused by uterine tachysystole, maternal hypotension, epidural or spinal anesthesia, IUGR, intraamniotic infection, or placental abruption. Spontaneous fetal movement, vaginal examination, fetal scalp stimulation, fetal reaction to external sounds, uterine contractions, fundal pressure and abdominal palpation are all likely to cause accelerations of the FHR. Early decelerations are most often the result of fetal head compression and may be caused by uterine contractions, fundal pressure, vaginal examination, and placement of an internal electrode. A variable deceleration is likely caused by umbilical cord compression. This may happen when the umbilical cord is around the baby's neck, arm, leg, or other body part or when there is a short cord, a knot in the cord, or a prolapsed cord.

11. A woman with gestational diabetes has had little or no experience reading and interpreting glucose levels. She shows the nurse her readings for the past few days. Which one should the nurse tell her indicates a need for adjustment (insulin or sugar)? A. 75 mg/dL before lunch. This is low; better eat now. B. 115 mg/dL 1 hour after lunch. This is a little high; maybe eat a little less next time. C. 115 mg/dL 2 hours after lunch; This is too high; it is time for insulin. D. 60 mg/dL just after waking up from a nap. This is too low; maybe eat a snack before going to sleep.

ANS: D 60 mg/dL after waking from a nap is too low. During hours of sleep glucose levels should not be less than 70 mg/dL. Snacks before sleeping can be helpful. The premeal acceptable range is 65 to 95 mg/dL. The readings 1 hour after a meal should be less than 140 mg/dL. Two hours after eating, the readings should be less than 120 mg/dL.

10. A woman has come to the clinic for preconception counseling because she wants to start trying to get pregnant in 3 months. She can expect the following advice: A. "Discontinue all contraception now." B. "Lose weight so that you can gain more during pregnancy." C. "You may take any medications you have been taking regularly." D. "Make sure that you include adequate folic acid in your diet."

ANS: D A healthy diet before conception is the best way to ensure that adequate nutrients are available for the developing fetus. A woman's folate or folic acid intake is of particular concern in the periconception period. Neural tube defects are more common in infants of women with a poor folic acid intake. Depending on the type of contraception used, discontinuing all contraception may not be appropriate advice. Losing weight is not appropriate advice. Depending on the type of medication the woman is taking, continuing its use may not be appropriate.

17. Which basic type of pelvis includes the correct description and percentage of occurrence in women? a. Gynecoid: classic female pelvis; heart shaped; 75% b. Android: resembling the male pelvis; wide oval; 15% c. Anthropoid: resembling the pelvis of the ape; narrow; 10% d. Platypelloid: flattened, wide, and shallow pelvis; 3%

ANS: D A platypelloid pelvis is flattened, wide, and shallow; approximately 3% of women have this shape. The gynecoid pelvis is the classic female shape, slightly ovoid and rounded; approximately 50% of women have this shape. An android or malelike pelvis is heart shaped; approximately 23% of women have this shape. An anthropoid or apelike pelvis is oval and wide; approximately 24% of women have this shape.

11. The most common cause of decreased variability in the fetal heart rate (FHR) that lasts 30 minutes or less is: a. altered cerebral blood flow. b. fetal hypoxemia. c. umbilical cord compression. d. fetal sleep cycles.

ANS: D A temporary decrease in variability can occur when the fetus is in a sleep state. These sleep states do not usually last longer than 30 minutes. Altered fetal cerebral blood flow would result in early decelerations in the FHR. Fetal hypoxemia would be evidenced by tachycardia initially and then bradycardia. A persistent decrease or loss of FHR variability may be seen. Umbilical cord compression would result in variable decelerations in the FHR.

26. Which pregnant woman should restrict her weight gain during pregnancy? A. Woman pregnant with twins B. Woman in early adolescence C. Woman shorter than 62 inches or 157 cm D. Woman who was 20 lbs overweight before pregnancy

ANS: D A weight gain of 5 to 9 kg will provide sufficient nutrients for the fetus. Overweight and obese women should be advised to lose weight before conception to achieve the best pregnancy outcomes. A higher weight gain in twin gestations may help prevent low birth weights. Adolescents need to gain weight toward the higher acceptable range, which provides for their own growth as well as for fetal growth. In the past, women of short stature were advised to restrict their weight gain; however, evidence to support these guidelines has not been found.

21. The most basic information a maternity nurse should have concerning conception is that: A. ova are considered fertile 48 to 72 hours after ovulation. B. sperm remain viable in the woman's reproductive system for an average of 12 to 24 hours. C. conception is achieved when a sperm successfully penetrates the membrane surrounding the ovum. D. implantation in the endometrium occurs 6 to 10 days after conception.

ANS: D After implantation, the endometrium is called the decidua. Ova are considered fertile for about 24 hours after ovulation. Sperm remain viable in the woman's reproductive system for an average of 2 to 3 days. Penetration of the ovum by the sperm is called fertilization. Conception occurs when the zygote, the first cell of the new individual, is formed.

8. Although remarkable developments have occurred in reproductive medicine, assisted reproductive therapies are associated with numerous legal and ethical issues. Nurses can provide accurate information about the risks and benefits of treatment alternatives so couples can make informed decisions about their choice of treatment. Which issue would not need to be addressed by an infertile couple before treatment? A. Risks of multiple gestation. B. Whether or how to disclose the facts of conception to offspring. C. Freezing embryos for later use. D. Financial ability to cover the cost of treatment.

ANS: D Although the method of payment is important, obtaining this information is not the responsibility of the nurse. Many states have mandated some form of insurance to assist couples with coverage for infertility. Risk of multiple gestation is a risk of treatment of which the couple needs to be aware. To minimize the chance of multiple gestation, generally only three or fewer embryos are transferred. The couple should be informed that there may be a need for multifetal reduction. Nurses can provide anticipatory guidance on this matter. Depending on the therapy chosen, there may be a need for donor oocytes, sperm, embryos, or a surrogate mother. Couples who have excess embryos frozen for later transfer must be fully informed before consenting to the procedure. A decision must be made regarding the disposal of embryos in the event of death or divorce or if the couple no longer wants the embryos at a future time.

29. The obstetric nurse is preparing the patient for an emergency cesarean birth, with no time to administer spinal anesthesia. The nurse is aware and prepared for the greatest risk of administering general anesthesia to the patient. This risk is: a. respiratory depression. b. uterine relaxation. c. inadequate muscle relaxation. d. aspiration of stomach contents.

ANS: D Aspiration of acidic gastric contents with possible airway obstruction is a potentially fatal complication of general anesthesia. Respirations can be altered during general anesthesia, and the anesthesiologist will take precautions to maintain proper oxygenation. Uterine relaxation can occur with some anesthesia; however, this can be monitored and prevented. Inadequate muscle relaxation can be improved with medication.

3. When a nulliparous woman telephones the hospital to report that she is in labor, the nurse initially should: a. tell the woman to stay home until her membranes rupture. b. emphasize that food and fluid intake should stop. c. arrange for the woman to come to the hospital for labor evaluation. d. ask the woman to describe why she believes she is in labor.

ANS: D Assessment begins at the first contact with the woman, whether by telephone or in person. By asking the woman to describe her signs and symptoms, the nurse can begin the assessment and gather data. The amniotic membranes may or may not spontaneously rupture during labor. The patient may be instructed to stay home until the uterine contractions become strong and regular. The nurse may want to discuss the appropriate oral intake for early labor such as light foods or clear liquids, depending on the preference of the patient or her primary health care provider. Before instructing the woman to come to the hospital, the nurse should initiate the assessment during the telephone interview.

28. While you are assessing the vital signs of a pregnant woman in her third trimester, the patient complains of feeling faint, dizzy, and agitated. Which nursing intervention is appropriate? A. Have the patient stand up and retake her blood pressure. B. Have the patient sit down and hold her arm in a dependent position. C. Have the patient lie supine for 5 minutes and recheck her blood pressure on both arms. D. Have the patient turn to her left side and recheck her blood pressure in 5 minutes.

ANS: D Blood pressure is affected by maternal position during pregnancy. The supine position may cause occlusion of the vena cava and descending aorta. Turning the pregnant woman to a lateral recumbent position alleviates pressure on the blood vessels and quickly corrects supine hypotension. Pressures are significantly higher when the patient is standing. This option causes an increase in systolic and diastolic pressures. The arm should be supported at the same level of the heart. The supine position may cause occlusion of the vena cava and descending aorta, creating hypotension.

23. As part of their participation in the gynecologic portion of the physical examination, nurses should: A. take a firm approach that encourages the patient to facilitate the examination by following the physician's instructions exactly. B. explain the procedure as it unfolds and continue to question the patient to get information in a timely manner. C. take the opportunity to explain that the trendy vulvar self-examination is only for women at risk for cancer. D. Help the woman relax through proper placement of her hands and proper breathing during the examination.

ANS: D Breathing techniques are important relaxation techniques that can help the patient during the examination. The nurse should encourage the patient to participate in an active partnership with the care provider. Explanations during the procedure are fine, but many women are uncomfortable answering questions in the exposed and awkward position of the examination. Vulvar self-examination on a regular basis should be encouraged and taught during the examination.

5. A woman at 26 weeks of gestation is being assessed to determine whether she is experiencing preterm labor. What finding indicates that preterm labor is occurring? a. Estriol is not found in maternal saliva. b. Irregular, mild uterine contractions are occurring every 12 to 15 minutes. c. Fetal fibronectin is present in vaginal secretions. d. The cervix is effacing and dilated to 2 cm.

ANS: D Cervical changes such as shortened endocervical length, effacement, and dilation are predictors of imminent preterm labor. Changes in the cervix accompanied by regular contractions indicate labor at any gestation. Estriol is a form of estrogen produced by the fetus that is present in plasma at 9 weeks of gestation. Levels of salivary estriol have been shown to increase before preterm birth. Irregular, mild contractions that do not cause cervical change are not considered a threat. The presence of fetal fibronectin in vaginal secretions between 24 and 36 weeks of gestation could predict preterm labor, but it has only a 20% to 40% positive predictive value. Of more importance are other physiologic clues of preterm labor such as cervical changes.

17. The _____ is/are responsible for oxygen and carbon dioxide transport to and from the maternal bloodstream. A. decidua basalis B. blastocyst C. germ layer D. chorionic villi

ANS: D Chorionic villi are finger-like projections that develop out of the trophoblast and extend into the blood-filled spaces of the endometrium. The villi obtain oxygen and nutrients from the maternal bloodstream and dispose of carbon dioxide and waste products into the maternal blood. The decidua basalis is the portion of the decidua (endometrium) under the blastocyst where the villi attach. The blastocyst is the embryonic development stage after the morula. Implantation occurs at this stage. The germ layer is a layer of the blastocyst.

26. As a powerful central nervous system stimulant, which of these substances can lead to miscarriage, preterm labor, placental separation (abruption), and stillbirth? a. Heroin b. Alcohol c. PCP d. Cocaine

ANS: D Cocaine is a powerful CNS stimulant. Effects on pregnancy associated with cocaine use include abruptio placentae, preterm labor, precipitous birth, and stillbirth. Heroin is an opiate. Its use in pregnancy is associated with preeclampsia, intrauterine growth restriction, miscarriage, premature rupture of membranes, infections, breech presentation, and preterm labor. The most serious effect of alcohol use in pregnancy is fetal alcohol syndrome. The major concerns regarding PCP use in pregnant women are its association with polydrug abuse and the neurobehavioral effects on the neonate.

27. The major source of nutrients in the diet of a pregnant woman should be composed of: A. simple sugars. B. fats. C. fiber. D. complex carbohydrates.

ANS: D Complex carbohydrates supply the pregnant woman with vitamins, minerals, and fiber. The most common simple carbohydrate is table sugar, which is a source of energy but does not provide any nutrients. Fats provide 9 kcal in each gram, in contrast to carbohydrates and proteins, which provide only 4 kcal in each gram. Fiber is supplied primarily by complex carbohydrates.

22. To reassure and educate their pregnant patients about changes in their blood pressure, maternity nurses should be aware that: A. a blood pressure cuff that is too small produces a reading that is too low; a cuff that is too large produces a reading that is too high. B. shifting the patient's position and changing from arm to arm for different measurements produces the most accurate composite blood pressure reading at each visit. C. the systolic blood pressure increases slightly as pregnancy advances; the diastolic pressure remains constant. D. compression of the iliac veins and inferior vena cava by the uterus contributes to hemorrhoids in the later stage of term pregnancy.

ANS: D Compression of the iliac veins and inferior vena cava also leads to varicose veins in the legs and vulva. The tightness of a cuff that is too small produces a reading that is too high; similarly the looseness of a cuff that is too large results in a reading that is too low. Because maternal positioning affects readings, blood pressure measurements should be obtained in the same arm and with the woman in the same position. The systolic blood pressure generally remains constant but may decline slightly as pregnancy advances. The diastolic blood pressure first decreases and then gradually increases.

21. Which statement is true about the term contraceptive failure rate? A. It refers to the percentage of users expected to have an accidental pregnancy over a 5-year span. B. It refers to the minimum level that must be achieved to receive a government license. C. It increases over time as couples become more careless. D. It varies from couple to couple, depending on the method and the users.

ANS: D Contraceptive effectiveness varies from couple to couple, depending on how well a contraceptive method is used and how well it suits the couple. The contraceptive failure rate measures the likelihood of accidental pregnancy in the first year only. Failure rates decline over time because users gain experience.

17. Since the gene for cystic fibrosis was identified in 1989, data can be collected for the purposes of genetic counseling for couples regarding carrier status. According to statistics, how often does cystic fibrosis occur in Caucasian live births? A. 1 in 100 B. 1 in 1200 C. 1 in 2500 D. 1 in 3000

ANS: D Cystic fibrosis occurs in about 1 in 3000 Caucasian live births.

18. A woman who is 8 months pregnant asks the nurse, "Does my baby have any antibodies to fight infection?" The most appropriate response by the nurse is: A. "Your baby has all the immune globulins necessary: IgG, IgM, and IgA." B. "Your baby won't receive any antibodies until he/she is born and you breastfeed him." C. "Your baby does not have any antibodies to fight infection." D. "Your baby has IgG and IgM."

ANS: D During the third trimester, the only immune globulin that crosses the placenta, IgG, provides passive acquired immunity to specific bacterial toxins. The fetus produces IgM by the end of the first trimester. IgA is not produced by the baby. By the third trimester, the fetus has IgG and IgM. Breastfeeding supplies the baby with IgA. "Your baby does not have any antibodies to fight infection" is an inaccurate statement.

9. With regard to the estimation and interpretation of the recurrence of risks for genetic disorders, nurses should be aware that: A. with a dominant disorder, the likelihood of the second child also having the condition is 100%. B. an autosomal recessive disease carries a one in eight risk of the second child also having the disorder. C. disorders involving maternal ingestion of drugs carry a one in four chance of being repeated in the second child. D. the risk factor remains the same no matter how many affected children are already in the family.

ANS: D Each pregnancy is an independent event. The risk factor (e.g., one in two, one in four) remains the same for each child, no matter how many children are born to the family.In a dominant disorder, the likelihood of recurrence in subsequent children is 50% (one in two). An autosomal recessive disease carries a one in four chance of recurrence. In disorders involving maternal ingestion of drugs, subsequent children would be at risk only if the mother continued to take drugs; the rate of risk would be difficult to calculate.

1. An 18-year-old pregnant woman, gravida 1, is admitted to the labor and birth unit with moderate contractions every 5 minutes that last 40 seconds. The woman states, "My contractions are so strong that I don't know what to do with myself." The nurse should: a. assess for fetal well-being. b. encourage the woman to lie on her side. c. disturb the woman as little as possible. d. recognize that pain is personalized for each individual.

ANS: D Each woman's pain during childbirth is unique and is influenced by a variety of physiologic, psychosocial, and environmental factors. A critical issue for the nurse is how support can make a difference in the pain of the woman during labor and birth. Assessing for fetal well-being includes no information that would indicate fetal distress or a logical reason to be overly concerned about the well-being of the fetus. The left lateral position is used to alleviate fetal distress, not maternal stress. The nurse has an obligation to provide physical, emotional, and psychosocial care and support to the laboring woman. This patient clearly needs support.

9. A woman in week 34 of pregnancy reports that she is very uncomfortable because of heartburn. The nurse would suggest that the woman: A. substitute other calcium sources for milk in her diet. B. lie down after each meal. C. reduce the amount of fiber she consumes. D. eat five small meals daily.

ANS: D Eating small, frequent meals may help with heartburn, nausea, and vomiting. Substituting other calcium sources for milk, lying down after eating, and reducing fiber intake are inappropriate dietary suggestions for all pregnant women and do not alleviate heartburn.

23. When assessing the relative advantages and disadvantages of internal and external electronic fetal monitoring, nurses comprehend that both: a. can be used when membranes are intact. b. measure the frequency, duration, and intensity of uterine contractions. c. may need to rely on the woman to indicate when uterine activity (UA) is occurring. d. can be used during the antepartum and intrapartum periods.

ANS: D External monitoring can be used in both periods; internal monitoring can be used only in the intrapartum period. For internal monitoring the membranes must have ruptured, and the cervix must be sufficiently dilated. Internal monitoring measures the intensity of contractions; external monitoring cannot do this. With external monitoring, the woman may need to alert the nurse that UA is occurring; internal monitoring does not require this.

13. Certain changes stimulate chemoreceptors in the aorta and carotid bodies to prepare the fetus for initiating respirations immediately after birth. Which change in fetal physiologic activity is not part of this process? a. Fetal lung fluid is cleared from the air passages during labor and vaginal birth. b. Fetal partial pressure of oxygen (PO2) decreases. c. Fetal partial pressure of carbon dioxide in arterial blood (PaCO2) increases. d. Fetal respiratory movements increase during labor.

ANS: D Fetal respiratory movements actually decrease during labor. Fetal lung fluid is cleared from the air passages during labor and vaginal birth. Fetal PO2 decreases, and fetal PaCO2 increases.

33. A nurse practitioner performs a clinical breast examination on a woman diagnosed with fibroadenoma. The nurse knows that fibroadenoma is characterized by: A. inflammation of the milk ducts and glands behind the nipples. B. thick, sticky discharge from the nipple of the affected breast. C. lumpiness in both breasts that develops 1 week before menstruation. D. a single lump in one breast that can be expected to shrink as the woman ages.

ANS: D Fibroadenomas are characterized by discrete, usually solitary lumps smaller than 3 cm in diameter. Fibroadenomas increase in size during pregnancy and shrink as the woman ages. Inflammation of the milk ducts is associated with mammary duct ectasia, not fibroadenoma. A thick, sticky discharge is associated with galactorrhea, not fibroadenoma. Lumpiness before menstruation is associated with fibrocystic changes of the breast.

14. With regard to the care management of preterm labor, nurses should be aware that: a. all women must be considered at risk for preterm labor and prediction is so hit-and-miss, teaching pregnant women the symptoms probably causes more harm through false alarms. b. Braxton Hicks contractions often signal the onset of preterm labor. c. preterm labor is likely to be the start of an extended labor, a woman with symptoms can wait several hours before contacting the primary caregiver. d. the diagnosis of preterm labor is based on gestational age, uterine activity, and progressive cervical change.

ANS: D Gestational age of 20 to 37 weeks, uterine contractions, and a cervix that is 80% effaced or dilated 2 cm indicates preterm labor. It is essential that nurses teach women how to detect the early symptoms of preterm labor. Braxton Hicks contractions resemble preterm labor contractions, but they are not true labor. Waiting too long to see a health care provider could result in not administering essential medications. Preterm labor is not necessarily long-term labor.

1. A woman arrives at the clinic seeking confirmation that she is pregnant. The following information is obtained: She is 24 years old with a body mass index (BMI) of 17.5. She admits to having used cocaine "several times" during the past year and drinks alcohol occasionally. Her blood pressure (BP) is 108/70 mm Hg, her pulse rate is 72 beats/min, and her respiratory rate is 16 breaths/min. The family history is positive for diabetes mellitus and cancer. Her sister recently gave birth to an infant with a neural tube defect (NTD). Which characteristics place the woman in a high risk category? A. Blood pressure, age, and BMI B. Drug/alcohol use, age, and family history C. Family history, blood pressure, and BMI D. Family history, BMI, and drug/alcohol abuse

ANS: D Her family history of NTD, low BMI, and substance abuse all are high risk factors of pregnancy. The woman's BP is normal, and her age does not put her at risk. Her BMI is low and may indicate poor nutritional status, which would be a high risk. The woman's drug/alcohol use and family history put her in a high risk category, but her age does not. The woman's family history puts her in a high risk category. Her BMI is low and may indicate poor nutritional status, which would be high risk. Her BP is normal.

12. Which statement correctly describes the effects of various pain factors? a. Higher prostaglandin levels arising from dysmenorrhea can blunt the pain of childbirth. b. Upright positions in labor increase the pain factor because they cause greater fatigue. c. Women who move around trying different positions are experiencing more pain. d. Levels of pain-mitigating β-endorphins are higher during a spontaneous, natural childbirth.

ANS: D Higher endorphin levels help women tolerate pain and reduce anxiety and irritability. Higher prostaglandin levels correspond to more severe labor pains. Upright positions in labor usually result in improved comfort and less pain. Moving freely to find more comfortable positions is important for reducing pain and muscle tension.

16. Human chorionic gonadotropin (hCG) is an important biochemical marker for pregnancy and the basis for many tests. A maternity nurse should be aware that: A. hCG can be detected 2.5 weeks after conception. B. the hCG level increases gradually and uniformly throughout pregnancy. C. much lower than normal increases in the level of hCG may indicate a postdate pregnancy. D. a higher than normal level of hCG may indicate an ectopic pregnancy or Down syndrome.

ANS: D Higher levels also could be a sign of multiple gestation. hCG can be detected 7 to 8 days after conception. The hCG level fluctuates during pregnancy: peaking, declining, stabilizing, and increasing again. Abnormally slow increases may indicate impending miscarriage.

18. To reassure and educate pregnant patients about changes in the cervix, vagina, and position of the fetus, nurses should be aware that: A. because of a number of changes in the cervix, abnormal Papanicolaou (Pap) tests are much easier to evaluate. B. Quickening is a technique of palpating the fetus to engage it in passive movement. C. the deepening color of the vaginal mucosa and cervix (Chadwick's sign) usually appears in the second trimester or later as the vagina prepares to stretch during labor. D. increased vascularity of the vagina increases sensitivity and may lead to a high degree of arousal, especially in the second trimester.

ANS: D Increased sensitivity and an increased interest in sex sometimes go together. This frequently occurs during the second trimester. Cervical changes make evaluation of abnormal Pap tests more difficult. Quickening is the first recognition of fetal movements by the mother. Ballottement is a technique used to palpate the fetus. Chadwick's sign appears from the sixth to eighth weeks.

18. Nurses should be aware that the induction of labor: a. can be achieved by external and internal version techniques. b. is also known as a trial of labor (TOL). c. is almost always done for medical reasons. d. is rated for viability by a Bishop score.

ANS: D Induction of labor is likely to be more successful with a Bishop score of 9 or higher for first-time mothers and 5 or higher for veterans. Version is turning of the fetus to a better position by a physician for an easier or safer birth. A trial of labor is the observance of a woman and her fetus for several hours of active labor to assess the safety of vaginal birth. Two thirds of cases of induced labor are elective and are not done for medical reasons.

20. When using intermittent auscultation (IA) for fetal heart rate, nurses should be aware that: a. they can be expected to cover only two or three patients when IA is the primary method of fetal assessment. b. the best course is to use the descriptive terms associated with electronic fetal monitoring (EFM) when documenting results. c. if the heartbeat cannot be found immediately, a shift must be made to EFM. d. ultrasound can be used to find the fetal heartbeat and reassure the mother if initial difficulty was a factor.

ANS: D Locating fetal heartbeats often takes time. Mothers can be reassured verbally and by the ultrasound pictures if ultrasound is used to help locate the heartbeat. When used as the primary method of fetal assessment, auscultation requires a nurse-to-patient ratio of one to one. Documentation should use only terms that can be numerically defined; the usual visual descriptions of EFM are inappropriate.

5. Nafarelin is currently used as a treatment for mild-to-severe endometriosis. The nurse should tell a woman taking this medication that the drug: A. stimulates the secretion of gonadotropin-releasing hormone (GnRH), thereby stimulating ovarian activity. B. should be sprayed into one nostril every other day. C. should be injected into subcutaneous tissue bid. D. can cause her to experience some hot flashes and bone loss.

ANS: D Nafarelin is a GnRH agonist, and its side effects are similar to effects of menopause. The hypoestrogenism effect results in hot flashes and bone loss. Nafarelin is a GnRH agonist that suppresses the secretion of GnRH and is administered twice daily by nasal spray.

27. Which method of pain management is safest for a gravida 3 para 2 admitted at 8 cm cervical dilation? a. Epidural anesthesia b. Narcotics c. Spinal block d. Breathing and relaxation techniques

ANS: D Nonpharmacologic methods of pain management may be the best option for a woman in advanced labor. It is unlikely that enough time remains to administer epidural or spinal anesthesia. A narcotic given at this time may reach its peak about the time of birth and result in respiratory depression in the newborn.

16. As a perinatal nurse you realize that a fetal heart rate that is tachycardic, is bradycardic, or has late decelerations or loss of variability is nonreassuring and is associated with: a. hypotension. b. cord compression. c. maternal drug use. d. hypoxemia.

ANS: D Nonreassuring heart rate patterns are associated with fetal hypoxemia. Fetal bradycardia may be associated with maternal hypotension. Fetal variable decelerations are associated with cord compression. Maternal drug use is associated with fetal tachycardia.

25. Which statement about a condition of pregnancy is accurate? A. Insufficient salivation (ptyalism) is caused by increases in estrogen. B. Acid indigestion (pyrosis) begins early but declines throughout pregnancy. C. Hyperthyroidism often develops (temporarily) because hormone production increases. D. Nausea and vomiting rarely have harmful effects on the fetus and may be beneficial.

ANS: D Normal nausea and vomiting rarely produce harmful effects, and nausea and vomiting periods may be less likely to result in miscarriage or preterm labor. Ptyalism is excessive salivation, which may be caused by a decrease in unconscious swallowing or stimulation of the salivary glands. Pyrosis begins in the first trimester and intensifies through the third trimester. Increased hormone production does not lead to hyperthyroidism in pregnant women.

7. In their role of implementing a plan of care for infertile couples, nurses should: A. be comfortable with their sexuality and nonjudgmental about others to counsel their patients effectively. B. know about such nonmedical remedies as diet, exercise, and stress management. C. be able to direct patients to sources of information about what herbs to take that might help and which ones to avoid. D. do all of the above plus be knowledgeable about potential drug and surgical remedies.

ANS: D Nurses should be open to and ready to help with a variety of medical and nonmedical approaches.

10. With regard to dysmenorrhea, nurses should be aware that: A. it is more common in older women. B. it is more common in leaner women who exercise strenuously. C. symptoms can begin at any point in the ovulatory cycle. D. pain usually occurs in the suprapubic area or lower abdomen.

ANS: D Pain is described as sharp and cramping or sometimes as a dull ache. It may radiate to the lower back or upper thighs. Dysmenorrhea is more common in women 17 to 24 years old, women who smoke, and women who are obese. Symptoms begin with menstruation or sometimes a few hours before the onset of flow.

24. The recommended treatment for the prevention of human immunodeficiency virus (HIV) transmission to the fetus during pregnancy is: a. acyclovir. b. ofloxacin. c. podophyllin. d. zidovudine.

ANS: D Perinatal transmission of HIV has decreased significantly in the past decade as a result of prophylactic administration of the antiretroviral drug zidovudine to pregnant women in the prenatal and perinatal periods. Acyclovir is an antiviral treatment for HSV. Ofloxacin is an antibacterial treatment for gonorrhea. Podophyllin is a solution used in the treatment of human papillomavirus.

23. When discussing work and travel during pregnancy with a pregnant patient, nurses should instruct them that: A. women should sit for as long as possible and cross their legs at the knees from time to time for exercise. B. women should avoid seat belts and shoulder restraints in the car because they press on the fetus. C. metal detectors at airport security checkpoints can harm the fetus if the woman passes through them a number of times. D. while working or traveling in a car or on a plane, women should arrange to walk around at least every 2 hours or so.

ANS: D Periodic walking helps prevent thrombophlebitis. Pregnant women should avoid sitting or standing for long periods and crossing the legs at the knees. Pregnant women must wear lap belts and shoulder restraints. The most common injury to the fetus comes from injury to the mother. Metal detectors at airport security checkpoints do not harm fetuses.

26. Glucose metabolism is profoundly affected during pregnancy because: A. pancreatic function in the islets of Langerhans is affected by pregnancy. B. the pregnant woman uses glucose at a more rapid rate than the nonpregnant woman. C. the pregnant woman increases her dietary intake significantly. D. placental hormones are antagonistic to insulin, thus resulting in insulin resistance.

ANS: D Placental hormones, estrogen, progesterone, and human placental lactogen (HPL) create insulin resistance. Insulin is also broken down more quickly by the enzyme placental insulinase. Pancreatic functioning is not affected by pregnancy. The glucose requirements differ because of the growing fetus. The pregnant woman should increase her intake by 200 calories a day.

7. Which assessment is least likely to be associated with a breech presentation? a. Meconium-stained amniotic fluid b. Fetal heart tones heard at or above the maternal umbilicus c. Preterm labor and birth d. Postterm gestation

ANS: D Postterm gestation is not likely to be seen with a breech presentation. The presence of meconium in a breech presentation may result from pressure on the fetal wall as it traverses the birth canal. Fetal heart tones heard at the level of the umbilical level of the mother are a typical finding in a breech presentation because the fetal back would be located in the upper abdominal area. Breech presentations often occur in preterm births.

4. The hormone responsible for maturation of mammary gland tissue is: a. estrogen. b. testosterone. c. prolactin. d. progesterone.

ANS: D Progesterone causes maturation of the mammary gland tissue, specifically acinar structures of the lobules. Estrogen increases the vascularity of the breast tissue. Testosterone has no bearing on breast development. Prolactin is produced after birth and released from the pituitary gland. It is produced in response to infant suckling and emptying of the breasts.

11. The nurse caring for the pregnant patient must understand that the hormone essential for maintaining pregnancy is: A. estrogen. B. human chorionic gonadotropin (hCG). C. oxytocin. D. progesterone.

ANS: D Progesterone is essential for maintaining pregnancy; it does so by relaxing smooth muscles. This reduces uterine activity and prevents miscarriage. Estrogen plays a vital role in pregnancy, but it is not the primary hormone for maintaining pregnancy. hCG levels increase at implantation but decline after 60 to 70 days. Oxytocin stimulates uterine contractions.

16. The two primary areas of risk for sexually transmitted infections (STIs) are: A. sexual orientation and socioeconomic status. B. age and educational level. C. large number of sexual partners and race. D. risky sexual behaviors and inadequate preventive health behaviors.

ANS: D Risky sexual behaviors and inadequate preventive health behaviors put a person at risk for acquiring or transmitting an STI. Although low socioeconomic status may be a factor in avoiding purchasing barrier protection, sexual orientation does not put one at higher risk. Younger individuals and individuals with less education may be unaware of proper prevention techniques; however, these are not the primary areas of risk for STIs. Having a large number of sexual partners is a risk-taking behavior, but race does not increase the risk for STIs.

7. A pregnant woman's diet history indicates that she likes the following list of foods. The nurse would encourage this woman to consume more of which food to increase her calcium intake? A. Fresh apricots B. Canned clams C. Spaghetti with meat sauce D. Canned sardines

ANS: D Sardines are rich in calcium. Fresh apricots, canned clams, and spaghetti with meat sauce are not high in calcium.

13. When caring for a pregnant woman with cardiac problems, the nurse must be alert for signs and symptoms of cardiac decompensation, which include: A. a regular heart rate and hypertension. B. an increased urinary output, tachycardia, and dry cough. C. shortness of breath, bradycardia, and hypertension. D. dyspnea; crackles; and an irregular, weak pulse.

ANS: D Signs of cardiac decompensation include dyspnea; crackles; an irregular, weak, rapid pulse; rapid respirations; a moist, frequent cough; generalized edema; increasing fatigue; and cyanosis of the lips and nail beds. A regular heart rate and hypertension are not generally associated with cardiac decompensation. Tachycardia would indicate cardiac decompensation, but increased urinary output and a dry cough would not. Shortness of breath would indicate cardiac decompensation, but bradycardia and hypertension would not.

28. A pregnant woman's diet may not meet her need for folates. A good source of this nutrient is: A. chicken. B. cheese. C. potatoes. D. green leafy vegetables.

ANS: D Sources of folates include green leafy vegetables, whole grains, fruits, liver, dried peas, and beans. Chicken and cheese are excellent sources of protein but are poor in folates. Potatoes contain carbohydrates and vitamins and minerals but are poor in folates.

7. Numerous changes in the integumentary system occur during pregnancy. Which change persists after birth? A. Epulis B. Chloasma C. Telangiectasia D. Striae gravidarum

ANS: D Striae gravidarum, or stretch marks, reflect separation within the underlying connective tissue of the skin. They usually fade after birth, although they never disappear completely. An epulis is a red, raised nodule on the gums that bleeds easily. Chloasma, or mask of pregnancy, is a blotchy, brown hyperpigmentation of the skin over the cheeks, nose, and forehead, especially in dark-complexioned pregnant women. Chloasma usually fades after the birth. Telangiectasia, or vascular spiders, are tiny, star-shaped or branch-like, slightly raised, pulsating end-arterioles usually found on the neck, thorax, face, and arms. They occur as a result of elevated levels of circulating estrogen. These usually disappear after birth.

19. During a physical assessment of an at-risk patient, the nurse notes generalized edema, crackles at the base of the lungs, and some pulse irregularity. These are most likely signs of: A. euglycemia. B. rheumatic fever. C. pneumonia. D. cardiac decompensation.

ANS: D Symptoms of cardiac decompensation may appear abruptly or gradually. Euglycemia is a condition of normal glucose levels. These symptoms indicate cardiac decompensation. Rheumatic fever can cause heart problems, but it does not manifest with these symptoms, which indicate cardiac decompensation. Pneumonia is an inflammation of the lungs and would not likely generate these symptoms, which indicate cardiac decompensation.

9. While evaluating an external monitor tracing of a woman in active labor whose labor is being induced, the nurse notes that the fetal heart rate (FHR) begins to decelerate at the onset of several contractions and returns to baseline before each contraction ends. The nurse should: a. change the woman's position. b. discontinue the oxytocin infusion. c. insert an internal monitor. d. document the finding in the patient's record.

ANS: D The FHR indicates early decelerations, which are not an ominous sign and do not require any intervention. The nurse should simply document these findings.

10. A woman is using the basal body temperature (BBT) method of contraception. She calls the clinic and tells the nurse, "My period is due in a few days, and my temperature has not gone up." The nurse's most appropriate response is: A. "This probably means that you're pregnant." B. "Don't worry; it's probably nothing." C. "Have you been sick this month?" D. "You probably didn't ovulate during this cycle."

ANS: D The absence of a temperature decrease most likely is the result of lack of ovulation. Pregnancy cannot occur without ovulation (which is being measured using the BBT method). A comment such as "Don't worry; it's probably nothing" discredits the patient's concerns. Illness would most likely cause an increase in BBT.

15. A woman currently uses a diaphragm and spermicide for contraception. She asks the nurse what the major differences are between the cervical cap and the diaphragm. The nurse's most appropriate response is: A. "No spermicide is used with the cervical cap, so it's less messy." B. "The diaphragm can be left in place longer after intercourse." C. "Repeated intercourse with the diaphragm is more convenient." D. "The cervical cap can safely be used for repeated acts of intercourse without adding more spermicide later."

ANS: D The cervical cap can be inserted hours before sexual intercourse without the need for additional spermicide later. No additional spermicide is required for repeated acts of intercourse. Spermicide should be used inside the cap as an additional chemical barrier. The cervical cap should remain in place for 6 hours after the last act of intercourse. Repeated intercourse with the cervical cap is more convenient because no additional spermicide is needed.

24. With regard to the noncontraceptive medical effects of combined oral contraceptive pills (COCs), nurses should be aware that: A. COCs can cause toxic shock syndrome if the prescription is wrong. B. hormonal withdrawal bleeding usually is a bit more profuse than in normal menstruation and lasts a week. C. COCs increase the risk of endometrial and ovarian cancer. D. the effectiveness of COCs can be altered by some over-the-counter medications and herbal supplements.

ANS: D The effectiveness of COCs can be altered by some over-the-counter medications and herbal supplements. Toxic shock syndrome can occur in some diaphragm users, but it is not a consequence of taking oral contraceptive pills. Hormonal withdrawal bleeding usually is lighter than in normal menstruation and lasts a couple of days. Oral contraceptive pills offer protection against the risk of endometrial and ovarian cancers.

19. Which statement regarding the care of a client in labor is correct and important to the nurse as he or she formulates the plan of care? a. The woman's blood pressure will increase during contractions and fall back to prelabor normal levels between contractions. b. The use of the Valsalva maneuver is encouraged during the second stage of labor to relieve fetal hypoxia. c. Having the woman point her toes will reduce leg cramps. d. Endogenous endorphins released during labor will raise the woman's pain threshold and produce sedation.

ANS: D The endogenous endorphins released during labor will raise the woman's pain threshold and produce sedation. In addition, physiologic anesthesia of the perineal tissues, caused by the pressure of the presenting part, decreases the mother's perception of pain. Blood pressure levels increase during contractions but remain somewhat elevated between them. The use of the Valsalva maneuver is discouraged during the second stage labor because of a number of unhealthy outcomes, including fetal hypoxia. Pointing the toes can cause leg cramps, as can the process of labor itself.

26. The priority nursing intervention after an amniotomy should be to: a. assess the color of the amniotic fluid. b. change the patient's gown. c. estimate the amount of amniotic fluid. d. assess the fetal heart rate.

ANS: D The fetal heart rate must be assessed immediately after the rupture of the membranes to determine whether cord prolapse or compression has occurred. Secondary to FHR assessment, amniotic fluid amount, color, odor, and consistency is assessed. Dry clothing is important for patient comfort; however, it is not the top priority.

26. If a woman complains of back labor pain, the nurse could best suggest that she: a. lie on her back for a while with her knees bent. b. do less walking around. c. take some deep, cleansing breaths. d. lean over a birth ball with her knees on the floor.

ANS: D The hands-and-knees position, with or without the aid of a birth ball, should help with the back pain. The supine position should be discouraged. Walking generally is encouraged.

1. A new mother asks the nurse when the "soft spot" on her son's head will go away. What is the nurse's best response, based upon her understanding of when the anterior frontal closes? a. 2 months b. 8 months c. 12 months d. 18 months

ANS: D The larger of the two fontanels, the anterior fontanel, closes by 18 months after birth. The posterior fontanel closes at 6 to 8 weeks. The remaining three options are too early for the anterior fontanel to close.

32. The most dangerous effect on the fetus of a mother who smokes cigarettes while pregnant is: A. genetic changes and anomalies. B. extensive central nervous system damage. C. fetal addiction to the substance inhaled. D. intrauterine growth restriction.

ANS: D The major consequences of smoking tobacco during pregnancy are low-birth-weight infants, prematurity, and increased perinatal loss. Cigarettes normally will not cause genetic changes or extensive central nervous system damage. Addiction to tobacco is not a usual concern related to the neonate.

20. The stimulated release of gonadotropin-releasing hormone and follicle-stimulating hormone is part of the: A. menstrual cycle. B. endometrial cycle. C. ovarian cycle. D. hypothalamic-pituitary cycle.

ANS: D The menstrual, endometrial, and ovarian cycles are interconnected. However, the cyclic release of hormones is the function of the hypothalamus and pituitary glands.

7. The nurse who performs vaginal examinations to assess a woman's progress in labor should: a. perform an examination at least once every hour during the active phase of labor. b. perform the examination with the woman in the supine position. c. wear two clean gloves for each examination. d. discuss the findings with the woman and her partner.

ANS: D The nurse should discuss the findings of the vaginal examination with the woman and her partner and report them to the primary care provider. A vaginal examination should be performed only when indicated by the status of the woman and her fetus. The woman should be positioned to avoid supine hypotension. The examiner should wear a sterile glove while performing a vaginal examination for a laboring woman.

25. Nutrition is one of the most significant factors influencing the outcome of a pregnancy. It is an alterable and important preventive measure for various potential problems, such as low birth weight and prematurity. While completing the physical assessment of the pregnant patient, the nurse can evaluate the patient's nutritional status by observing a number of physical signs. Which sign would indicate that the patient has unmet nutritional needs? A. Normal heart rate, rhythm, and blood pressure B. Bright, clear, shiny eyes C. Alert, responsive, and good endurance D. Edema, tender calves, and tingling

ANS: D The physiologic changes of pregnancy may complicate the interpretation of physical findings. Lower-extremity edema often occurs when caloric and protein deficiencies are present; however, it may also be a common physical finding during the third trimester. It is essential that the nurse complete a thorough health history and physical assessment and request further laboratory testing if indicated. A malnourished pregnant patient may display rapid heart rate, abnormal rhythm, enlarged heart, and elevated blood pressure. A patient receiving adequate nutrition has bright, shiny eyes with no sores and moist, pink membranes. Pale or red membranes, dryness, infection, dull appearance of the cornea, or blue sclerae all are signs of poor nutrition. This patient is well nourished. Cachexia, listlessness, and tiring easily would be indications of poor nutritional status.

23. The nurse practicing in a labor setting knows that the woman most at risk for uterine rupture is: a. a gravida 3 who has had two low-segment transverse cesarean births. b. a gravida 2 who had a low-segment vertical incision for delivery of a 10-lb infant. c. a gravida 5 who had two vaginal births and two cesarean births. d. a gravida 4 who has had all cesarean births.

ANS: D The risk of uterine rupture increases for the patient who has had multiple prior births with no vaginal births. As the number of prior uterine incisions increases, so does the risk for uterine rupture. Low-segment transverse cesarean scars do not predispose the patient to uterine rupture.

28. A nurse may be called on to stimulate the fetal scalp: a. as part of fetal scalp blood sampling. b. in response to tocolysis. c. in preparation for fetal oxygen saturation monitoring. d. to elicit an acceleration in the fetal heart rate (FHR).

ANS: D The scalp can be stimulated using digital pressure during a vaginal examination. Fetal scalp blood sampling involves swabbing the scalp with disinfectant before a sample is collected. The nurse would stimulate the fetal scalp to elicit an acceleration of the FHR. Tocolysis is relaxation of the uterus. Fetal oxygen saturation monitoring involves the insertion of a sensor.

7. A woman who is 14 weeks pregnant tells the nurse that she always had a glass of wine with dinner before she became pregnant. She has abstained during her first trimester and would like to know if it is safe for her to have a drink with dinner now. The nurse would tell her: A. "Since you're in your second trimester, there's no problem with having one drink with dinner." B. "One drink every night is too much. One drink three times a week should be fine." C. "Since you're in your second trimester, you can drink as much as you like." D. "Because no one knows how much or how little alcohol it takes to cause fetal problems, the best course is to abstain throughout your pregnancy."

ANS: D The statement "Because no one knows how much or how little alcohol it takes to cause fetal problems, the best course is to abstain throughout your pregnancy" is accurate. A safe level of alcohol consumption during pregnancy has not yet been established. Although the consumption of occasional alcoholic beverages may not be harmful to the mother or her developing fetus, complete abstinence is strongly advised.

9. A woman is 3 months pregnant. At her prenatal visit, she tells the nurse that she does not know what is happening; one minute she's happy that she is pregnant, and the next minute she cries for no reason. Which response by the nurse is most appropriate? A. "Don't worry about it; you'll feel better in a month or so." B. "Have you talked to your husband about how you feel?" C. "Perhaps you really don't want to be pregnant." D. "Hormonal changes during pregnancy commonly result in mood swings."

ANS: D The statement "Hormonal changes during pregnancy commonly result in mood swings" is accurate and the most appropriate response by the nurse. The statement "Don't worry about it; you'll feel better in a month or so" dismisses the patient's concerns and is not the most appropriate response. Although women should be encouraged to share their feelings, "Have you talked to your husband about how you feel" is not the most appropriate response and does not provide the patient with a rationale for the psychosocial dynamics of her pregnancy. "Perhaps you really don't want to be pregnant" is completely inappropriate and deleterious to the psychologic well-being of the woman. Hormonal and metabolic adaptations often cause mood swings in pregnancy. The woman's responses are normal. She should be reassured about her feelings.

1. A 22-year-old woman pregnant with a single fetus has a preconception body mass index (BMI) of 24. When she was seen in the clinic at 14 weeks of gestation, she had gained 1.8 kg (4 lbs) since conception. How would the nurse interpret this? A. This weight gain indicates possible gestational hypertension. B. This weight gain indicates that the woman's infant is at risk for intrauterine growth restriction (IUGR). C. This weight gain cannot be evaluated until the woman has been observed for several more weeks. D. The woman's weight gain is appropriate for this stage of pregnancy.

ANS: D The statement "The woman's weight gain is appropriate for this stage of pregnancy" is accurate. This woman's BMI is within the normal range. During the first trimester, the average total weight gain is only 1 to 2 kg. Although weight gain does indicate possible gestational hypertension, it does not apply to this patient. The desirable weight gain during pregnancy varies among women. The primary factor to consider in making a weight gain recommendation is the appropriateness of the prepregnancy weight for the woman's height. A commonly used method of evaluating the appropriateness of weight for height is the BMI. Although weight gain does indicate risk for IUGR, this does not apply to this patient. Weight gain should occur at a steady rate throughout the pregnancy. The optimal rate of weight gain also depends on the stage of the pregnancy.

9. In comparing the abdominal and transvaginal methods of ultrasound examination, nurses should explain to their patients that: A. both require the woman to have a full bladder. B. the abdominal examination is more useful in the first trimester. C. initially the transvaginal examination can be painful. D. the transvaginal examination allows pelvic anatomy to be evaluated in greater detail.

ANS: D The transvaginal examination allows pelvic anatomy to be evaluated in greater detail and allows intrauterine pregnancies to be diagnosed earlier. The abdominal examination requires a full bladder; the transvaginal examination requires an empty bladder. The transvaginal examination is more useful in the first trimester; the abdominal examination works better after the first trimester. Neither method should be painful, although with the transvaginal examination the woman feels pressure as the probe is moved.

10. The long-term treatment plan for an adolescent with an eating disorder focuses on: A. managing the effects of malnutrition. B. establishing sufficient caloric intake. C. improving family dynamics. D. restructuring perception of body image.

ANS: D The treatment of eating disorders is initially focused on reestablishing physiologic homeostasis. Once body systems are stabilized, the next goal of treatment for eating disorders is maintaining adequate caloric intake. Although family therapy is indicated when dysfunctional family relationships exist, the primary focus of therapy for eating disorders is to help the adolescent cope with complex issues. The focus of treatment in individual therapy for an eating disorder involves restructuring cognitive perceptions about the individual's body image.

1. The two primary functions of the ovaries are: A. normal female development and sex hormone release. B. ovulation and internal pelvic support. C. sexual response and ovulation. D. ovulation and hormone production.

ANS: D The two functions of the ovaries are ovulation and hormone production. The presence of ovaries does not guarantee normal female development. The ovaries produce estrogen, progesterone, and androgen. Ovulation is the release of a mature ovum from the ovary; the ovaries are not responsible for internal pelvic support. Sexual response is a feedback mechanism involving the hypothalamus, anterior pituitary gland, and the ovaries. Ovulation does occur in the ovaries.

8. A multiparous woman has been in labor for 8 hours. Her membranes have just ruptured. The nurse's initial response would be to: a. prepare the woman for imminent birth. b. notify the woman's primary health care provider. c. document the characteristics of the fluid. d. assess the fetal heart rate and pattern.

ANS: D The umbilical cord may prolapse when the membranes rupture. The fetal heart rate and pattern should be monitored closely for several minutes immediately after ROM to ascertain fetal well-being, and the findings should be documented. Rupture of membranes (ROM) may increase the intensity and frequency of the uterine contractions, but it does not indicate that birth is imminent. The nurse may notify the primary care provider after ROM occurs and fetal well-being and the response to ROM have been assessed. The nurse's priority is to assess fetal well-being. The nurse should document the characteristics of the amniotic fluid, but the initial response is to assess fetal well-being and the response to ROM.

37. The nurse providing care in a women's health care setting must be aware regarding which sexually transmitted infection that can be successfully treated and cured? A. Herpes B. Acquired immunodeficiency syndrome (AIDS) C. Venereal warts D. Chlamydia

ANS: D The usual treatment for infection by the bacterium Chlamydia is doxycycline or azithromycin. Concurrent treatment of all sexual partners is needed to prevent recurrence. There is no known cure for herpes, and treatment focuses on pain relief and preventing secondary infections. Because there is no known cure for AIDS, prevention and early detection are the primary focus of care management. Condylomata acuminata are caused by human papillomavirus. No treatment eradicates the virus.

17. What represents a typical progression through the phases of a woman's establishing a relationship with the fetus? A. Accepts the fetus as distinct from herself—accepts the biologic fact of pregnancy—has a feeling of caring and responsibility. B. Fantasizes about the child's gender and personality—views the child as part of herself—becomes introspective. C. Views the child as part of herself—has feelings of well-being—accepts the biologic fact of pregnancy. D. "I am pregnant."—"I am going to have a baby."—"I am going to be a mother."

ANS: D The woman first centers on herself as pregnant, then on the baby as an entity separate from herself, and then on her responsibilities as a mother. The expressions, "I am pregnant," "I am going to have a baby," and "I am going to be a mother" sum up the progression through the three phases.

37. A primigravida at 39 weeks of gestation is observed for 2 hours in the intrapartum unit. The fetal heart rate has been normal. Contractions are 5 to 9 minutes apart, 20 to 30 seconds in duration, and of mild intensity. Cervical dilation is 1 to 2 cm and uneffaced (unchanged from admission). Membranes are intact. The nurse should expect the woman to be: a. admitted and prepared for a cesarean birth. b. admitted for extended observation. c. discharged home with a sedative. d. discharged home to await the onset of true labor.

ANS: D This situation describes a woman with normal assessments who is probably in false labor and will likely not deliver rapidly once true labor begins. There is no indication that further assessments or observations are indicated; therefore, the patient will be discharged along with instructions to return when contractions increase in intensity and frequency. Neither a cesarean birth nor a sedative is required at this time.

6. A woman in the active phase of the first stage of labor is using a shallow pattern of breathing, which is about twice the normal adult breathing rate. She starts to complain about feeling light-headed and dizzy and states that her fingers are tingling. The nurse should: a. notify the woman's physician. b. tell the woman to slow the pace of her breathing. c. administer oxygen via a mask or nasal cannula. d. help her breathe into a paper bag.

ANS: D This woman is experiencing the side effects of hyperventilation, which include the symptoms of lightheadedness, dizziness, tingling of the fingers, or circumoral numbness. Having the woman breathe into a paper bag held tightly around her mouth and nose may eliminate respiratory alkalosis. This enables her to rebreathe carbon dioxide and replace the bicarbonate ion.

28. Informed consent concerning contraceptive use is important because some of the methods: A. are invasive procedures that require hospitalization. B. require a surgical procedure to insert. C. may not be reliable. D. have potentially dangerous side effects.

ANS: D To make an informed decision about the use of contraceptives, it is important for couples to be aware of potential side effects. The only contraceptive method that is a surgical procedure and requires hospitalization is sterilization. Some methods have greater efficacy than others, and this should be included in the teaching.

13. You are evaluating the fetal monitor tracing of your patient, who is in active labor. Suddenly you see the fetal heart rate (FHR) drop from its baseline of 125 beats/min down to 80 beats/min. You reposition the mother, provide oxygen, increase intravenous (IV) fluid, and perform a vaginal examination. The cervix has not changed. Five minutes have passed, and the fetal heart rate remains in the 80s. What additional nursing measures should you take? a. Call for staff assistance. b. Insert a Foley catheter. c. Start Pitocin. d. Notify the care provider immediately.

ANS: D To relieve an FHR deceleration, the nurse can reposition the mother, increase IV fluid, and provide oxygen. If oxytocin is infusing, it should be discontinued. If the FHR does not resolve, the primary care provider should be notified immediately. Inserting a Foley catheter is an inappropriate nursing action. If the FHR were to continue in a nonreassuring pattern, a cesarean section could be warranted, which would require a Foley catheter. However, the physician must make that determination. Pitocin may place additional stress on the fetus.

18. Maternity nurses often have to answer questions about the many, sometimes unusual ways people have tried to make the birthing experience more comfortable. For instance, nurses should be aware that: a. music supplied by the support person has to be discouraged because it could disturb others or upset the hospital routine. b. women in labor can benefit from sitting in a bathtub, but they must limit immersion to no longer than 15 minutes at a time. c. effleurage is permissible, but counterpressure is almost always counterproductive. d. electrodes attached to either side of the spine to provide high-intensity electrical impulses facilitate the release of endorphins.

ANS: D Transcutaneous electrical nerve stimulation does help. Music may be very helpful for reducing tension and certainly can be accommodated by the hospital. Women can stay in a bath as long as they want, although repeated baths with breaks may be more effective than a long soak. Counterpressure can help the woman cope with lower back pain.

5. A 40-year-old woman is 10 weeks pregnant. Which diagnostic tool would be appropriate to suggest to her at this time? A. Biophysical profile (BPP) B. Amniocentesis C. Maternal serum alpha-fetoprotein (MSAFP) screening D. Transvaginal ultrasound

ANS: D Ultrasound would be performed at this gestational age for biophysical assessment of the infant. BPP would be a method of biophysical assessment of fetal well-being in the third trimester. Amniocentesis is performed after the 14th week of pregnancy. MSAFP screening is performed from week 15 to week 22 of gestation (weeks 16 to 18 are ideal).

32. To determine the cultural influence on a patient's diet, the nurse should first: A. evaluate the patient's weight gain during pregnancy. B. assess the socioeconomic status of the patient. C. discuss the four food groups with the patient. D. identify the food preferences and methods of food preparation common to that culture.

ANS: D Understanding the patient's food preferences and how she prepares food will assist the nurse in determining whether the patient's culture is adversely affecting her nutritional intake. Evaluation of a patient's weight gain during pregnancy should be included for all patients, not just for patients who are culturally different. The socioeconomic status of the patient may alter the nutritional intake but not the cultural influence. Teaching the food groups to the patient should come after assessing food preferences.

1. A man smokes two packs of cigarettes a day. He wants to know if smoking is contributing to the difficulty he and his wife are having getting pregnant. The nurse's most appropriate response is: A. "Your sperm count seems to be okay in the first semen analysis." B. "Only marijuana cigarettes affect sperm count." C. "Smoking can give you lung cancer, even though it has no effect on sperm." D. "Smoking can reduce the quality of your sperm."

ANS: D Use of tobacco, alcohol, and marijuana may affect sperm counts. "Your sperm count seems to be okay in the first semen analysis" is inaccurate. Sperm counts vary from day to day and depend on emotional and physical status and sexual activity. A single analysis may be inconclusive. A minimum of two analyses must be performed several weeks apart to assess male fertility.

23. In documenting labor experiences, nurses should know that a uterine contraction is described according to all these characteristics except: a. frequency (how often contractions occur). b. intensity (the strength of the contraction at its peak). c. resting tone (the tension in the uterine muscle). d. appearance (shape and height).

ANS: D Uterine contractions are described in terms of frequency, intensity, duration, and resting tone.

6. A pregnant woman is at 38 weeks of gestation. She wants to know whether there are any signs that "labor is getting close to starting." Which finding is an indication that labor may begin soon? a. Weight gain of 1.5 to 2 kg (3 to 4 lb) b. Increase in fundal height c. Urinary retention d. Surge of energy

ANS: D Women speak of having a burst of energy before labor. The woman may lose 0.5 to 1.5 kg, as a result of water loss caused by electrolyte shifts that, in turn, are caused by changes in the estrogen and progesterone levels. When the fetus descends into the true pelvis (called lightening), the fundal height may decrease. Urinary frequency may return before labor.

19. Which vitamins or minerals can lead to congenital malformations of the fetus if taken in excess by the mother? A. Zinc B. Vitamin D C. Folic acid D. Vitamin A

ANS: D Zinc, vitamin D, and folic acid are vital to good maternal and fetal health and are highly unlikely to be consumed in excess. Vitamin A taken in excess causes a number of problems. An analog of vitamin A appears in prescribed acne medications, which must not be taken during pregnancy.

2. When a nurse is counseling a woman for primary dysmenorrhea, which nonpharmacologic intervention might be recommended? A. Increasing the intake of red meat and simple carbohydrates B. Reducing the intake of diuretic foods such as peaches and asparagus C. Temporarily substituting physical activity for a sedentary lifestyle D. Using a heating pad on the abdomen to relieve cramping

ANS: DHeat minimizes cramping by increasing vasodilation and muscle relaxation and minimizing uterine ischemia. Dietary changes such as eating less red meat may be recommended for women experiencing dysmenorrhea. Increasing the intake of diuretics, including natural diuretics such as asparagus, cranberry juice, peaches, parsley, and watermelon, may help ease the symptoms associated with dysmenorrhea. Exercise has been found to help relieve menstrual discomfort through increased vasodilation and subsequent decreased ischemia.

What action does the nurse take to relieve choking in a pregnant patient who is in the third trimester? A. Administering anesthesia B. Administering chest thrusts C. Placing a towel under the hips D. Positioning the patient onto one side

B (Choking is often relieved in patients by administering abdominal thrusts. However if the patient is in the third trimester of pregnancy, chest thrusts are administered to prevent injury to the uterus. Administering anesthesia or positioning the patient onto one side will not help dislodge the object and relieve choking. The nurse needs to place a towel under the hips to displace the uterus while administering cardiopulmonary resuscitation (CPR).)

The nurse observes that a pregnant patient with gestational hypertension who is on magnesium sulfate therapy is prescribed nifedipine (Adalat). What action does the nurse take? A. Evaluates the patient's renal function test B. Obtains a prescription for a change of drug C. Reduces the nifedipine (Adalat) dose by 50% D. Administers both medications simultaneously

B (Concurrent use of nifedipine (Adalat) and magnesium sulfate can result in skeletal muscle blockade in the patient. Therefore the nurse needs to report immediately to the primary health care provider (PHP) and obtain a prescription for a change of drug. The nurse assesses the patient's renal function to determine the risk for toxicity after administering any drug. However, it is not a priority in this case. Reducing the nifedipine (Adalat) dose is not likely to prevent the drug interaction in the patient. The nurse does not administer both drugs simultaneously because it may be harmful for the patient.)

The nurse is measuring a contraction from the beginning of the increment to the end of the decrement for the same contraction, documenting this as which of the following? a) Peak b) Duration c) Frequency d) Intensity

B (Duration Rationale: Duration refers to how long a contraction lasts and is measured from the beginning of the increment to the end of the decrement for the same contraction. Intensity refers to the strength of the contraction determined by manual palpation or measured by an internal intrauterine catheter. Frequency refers to how often contractions occur and is measured from the increment of one contraction to the increment of the next contraction. The peak or acme of a contraction is the highest intensity of a contraction.)

Which is an important nursing intervention when a patient has an incomplete miscarriage with heavy bleeding? A. Initiate expectant management at once. B. Prepare the patient for dilation and curettage. C. Administer the prescribed oxytocin (Pitocin). D. Obtain a prescription for ergonovine (Methergine).

B (In the case of an incomplete miscarriage, sometimes there is heavy bleeding and excessive cramping and some part of fetal tissue remains in the uterus. Therefore the nurse needs to prepare the patient for dilation and curettage for the removal of the fetal tissue. Expectant management is initiated if the pregnancy continues after a threatened miscarriage. Oxytocin (Pitocin) is administered to prevent hemorrhage after evacuation of the uterus. Ergonovine (Methergine) is administered to contract the uterus.)

The nurse assesses a client in labor and finds that the fetal long axis is longitudinal to the maternal long axis. The nurse documents this finding as which of the following? a) Attitude b) Lie c) Position d) Presentation

B (Lie Rationale: The nurse is assessing fetal lie, the relationship of the fetal long axis to the maternal long axis. When the fetal long axis is longitudinal to the maternal long axis, the lie is said to longitudinal. Presentation is the portion of the fetus that overlies the maternal pelvic inlet. Attitude is the relationship of the different fetal parts to one another. Position is the relationship of the fetal denominator to the different sides of the maternal pelvis.)

A woman with severe preeclampsia is being treated with an intravenous infusion of magnesium sulfate. This treatment is considered successful if: A. blood pressure is reduced to prepregnant baseline. B. seizures do not occur. C. deep tendon reflexes become hypotonic. D. diuresis reduces fluid retention

B (Magnesium sulfate is a central nervous system (CNS) depressant given primarily to prevent seizures . A temporary decrease in blood pressure can occur; however, this is not the purpose of administering this medication. Hypotonia is a sign of an excessive serum level of magnesium. It is critical that calcium gluconate be on hand to counteract the depressant effects of magnesium toxicity. Diuresis is not an expected outcome of magnesium sulfate administration.)

The most prevalent clinical manifestation of abruptio placentae (as opposed to placenta previa) is: A. bleeding. B. intense abdominal pain. C. uterine activity. D. cramping

B (Pain is absent with placenta previa but may be agonizing with abruptio placentae. Bleeding may be present in varying degrees for both placental conditions. Uterine activity may be present with both placental conditions. Cramping is a form of uterine activity that may be present in both placental conditions.)

Which intervention does the nurse implement for a patient immediately after a severe abdominal trauma? A. Prep the patient for cesarean birth. B. Send the patient for pelvic computed tomography (CT) scanning. C. Provide fluids to the patient as part of the protocol for ultrasound examination. D. Prepare to administer Rho(D) immunoglobulin.

B (Pelvic CT scanning helps visualize extraperitoneal and retroperitoneal structures and the genitourinary tract. The nurse needs to prepare the patient for cesarean birth if there is no evidence of a maternal pulse. Ultrasound examination is not as effective as electronic fetal monitoring for determining placental abruption in the patient after the trauma. Therefore the nurse prepares the patient for a CT scan after a severe abdominal trauma. The nurse needs to administer Rho(D) immunoglobulin in an Rh-negative pregnant trauma patient. This helps protect the patient from isoimmunization.)

A 32-year-old woman presents to the labor-and-delivery suite in active labor. She is multigravida, relaxed, and talking with her husband. When examined by the nurse, the fetus is found to be in a cephalic presentation. His occiput is facing toward the front and slightly to the right of the mother's pelvis, and he is exhibiting a flexed attitude. How does the nurse document the position of the fetus? a) LOA b) ROA c) LOP d) ROP

B (Rationale: Document the fetal position in the clinical record using abbreviations (Box 8-1). The first letter describes the side of the maternal pelvis toward which the presenting part is facing ("R" for right and "L" for left). The second letter or abbreviation indicates the reference point ("O" for occiput, "Fr" for frontum, etc.). The last part of the designation specifies whether the presenting part is facing the anterior (A) or the posterior (P) portion of the pelvis, or whether it is in a transverse (T) position.)

A nurse is performing a vaginal examination of a woman in the early stages of labor. The woman has been at 2 cm dilated for the past 2 hours, but effacement has progressed steadily. Which of the following should the nurse do to best encourage the client regarding her progress? a) Don't mention anything to the client yet; wait for further dilatation to occur. b) Say, "you are still 2 cm dilated, but the cervix is thinning out nicely." c) Say, "you haven't dilated any further, but hang in there; it will happen eventually." d) Say, "there has been no further dilatation; effacement is progressing."

B (Say, "you are still 2 cm dilated, but the cervix is thinning out nicely." Rationale: Women are anxious to have frequent reports during labor, to reassure them everything is progressing well. If giving a progress report, remember most women are aware of the word dilatation but not effacement. Just saying, "no further dilatation", therefore, is a depressing report. "You're not dilated a lot more, but a lot of thinning is happening and that's just as important" is the same report given in a positive manner.)

A pregnant patient has a systolic blood pressure that exceeds 160 mm Hg. Which action should the nurse take for this patient? A. Administer magnesium sulfate intravenously. B. Obtain a prescription for antihypertensive medications. C. Restrict intravenous and oral fluids to 125 mL/hr. D. Monitor fetal heart rate (FHR) and uterine contractions (UCs).

B (Systolic blood pressure exceeding 160 mm Hg indicates severe hypertension in the patient. The nurse should alert the health care provider and obtain a prescription for antihypertensive medications, such as nifedipine (Adalat) and labetalol hydrochloride (Normodyne). Magnesium sulfate would be administered if the patient was experiencing eclamptic seizures. Oral and intravenous fluids are restricted when the patient is at risk for pulmonary edema. Monitoring FHR and UCs is a priority when the patient experiences a trauma so that any complications can be addressed immediately.)

To teach patients about the process of labor adequately, the nurse knows that which event is the best indicator of true labor? a. Bloody show b. Cervical dilation and effacement c. Fetal descent into the pelvic inlet d. Uterine contractions every 7 minutes

B (The conclusive distinction between true and false labor is that contractions of true labor cause progressive change in the cervix. Bloody show can occur before true labor. Fetal descent can occur before true labor. False labor may have contractions that occur this frequently; however, this is usually inconsistent.)

The nurse has received report regarding her patient in labor. The woman's last vaginal examination was recorded as 3 cm, 30%, and ?2-2. The nurse's interpretation of this assessment is that: a. The cervix is effaced 3 cm, it is dilated 30%, and the presenting part is 2 cm above the ischial spines. b. The cervix is 3 cm dilated, it is effaced 30%, and the presenting part is 2 cm above the ischial spines. c. The cervix is effaced 3 cm, it is dilated 30%, and the presenting part is 2 cm below the ischial spines. d. The cervix is dilated 3 cm, it is effaced 30%, and the presenting part is 2 cm below the ischial spines.

B (The correct description of the vaginal examination for this woman in labor is the cervix is 3 cm dilated, it is effaced 30%, and the presenting part is 2 cm above the ischial spines. The sterile vaginal examination is recorded as centimeters of cervical dilation, percentage of cervical dilation, and the relationship of the presenting part to the ischial spines (either above or below).)

A pregnant patient is at risk for cardiac arrest as a result of profound hypovolemia after a trauma. Which action does the nurse take? The nurse: A. Assesses airway, breathing, and pulse rate. B. Administers warmed crystalloid solutions. C. Administers calcium gluconate intravenously. D. Obtains a prescription for magnesium sulfate.

B (The nurse administers warmed crystalloid solutions for massive fluid resuscitation in the patient who has profound hypovolemia after a trauma. The nurse needs to assess the airway, breathing, and pulse in a patient after a convulsion so that prompt actions can be taken to stabilize the patient. The nurse administers calcium gluconate as an antidote to a patient who has magnesium toxicity. The nurse may administer magnesium sulfate for the treatment of eclamptic seizures in a patient with preeclampsia.)

As relates to fetal positioning during labor, nurses should be aware that: a. Position is a measure of the degree of descent of the presenting part of the fetus through the birth canal. b. Birth is imminent when the presenting part is at +4 to +5 cm below the spine. c. The largest transverse diameter of the presenting part is the suboccipitobregmatic diameter. d. Engagement is the term used to describe the beginning of labor.

B (The station of the presenting part should be noted at the beginning of labor so that the rate of descent can be determined. Position is the relation of the presenting part of the fetus to the four quadrants of the mother's pelvis; station is the measure of degree of descent. The largest diameter usually is the biparietal diameter. The suboccipitobregmatic diameter is the smallest, although one of the most critical. Engagement often occurs in the weeks just before labor in nulliparas and before or during labor in multiparas.)

A 24-year-old primipara, 10 weeks pregnant, who has been experiencing vomiting every morning for the past few weeks, asks the nurse at her check-up how long this "morning sickness" will continue. Which statement by the nurse is most accurate? A. "It will end by the 15th week of pregnancy." B. "It usually subsides by the 20th week of pregnancy." C. "It's a very common but not serious problem." D. "In some women, it can last throughout the pregnancy and become serious."

B (This discomfort of pregnancy usually subsides by the 20th week of pregnancy. An absolute definite end of vomiting during pregnancy can never be stated. Test-Taking Tip: Identifying content and what is being asked about that content is critical to your choosing the correct response. Be alert for words in the stem of the item that are the same or similar in nature to those in one or two of the options. Example: If the item relates to and identifies stroke rehabilitation as its focus and only one of the options contains the word stroke in relation to rehabilitation, you are safe in identifying this choice as the correct response.)

The most prevalent clinical manifestation of abruptio placentae (as opposed to placenta previa) is: A. bleeding. B. intense abdominal pain. C. uterine activity. D. cramping.

B (intense abdominal pain. Bleeding may be present in varying degrees for both placental conditions. Pain is absent with placenta previa and may be agonizing with abruptio placentae. Uterine activity may be present with both placental conditions. Cramping is a form of uterine activity that may be present in both placental conditions.)

A woman with severe preeclampsia is being treated with an IV infusion of magnesium sulfate. This treatment is considered successful if: A. blood pressure is reduced to prepregnant baseline. B. seizures do not occur. C. deep tendon reflexes become hypotonic. D. diuresis reduces fluid retention.

B (seizures do not occur. A temporary decrease in blood pressure can occur; however, this is not the purpose of administering this medication. Magnesium sulfate is a central nervous system (CNS) depressant given primarily to prevent seizures. Hypotonia is a sign of an excessive serum level of magnesium. It is critical that calcium gluconate be on hand to counteract the depressant effects of magnesium toxicity. Diuresis is not an expected outcome of magnesium sulfate administration.)

A new client and her partner arrive on the labor, delivery, recovery, and postpartum unit for the birth of their first child. You apply the electronic fetal monitor (EFM) to the woman. Her partner asks you to explain what is printing on the graph, referring to the EFM strip. He wants to know what the baby's heart rate should be. Your best response is: a. "Don't worry about that machine; that's my job." b. "The top line graphs the baby's heart rate. Generally the heart rate is between 110 and 160. The heart rate will fluctuate in response to what is happening during labor." c. "The top line graphs the baby's heart rate, and the bottom line lets me know how strong the contractions are." d. "Your doctor will explain all of that later."

B "The top line graphs the baby's heart rate. Generally the heart rate is between 110 and 160. The heart rate will fluctuate in response to what is happening during labor" educates the partner about fetal monitoring and provides support and information to alleviate his fears. "Don't worry about that machine; that's my job" discredits the partner's feelings and does not provide the teaching he is requesting. "The top line graphs the baby's heart rate, and the bottom line lets me know how strong the contractions are" provides inaccurate information and does not address the partner's concerns about the fetal heart rate. The EFM graphs the frequency and duration of the contractions, not the intensity. Nurses should take every opportunity to provide client and family teaching, especially when information is requested.

What correctly matches the type of deceleration with its likely cause? a. Early deceleration—umbilical cord compression b. Late deceleration—uteroplacental inefficiency c. Variable deceleration—head compression d. Prolonged deceleration—cause unknown

B Late deceleration is caused by uteroplacental inefficiency. Early deceleration is caused by head compression. Variable deceleration is caused by umbilical cord compression. Prolonged deceleration has a variety of either benign or critical causes.

What three measures should the nurse implement to provide intrauterine resuscitation? Select the response that best indicates the priority of actions that should be taken. a. Call the provider, reposition the mother, and perform a vaginal examination. b. Reposition the mother, increase intravenous (IV) fluid, and provide oxygen via face mask. c. Administer oxygen to the mother, increase IV fluid, and notify the care provider. d. Perform a vaginal examination, reposition the mother, and provide oxygen via face mask.

B Repositioning the mother, increasing intravenous (IV) fluid, and providing oxygen via face mask are correct nursing actions for intrauterine resuscitation. The nurse should initiate intrauterine resuscitation in an ABC manner, similar to basic life support. The first priority is to open the maternal and fetal vascular systems by repositioning the mother for improved perfusion. The second priority is to increase blood volume by increasing the IV fluid. The third priority is to optimize oxygenation of the circulatory volume by providing oxygen via face mask. If these interventions do not resolve the fetal heart rate issue quickly, the primary provider should be notified immediately.

During labor a fetus with an average heart rate of 135 beats/min over a 10-minute period would be considered to have: a. Bradycardia. c. Tachycardia. b. A normal baseline heart rate. d. Hypoxia.

B The baseline heart rate is measured over 10 minutes; a normal range is 110 to 160 beats/min. Bradycardia is a fetal heart rate (FHR) below 110 beats/min for 10 minutes or longer. Tachycardia is an FHR over 160 beats/min for 10 minutes or longer. Hypoxia is an inadequate supply of oxygen; no indication of this condition exists with a baseline heart rate in the normal range.

Why is continuous electronic fetal monitoring usually used when oxytocin is administered? a. The mother may become hypotensive. b. Uteroplacental exchange may be compromised. c. Maternal fluid volume deficit may occur. d. Fetal chemoreceptors are stimulated.

B The uterus may contract more firmly, and the resting tone may be increased with oxytocin use. This response reduces entrance of freshly oxygenated maternal blood into the intervillous spaces, thus depleting fetal oxygen reserves. Hypotension is not a common side effect of oxytocin. All laboring women are at risk for fluid volume deficit; oxytocin administration does not increase the risk. Oxytocin affects the uterine muscles.

The emergency department nurse is assessing a pregnant trauma victim who just arrived at the hospital. What are the nurse's MOST appropriate actions? (Select all that apply.) A. Place the patient in a supine position. B. Assess for point of maximal impulse at fourth intercostal space. C. Collect urine for urinalysis and culture. D. Frequent vital sign monitoring. E. Assist with ambulation to decrease risk of thrombosis.

B, C, D (Passive regurgitation may occur if patient is supine, leading to high risk for aspiration. Placental perfusion is decreased when the patient is in a supine position as well. The heart is displaced upward and to the left in pregnant patients. During pregnancy, there is dilation of the ureters and urethra, and the bladder is displaced forward placing the pregnant trauma patient at higher risk for urinary stasis, infection, and bladder trauma. The trauma patient can suffer blood loss and other complications, necessitating frequent monitoring of vital signs. While the pregnant patient is at risk for thrombus formation, the patient must be cleared by the health care provider before ambulating. The pregnant trauma patient is at higher risk for pelvic fracture, and therefore this condition must be ruled out first as well.)

What does the nurse include in the plan of care of a pregnant patient with mild preeclampsia? Select all that apply. A. Ensure prolonged bed rest. B. Provide diversionary activities. C. Encourage the intake of more fluids. D. Restrict sodium and zinc in the diet. E. Refer to Internet-based support group

B, C, E (Activity is restricted in patients with preeclampsia, so it is necessary to provide diversionary activities to such patients to prevent boredom. The nurse encourages the patient to increase fluid intake to enhance renal perfusion and bowel function. The nurse can suggest Internet-based support groups to reduce boredom and stress in the patient. Patients need to restrict activity, but complete bed rest is not advised because it may cause cardiovascular deconditioning, muscle atrophy, and psychological stress. The patient needs to include adequate zinc and sodium in the diet for proper fetal development.)

A pregnant woman presents to the emergency department complaining of persistent nausea and vomiting. She is diagnosed with hyperemesis gravidarum. The nurse should include which information when teaching about diet for hyperemesis? (Select all that apply.) A. Eat three larger meals a day. B. Eat a high-protein snack at bedtime. C. Ice cream may stay down better than other foods. D. Avoid ginger tea or sweet drinks. E. Eat what sounds good to you even if your meals are not well-balanced.

B, C, E (The diet for hyperemesis includes: • Avoid an empty stomach. Eat frequently, at least every 2 to 3 hours. Separate liquids from solids and alternate every 2 to 3 hours. • Eat a high-protein snack at bedtime. • Eat dry, bland, low-fat, and high-protein foods. Cold foods may be better tolerated than those served at a warm temperature. • In general eat what sounds good to you rather than trying to balance your meals. • Follow the salty and sweet approach; even so-called junk foods are okay. • Eat protein after sweets. • Dairy products may stay down more easily than other foods. • If you vomit even when your stomach is empty, try sucking on a Popsicle. • Try ginger tea. Peel and finely dice a knuckle-sized piece of ginger and place it in a mug of boiling water. Steep for 5 to 8 minutes and add brown sugar to taste. • Try warm ginger ale (with sugar, not artificial sweetener) or water with a slice of lemon. • Drink liquids from a cup with a lid.)

A pregnant woman presents to the emergency department complaining of persistent nausea and vomiting. She is diagnosed with hyperemesis gravidarum. The nurse should include which information when teaching about diet for hyperemesis? Select all that apply. A. Eat three larger meals a day. B. Eat a high-protein snack at bedtime. C. Ice cream may stay down better than other foods. D. Avoid ginger tea or sweet drinks. E. Eat what sounds good to you even if your meals are not well-balanced.

B, C, E (The diet for hyperemesis includes: (1) Avoid an empty stomach. Eat frequently, at least every 2 to 3 hours. Separate liquids from solids and alternate every 2 to 3 hours. (2) Eat a high-protein snack at bedtime. (3) Eat dry, bland, low-fat, and high-protein foods. Cold foods may be better tolerated than those served at a warm temperature. (4) In general eat what sounds good to you rather than trying to balance your meals. (5) Follow the salty and sweet approach; even so-called junk foods are okay. (6) Eat protein after sweets. (7) Dairy products may stay down more easily than other foods. (8) If you vomit even when your stomach is empty, try sucking on a Popsicle. (9) Try ginger tea. Peel and finely dice a knuckle-sized piece of ginger and place it in a mug of boiling water. Steep for 5 to 8 minutes and add brown sugar to taste. (10) Try warm ginger ale (with sugar, not artificial sweetener) or water with a slice of lemon. (11) Drink liquids from a cup with a lid.)

Diabetes in pregnancy puts the fetus at risk in several ways. Nurses should be aware that: A. with good control of maternal glucose levels, sudden and unexplained stillbirth is no longer a major concern. B. the most important cause of perinatal loss in diabetic pregnancy is congenital malformations. C. infants of mothers with diabetes have the same risks for respiratory distress syndrome because of the careful monitoring. D. at birth, the neonate of a diabetic mother is no longer at any greater risk.

B. the most important cause of perinatal loss in diabetic pregnancy is congenital malformations. Even with good control, sudden and unexplained stillbirth remains a major concern. Congenital malformations account for 30% to 50% of perinatal deaths. Infants of diabetic mothers are at increased risk for respiratory distress syndrome. The transition to extrauterine life often is marked by hypoglycemia and other metabolic abnormalities.

The nurse should initially implement which intervention when a nulliparous woman telephones the hospital to report that she is in labor. a) Emphasize that food and fluid should stop or be light b) Tell the woman to stay home until her membranes rupture c) Ask the woman to describe why she believes that she is in labor d) Arrange for the woman to come to the hospital for labor evaluation

C (Ask the woman to describe why she believes that she is in labor Rationale: The nurse needs further information to assist in determining if the woman is in true or false labor. She will need to ask the patient questions to seek further assessment and triage information. Having her wait until membranes rupture may be dangerous, as she may give birth before reaching the hospital. She should continue fluid intake until it is determined whether or not she is in labor. She may be in false labor, and more information should be obtained before she is brought to the hospital.)

When assessing a woman in labor, the nurse is aware that the relationship of the fetal body parts to one another is called fetal: a. Lie. b. Presentation. c. Attitude. d. Position.

C (Attitude is the relation of the fetal body parts to one another. Lie is the relation of the long axis (spine) of the fetus to the long axis (spine) of the mother. Presentation refers to the part of the fetus that enters the pelvic inlet first and leads through the birth canal during labor at term. Position is the relation of the presenting part to the four quadrants of the mother's pelvis.)

The nurse observes that maternal hypotension has decreased uterine and fetal perfusion in a pregnant patient. What does the nurse need to assess further to understand the maternal status? A. D-dimer blood test B. Kleihauer-Betke (KB) test C. Electronic fetal monitoring D. Electrocardiogram reading

C (Electronic fetal monitoring reflects fetal cardiac responses to hypoxia and hypoperfusion and helps to assess maternal status after a trauma. The D-dimer blood test is used to rule out the presence of a thrombus. The KB test is used to evaluate transplacental hemorrhage. Electrocardiogram reading is more useful to assess the cardiac functions in nonpregnant cardiac patients.)

The skull is the most important factor in relation to the labor and birth process. The fetal skull must be small enough to travel through the bony pelvis. What feature of the fetal skull helps to make this passage possible? a) Vertex presentation b) Caput succedaneum c) Molding d) Cephalohematoma

C (Molding Rationale: The cartilage between the bones allows the bones to overlap during labor, a process called molding that elongates the fetal skull thereby reducing the diameter of the head.)

The four essential components of labor are known as the "four P's". Which of the four P's involves the pelvis? a) Psyche b) Powers c) Passageway d) Passenger

C (Passageway Rationale: The passageway is one of the 4 P's and involves the pelvis, both bony pelvis and the soft tissues, cervix, and vagina. The "passenger" refers to the fetus. The primary powers are the involuntary contractions of the uterus, whereas the secondary powers come from the maternal abdominal muscles. The psyche refers to the mother's mental state.)

Which intervention will help prevent the risk of pulmonary edema in a pregnant patient with severe preeclampsia? A. Assess fetal heart rate (FHR) abnormalities regularly. B. Place the patient on bed rest in a darkened environment. C. Restrict total intravenous (I.V.) and oral fluids to 125 mL/hr. D. Ensure that magnesium sulfate is administered as prescribed.

C (Pulmonary edema may be seen in patients with severe preeclampsia. Therefore the nurse needs to restrict total intravenous (I.V.) and oral fluids to 125 mL/hr. FHR monitoring helps assess any fetal complications. The patient is placed on bed rest in a darkened environment to prevent stress. Magnesium sulfate is administered to prevent eclamptic seizures.)

In preparing for the actual birth, which fetal presentation would a nurse be least likely to find? a) Transverse lie b) Breech c) Shoulder d) Oblique lie

C (Shoulder Rationale: Shoulder presentations are the least likely to occur in less than 0.3 percent of all births. Approximately 97 percent of fetuses are in a cephalic presentation at the end of pregnancy. A longitudinal lie, in which the long axis of the fetus is parallel to the long axis of the mother, is the most common. When the fetus is in a transverse lie, the long axis of the fetus is perpendicular to the long axis of the woman. An oblique lie is in between the two.)

With regard to factors that affect how the fetus moves through the birth canal, nurses should be aware that: a. The fetal attitude describes the angle at which the fetus exits the uterus. b. Of the two primary fetal lies, the horizontal lie is that in which the long axis of the fetus is parallel to the long axis of the mother. c. The normal attitude of the fetus is called general flexion. d. The transverse lie is preferred for vaginal birth.

C (The normal attitude of the fetus is general flexion. The fetal attitude is the relation of fetal body parts to one another. The horizontal lie is perpendicular to the mother; in the longitudinal (or vertical) lie the long axes of the fetus and the mother are parallel. Vaginal birth cannot occur if the fetus stays in a transverse lie.)

A blunt abdominal trauma causes fetal hemorrhage in a pregnant patient. The nurse finds that the patient is Rh negative. What action does the nurse take? A. Initiate magnesium sulfate per protocol. B. Administer oxytocin (pitocin). C. Administer prescribed Rho (D) immunoglobulin. D. Prepare the patient for magnetic resonance imaging (MRI).

C (The nurse administers the prescribed Rho(D) immunoglobulin to the patient to protect the patient from isoimmunization. The nurse needs to obtain a prescription for magnesium sulfate if there are eclamptic seizures in a patient with preeclampsia. Oxytocin (Pitocin) is administered to prevent bleeding after birth or the evacuation of the uterus. Magnetic resonance imaging (MRI) is used to assess injuries in a patient after trauma.)

The nurse knows that the second stage of labor, the descent phase, has begun when: a) The amniotic membranes rupture b) The cervix cannot be felt during a vaginal examination c) The woman experiences a strong urge to bear down d) The presenting part is below the ischial spines

C (The woman experiences a strong urge to bear down)

A post delivery CBC has noted an elevated WBC count of 22,000/mm3. Which rationale is accurate regarding the elevated WBC count? a) Abnormal finding and she needs antibiotics b) Might be a false result, recommend re-testing c) This is a normal variation due to stress of labor d) Occurs in patients who have cesarean birth, from the trauma of surgery

C (This is a normal variation due to stress of labor Rationale: An elevation of WBC up to 30,000mm/3 can be normal variation for any woman after delivery. This is related to the stress on her body from labor and delivery. Antibiotics are not indicated as this is a normal response to intense stress. The increase in WBC is not related to cesarean birth. Retesting would be wasteful as it is known that this is a normal response to any stress.)

The maternity nurse understands that as the uterus contracts during labor, maternal-fetal exchange of oxygen and waste products: a. Continues except when placental functions are reduced. b. Increases as blood pressure decreases. c. Diminishes as the spiral arteries are compressed. d. Is not significantly affected.

C (Uterine contractions during labor tend to decrease circulation through the spiral electrodes and subsequent perfusion through the intervillous space. The maternal blood supply to the placenta gradually stops with contractions. The exchange of oxygen and waste products decreases. The exchange of oxygen and waste products is affected by contractions.)

Fetal bradycardia is most common during: a. Intraamniotic infection. b. Fetal anemia. c. Prolonged umbilical cord compression. d. Tocolytic treatment using terbutaline.

C Fetal bradycardia can be considered a later sign of fetal hypoxia and is known to occur before fetal death. Bradycardia can result from placental transfer of drugs, prolonged compression of the umbilical cord, maternal hypothermia, and maternal hypotension. Intraamniotic infection, fetal anemia, and tocolytic treatment using terbutaline would most likely result in fetal tachycardia.

The nurse caring for the woman in labor should understand that maternal hypotension can result in: a. Early decelerations. c. Uteroplacental insufficiency. b. Fetal dysrhythmias. d. Spontaneous rupture of membranes.

C Low maternal blood pressure reduces placental blood flow during uterine contractions and results in fetal hypoxemia. Maternal hypotension is not associated with early decelerations, fetal dysrhythmias, or spontaneous rupture of membranes.

The uterine contractions of a woman early in the active phase of labor are assessed by an internal uterine pressure catheter (IUPC). The nurse notes that the intrauterine pressure at the peak of the contraction ranges from 65 to 70 mm Hg and the resting tone range is 6 to 10 mm Hg. The uterine contractions occur every 3 to 4 minutes and last an average of 55 to 60 seconds. On the basis of this information, the nurse should: a. Notify the woman's primary health care provider immediately. b. Prepare to administer an oxytocic to stimulate uterine activity. c. Document the findings because they reflect the expected contraction pattern for the active phase of labor. d. Prepare the woman for the onset of the second stage of labor.

C The nurse is responsible for monitoring the uterine contractions to ascertain whether they are powerful and frequent enough to accomplish the work of expelling the fetus and the placenta. In addition, the nurse would document these findings in the client's medical record. This labor pattern indicates that the client is in the active phase of the first stage of labor. Nothing indicates a need to notify the primary care provider at this time. Oxytocin augmentation is not needed for this labor pattern; this contraction pattern indicates adequate active labor. Her contractions eventually will become stronger, last longer, and come closer together during the transition phase of the first stage of labor. The transition phase precedes the second stage of labor, or delivery of the fetus.

What is an advantage of external electronic fetal monitoring? a. The ultrasound transducer can accurately measure short-term variability and beat-to-beat changes in the fetal heart rate. b. The tocotransducer can measure and record the frequency, regularity, intensity, and approximate duration of uterine contractions (UCs). c. The tocotransducer is especially valuable for measuring uterine activity during the first stage of labor. d. Once correctly applied by the nurse, the transducer need not be repositioned even when the woman changes positions.

C The tocotransducer is especially valuable for measuring uterine activity during the first stage of labor, particularly when the membranes are intact. Short-term changes cannot be measured with this technology. The tocotransducer cannot measure and record the intensity of UCs. The transducer must be repositioned when the woman or fetus changes position.

The nurse providing care for the laboring woman should understand that late fetal heart rate (FHR) decelerations are the result of: a. Altered cerebral blood flow. c. Uteroplacental insufficiency. b. Umbilical cord compression. d. Meconium fluid.

C Uteroplacental insufficiency would result in late decelerations in the FHR. Altered fetal cerebral blood flow would result in early decelerations in the FHR. Umbilical cord compression would result in variable decelerations in the FHR. Meconium-stained fluid may or may not produce changes in the fetal heart rate, depending on the gestational age of the fetus and whether other causative factors associated with fetal distress are present.

Which of the following would be least effective in promoting a positive birth outcome for a woman in labor? a) Promoting the woman's feelings of control b) Providing clear information about procedures c) Encouraging the woman to use relaxation techniques d) Allowing the woman time to be alone

D (Allowing the woman time to be alone Rationale: Positive support, not being alone, promotes a positive birth experience. Being alone can increase anxiety and fear, decreasing the woman's ability to cope. Feelings of control promote self-confidence and self-esteem, which in turn help the woman to cope with the challenges of labor. Information about procedures reduces anxiety about the unknown and fosters cooperation and self-confidence in her abilities to deal with labor. Catecholamines are secreted in response to anxiety and fear and can inhibit uterine blood flow and placental perfusion. Relaxation techniques can help to reduce anxiety and fear, in turn decreasing the secretion of catecholamines and ultimately improving the woman's ability to cope with labor.)

In order to care for obstetric patients adequately, the nurse understands that labor contractions facilitate cervical dilation by: a. Contracting the lower uterine segment. b. Enlarging the internal size of the uterus. c. Promoting blood flow to the cervix. d. Pulling the cervix over the fetus and amniotic sac.

D (Effective uterine contractions pull the cervix upward at the same time that the fetus and amniotic sac are pushed downward. The contractions are stronger at the fundus. The internal size becomes smaller with the contractions; this helps to push the fetus down. Blood flow decreases to the uterus during a contraction.)

What does the nurse administer to a patient if there is excessive bleeding after suction curettage? A. Nifedipine (Procardia) B. Methyldopa (Aldomet) C. Hydralazine (Apresoline) D. Ergonovine (Methergine)

D (Ergonovine (Methergine) is an ergot product, which is administered to contract the uterus when there is excessive bleeding after suction curettage. Nifedipine (Procardia) is prescribed for gestational hypertension or severe preeclampsia. Methyldopa (Aldomet) is an antihypertensive medication indicated for pregnant patients with hypertension. Hydralazine (Apresoline) is also an antihypertensive medication used for treating hypertension intrapartum.)

What term is used to describe the position of the fetal long axis in relation to the long axis of the mother? a) Fetal position b) Fetal presentation c) Fetal attitude d) Fetal lie

D (Fetal lie Rationale: Fetal lie describes the position of the long axis of the fetus in relation to the long axis of the pregnant woman.)

The third stage of labor is considered to be expulsion of the placenta. This stage can last anywhere from five to 20 minutes. What is a sign that the placenta is separating from the wall of the uterus? a) Expulsion of blood clots b) Firm fundus c) Shortening of the umbilical cord d) Globular shape to the fundus

D (Globular shape to the fundus Rationale: Signs that indicate the placenta is separating from the uterine wall include a gush of blood, lengthening of the umbilical cord, and a globular shape to the fundus.)

A new OB/GYN physician has just finished evaluating her one hundredth patient. In reviewing the documentation from all patients thus far, which types of pelvis would you assume the physician has seen the most and the least? a) Android and platypelloid, respectively b) Gynecoid and android, respectively c) Anthropoid and gynecoid, respectively d) Gynecoid and platypelloid, respectively

D (Gynecoid and platypelloid, respectively Rationale: The gynecoid is the typical female pelvis shape; platypelloid pelvis is the least common type of pelvis in women. Women with anthropoid pelvic shapes are able to deliver normally one third of the time, and are somewhat rare. An android pelvis is similar to a male pelvis, and is seen in 16% of nonwhite women.)

A patient reports excessive vomiting in the first trimester of the pregnancy, which has resulted in nutritional deficiency and weight loss. The urinalysis report of the patient indicates ketonuria. Which disorder does the patient have? A. Preeclampsia B. Hyperthyroid disorder C. Gestational hypertension D. Hyperemesis gravidarum

D (Hyperemesis gravidarum is characterized by excessive vomiting during pregnancy, which causes nutritional deficiency and weight loss. The presence of ketonuria is another indication of this disorder. Preeclampsia refers to hypertension and proteinuria in patients after 20 weeks' gestation. Hyperthyroid disorder may be one of the causes of hyperemesis gravidarum. Gestational hypertension also develops after 20 weeks' gestation.)

Which fetal risk is associated with an ectopic pregnancy? A. Miscarriage B. Fetal anemia C. Preterm birth D. Fetal deformity

D (In an ectopic pregnancy, the risk for fetal deformity is high because of the pressure deformities caused by oligohydramnios. There may be facial or cranial asymmetry, various joint deformities, limb deficiency, and central nervous system (CNS) anomalies. Miscarriage is not likely to happen in an ectopic pregnancy. Instead, the patient is at risk for pregnancy-related death resulting from ectopic rupture. Fetal anemia is a risk associated with placenta previa. Preterm birth is not possible because the pregnancy is dissolved when it is diagnosed or a surgery is performed to remove the fetus.)

When going through the transition phase of labor women often feel out of control. What do women in the transition phase of labor need? a) Their significant other beside them b) Intense nursing care c) Just to be left alone d) Positive reinforcement

D (Positive reinforcement Rationale: Any women, even ones who have had natural childbirth classes, have a difficult time maintaining positive coping strategies during this phase of labor. Many women describe feeling out of control during this phase of labor. A woman in transition needs support, encouragement, and positive reinforcement.)

A 32-year-old woman presents to the labor-and-delivery suite in active labor. She is multigravida, relaxed, and talking with her husband. When examined by the nurse, the fetus is found to be in a cephalic presentation. His occiput is facing toward the front and slightly to the right of the mother's pelvis, and he is exhibiting a flexed attitude. How does the nurse document the position of the fetus? a) ROP b) LOP c) LOA d) ROA

D (ROA Rationale: Document the fetal position in the clinical record using abbreviations (Box 8-1). The first letter describes the side of the maternal pelvis toward which the presenting part is facing ("R" for right and "L" for left). The second letter or abbreviation indicates the reference point ("O" for occiput, "Fr" for frontum, etc.). The last part of the designation specifies whether the presenting part is facing the anterior (A) or the posterior (P) portion of the pelvis, or whether it is in a transverse (T) position.)

In order to accurately assess the health of the mother accurately during labor, the nurse should be aware that: a) The woman's blood pressure increases during contractions and falls back to prelabor normal between contractions b) Use of the Valsalva maneuver is encouraged during the second stage of labor to relieve fetal hypoxia c) Having the woman point her toes reduces leg cramps d) The endogenous endorphins released during labor raise the woman's pain threshold and produce sedation

D (The endogenous endorphins released during labor raise the woman's pain threshold and produce sedation.)

In order to evaluate the condition of the patient accurately during labor, the nurse should be aware that: a. The woman's blood pressure will increase during contractions and fall back to prelabor normal between contractions. b. Use of the Valsalva maneuver is encouraged during the second stage of labor to relieve fetal hypoxia. c. Having the woman point her toes will reduce leg cramps. d. The endogenous endorphins released during labor will raise the woman's pain threshold and produce sedation.

D (The endogenous endorphins released during labor will raise the woman's pain threshold and produce sedation. In addition, physiologic anesthesia of the perineal tissues, caused by the pressure of the presenting part, decreases the mother's perception of pain. Blood pressure increases during contractions but remains somewhat elevated between them. Use of the Valsalva maneuver is discouraged during second stage labor because of a number of unhealthy outcomes, including fetal hypoxia. Pointing the toes can cause leg cramps, as can the process of labor itself.)

A new mother asks the nurse when the "soft spot" on her son's head will go away. The nurse's answer is based on the knowledge that the anterior fontanel closes after birth by _____ months. a. 2 b. 12 c. 8 d. 18

D (The larger of the two fontanels, the anterior fontanel, closes by 18 months after birth.)

A primigravida at 39 weeks of gestation is observed for 2 hours in the intrapartum unit. The fetal heart rate has been normal. Contractions are 5 to 9 minutes apart, 20 to 30 seconds in duration, and of mild intensity. Cervical dilation is 1 to 2 cm and uneffaced (unchanged from admission). Membranes are intact. The nurse should expect the woman to be: a. Admitted and prepared for a cesarean birth. b. Admitted for extended observation. c. Discharged home with a sedative. d. Discharged home to await the onset of true labor.

D (This situation describes a woman with normal assessments who is probably in false labor and will probably not deliver rapidly once true labor begins. These are all indications of false labor without fetal distress. There is no indication that further assessment or cesarean birth is indicated. The patient will likely be discharged; however, there is no indication that a sedative is needed.)

A woman at 39 weeks of gestation with a history of preeclampsia is admitted to the labor and birth unit. She suddenly experiences increased contraction frequency of every 1 to 2 minutes; dark red vaginal bleeding; and a tense, painful abdomen. The nurse suspects the onset of: A. eclamptic seizure. B. rupture of the uterus. C. placenta previa. D. placental abruption.

D (placental abruption. Eclamptic seizures are evidenced by the presence of generalized tonic-clonic convulsions. Uterine rupture presents as hypotonic uterine activity, signs of hypovolemia, and in many cases the absence of pain. Placenta previa presents with bright red, painless vaginal bleeding. Uterine tenderness in the presence of increasing tone may be the earliest finding of premature separation of the placenta (abruptio placentae or placental abruption). Women with hypertension are at increased risk for an abruption.)

The most common cause of decreased variability in the fetal heart rate (FHR) that lasts 30 minutes or less is: a. Altered cerebral blood flow. c. Umbilical cord compression. b. Fetal hypoxemia. d. Fetal sleep cycles.

D A temporary decrease in variability can occur when the fetus is in a sleep state. These sleep states do not usually last longer than 30 minutes. Altered fetal cerebral blood flow would result in early decelerations in the FHR. Fetal hypoxemia would be evidenced by tachycardia initially and then bradycardia. A persistent decrease or loss of FHR variability may be seen. Umbilical cord compression would result in variable decelerations in the FHR.

Increasing the infusion rate of nonadditive intravenous fluids can increase fetal oxygenation primarily by: a. Maintaining normal maternal temperature. b. Preventing normal maternal hypoglycemia. c. Increasing the oxygen-carrying capacity of the maternal blood. d. Expanding maternal blood volume.

D Filling the mother's vascular system makes more blood available to perfuse the placenta and may correct hypotension. Increasing fluid volume may alter the maternal temperature only if she is dehydrated. Most intravenous fluids for laboring women are isotonic and do not provide extra glucose. Oxygen-carrying capacity is increased by adding more red blood cells.

Which fetal heart rate (FHR) finding would concern the nurse during labor? a. Accelerations with fetal movement c. An average FHR of 126 beats/min b. Early decelerations d. Late decelerations

D Late decelerations are caused by uteroplacental insufficiency and are associated with fetal hypoxemia. They are considered ominous if persistent and uncorrected. Accelerations in the FHR are an indication of fetal well-being. Early decelerations in the FHR are associated with head compression as the fetus descends into the maternal pelvic outlet; they generally are not a concern during normal labor.

When using intermittent auscultation (IA) for fetal heart rate, nurses should be aware that: a. They can be expected to cover only two or three clients when IA is the primary method of fetal assessment. b. The best course is to use the descriptive terms associated with electronic fetal monitoring (EFM) when documenting results. c. If the heartbeat cannot be found immediately, a shift must be made to EFM. d. Ultrasound can be used to find the fetal heartbeat and reassure the mother if initial difficulty was a factor.

D Locating fetal heartbeats often takes time. Mothers can be reassured verbally and by the ultrasound pictures if ultrasound is used to help locate the heartbeat. When used as the primary method of fetal assessment, auscultation requires a nurse-to-client ratio of one to one. Documentation should use only terms that can be numerically defined; the usual visual descriptions of EFM are inappropriate.

As a perinatal nurse you realize that a fetal heart rate that is tachycardic, is bradycardic, or has late decelerations or loss of variability is nonreassuring and is associated with: a. Hypotension. c. Maternal drug use. b. Cord compression. d. Hypoxemia.

D Nonreassuring heart rate patterns are associated with fetal hypoxemia. Fetal bradycardia may be associated with maternal hypotension. Fetal variable decelerations are associated with cord compression. Maternal drug use is associated with fetal tachycardia.

You are evaluating the fetal monitor tracing of your client, who is in active labor. Suddenly you see the fetal heart rate (FHR) drop from its baseline of 125 down to 80. You reposition the mother, provide oxygen, increase intravenous (IV) fluid, and perform a vaginal examination. The cervix has not changed. Five minutes have passed, and the fetal heart rate remains in the 80s. What additional nursing measures should you take? a. Scream for help. b. Insert a Foley catheter. c. Start Pitocin. d. Notify the care provider immediately.

D To relieve an FHR deceleration, the nurse can reposition the mother, increase IV fluid, and provide oxygen. If oxytocin is infusing, it should be discontinued. If the FHR does not resolve, the primary care provider should be notified immediately. Inserting a Foley catheter is an inappropriate nursing action. If the FHR were to continue in a nonreassuring pattern, a cesarean section could be warranted, which would require a Foley catheter. However, the physician must make that determination. Pitocin may place additional stress on the fetus.

A pregnant woman at 28 weeks of gestation has been diagnosed with gestational diabetes. The nurse caring for this client understands that: A. oral hypoglycemic agents can be used if the woman is reluctant to give herself insulin. B. dietary modifications and insulin are both required for adequate treatment. C. glucose levels are monitored by testing urine 4r times a day and at bedtime. D. dietary management involves distributing nutrient requirements over three meals and two or three snacks.

D. dietary management involves distributing nutrient requirements over three meals and two or three snacks. Oral hypoglycemic agents can be harmful to the fetus and less effective than insulin in achieving tight glucose control. In some women gestational diabetes can be controlled with dietary modifications alone. Blood, not urine, glucose levels are monitored several times a day. Urine is tested for ketone content; results should be negative. Small frequent meals over a 24-hour period help decrease the risk for hypoglycemia and ketoacidosis.

19. Which nutritional recommendation about fluids is accurate? a. A woman's daily intake should be eight to ten glasses (2.3 L) of water, milk, or juice. b. Coffee should be limited to no more than two cups, but tea and cocoa can be consumed without worry. c. Of the artificial sweeteners, only aspartame has not been associated with any maternity health concerns. d. Water with fluoride is especially encouraged because it reduces the child's risk of tooth decay.

a. A woman's daily intake should be eight to ten glasses (2.3 L) of water, milk, or juice.

17. Which statement regarding acronyms in nutrition is accurate? a. Dietary reference intakes (DRIs) consist of recommended dietary allowances (RDAs), adequate intakes (AIs), and upper limits (ULs). b. RDAs are the same as ULs except with better data. c. AIs offer guidelines for avoiding excessive amounts of nutrients. d. They all refer to green leafy vegetables, whole grains, and fruit.

a. Dietary reference intakes (DRIs) consist of recommended dietary allowances (RDAs), adequate intakes (AIs), and upper limits (ULs).

37. Identify a goal of a patient with the following nursing diagnosis: Imbalanced Nutrition: Less Than Body Requirements related to diet choices inadequate to meet nutrient requirements of pregnancy. a. Gain a total of 30 lb. b. Take daily supplements consistently. c. Decrease intake of snack foods. d. Increase intake of complex carbohydrates.

a. Gain a total of 30 lb.

18. With regard to protein in the diet of pregnant women, nurses should be aware that: a. Many protein-rich foods are also good sources of calcium, iron, and B vitamins. b. Many women need to increase their protein intake during pregnancy. c. As with carbohydrates and fat, no specific recommendations exist for the amount of protein in the diet. d. High-protein supplements can be used without risk by women on macrobiotic diets.

a. Many protein-rich foods are also good sources of calcium, iron, and B vitamins.

6. A pregnant woman reports that she is still playing tennis at 32 weeks of gestation. The nurse would be most concerned that during and after tennis matches this woman consumes: a. Several glasses of fluid. b. Extra protein sources such as peanut butter. c. Salty foods to replace lost sodium. d. Easily digested sources of carbohydrate.

a. Several glasses of fluid.

26. To help a woman reduce the severity of nausea caused by morning sickness, the nurse might suggest that she: a. Try a tart food or drink such as lemonade or salty foods such as potato chips. b. Drink plenty of fluids early in the day. c. Brush her teeth immediately after eating. d. Never snack before bedtime.

a. Try a tart food or drink such as lemonade or salty foods such as potato chips.

12. To prevent gastrointestinal upset, clients should be instructed to take iron supplements: a. On a full stomach. b. At bedtime. c. After eating a meal. d. With milk.

b. At bedtime.

16. Maternal nutritional status is an especially significant factor of the many factors that influence the outcome of pregnancy because: a. It is very difficult to adjust because of people's ingrained eating habits. b. It is an important preventive measure for a variety of problems. c. Women love obsessing about their weight and diets. d. A woman's preconception weight becomes irrelevant.

b. It is an important preventive measure for a variety of problems.

34. If a patient's normal prepregnancy diet contains 45 g of protein daily, how many more grams of protein should she consume per day during pregnancy? a. 5 b. 10 c. 25 d. 30

c. 25

32. When providing care to the prenatal patient, the nurse understands that pica is defined as: a. Intolerance of milk products b. Iron deficiency anemia c. Ingestion of nonfood substances d. Episodes of anorexia and vomiting

c. Ingestion of nonfood substances

20. Which minerals and vitamins usually are recommended to supplement a pregnant woman's diet? a. Fat-soluble vitamins A and D b. Water-soluble vitamins C and B6 c. Iron and folate d. Calcium and zinc

c. Iron and folate

15. Pregnant adolescents are at high risk for _____ because of lower body mass indices (BMIs) and "fad" dieting. a. Obesity b. Diabetes c. Low-birth-weight babies d. High-birth-weight babies

c. Low-birth-weight babies

23. While taking a diet history, the nurse might be told that the expectant mother has cravings for ice chips, cornstarch, and baking soda. This represents a nutritional problem known as: a. Preeclampsia. b. Pyrosis. c. Pica. d. Purging.

c. Pica.

25. The labor and delivery nurse is preparing a bariatric patient for an elective cesarean birth. Which piece of "specialized" equipment is unnecessary when providing care for this pregnant woman. a. Extra long surgical instruments b. Wide surgical table c. Temporal thermometer d. Increased diameter blood pressure cuff

c. Temporal thermometer

4. A pregnant woman's diet consists almost entirely of whole grain breads and cereals, fruits, and vegetables. The nurse would be most concerned about this woman's intake of: a. Calcium. b. Protein. c. Vitamin B12. d. Folic acid.

c. Vitamin B12.

30. The major source of nutrients in the diet of a pregnant woman should be composed of: a. Simple sugars b. Fats c. Fiber d. Complex carbohydrates

d. Complex carbohydrates

31. A pregnant woman's diet may not meet her need for folates. A good source of this nutrient is: a. Chicken b. Cheese c. Potatoes d. Green leafy vegetables

d. Green leafy vegetables

36. To determine the cultural influence on a patient's diet, the nurse should first: a. Evaluate the patient's weight gain during pregnancy b. Assess the socioeconomic status of the patient c. Discuss the four food groups with the patient d. Identify the food preferences and methods of food preparation common to that culture

d. Identify the food preferences and methods of food preparation common to that culture

1. A 22-year-old woman pregnant with a single fetus has a preconception body mass index (BMI) of 24. When she was seen in the clinic at 14 weeks of gestation, she had gained 1.8 kg (4 lb) since conception. How would the nurse interpret this? a. This weight gain indicates possible gestational hypertension. b. This weight gain indicates that the woman's infant is at risk for intrauterine growth restriction (IUGR). c. This weight gain cannot be evaluated until the woman has been observed for several more weeks. d. The woman's weight gain is appropriate for this stage of pregnancy.

d. The woman's weight gain is appropriate for this stage of pregnancy.

21. Which vitamins or minerals can lead to congenital malformations of the fetus if taken in excess by the mother? a. Zinc b. Vitamin D c. Folic acid d. Vitamin A

d. Vitamin A

29. Which pregnant woman should restrict her weight gain during pregnancy? a. Woman pregnant with twins b. Woman in early adolescence c. Woman shorter than 62 inches or 157 cm d. Woman who was 20 pounds overweight before pregnancy

d. Woman who was 20 pounds overweight before pregnancy


Ensembles d'études connexes

Introduction to Literature shorter version

View Set

Week 2: Software Process - CS 427

View Set

Topics 4 and 5: Kinetic Theory and Thermodynamics

View Set

Dr. Britt PSYCH 101 Exam 2 Chps 3-4

View Set

Chapter 17 Eye and Ear Diseases/ Disorders

View Set

CH 30: Management of Patients with Hematologic Neoplasms

View Set